You are on page 1of 303



Question bank
PAST-PAPERS
‫‪‬إ‪‬اء‪‬‬
‫‪ ‬آﯾﺔ ﻣﻦ ﻛﺘﺎب ﷲ ﺣﻔﻈﻨﺎھﺎ وﻟﻜﻞ ﻣﻌﻠﻮﻣﺔ ﻗﺮأﻧﺎھﺎ وﻧﺴﯿﻨﺎھﺎ ﻟﻌﻞ ﷲ ﯾﺤﻤﯿﻨﺎ ﻣﻦ اﻟﻨﺴﯿﺎن وﯾﺮدھﺎ إﻟﯿﻨﺎ‬
‫‪ ‬ﺷﺨﺺ دﻋﻮﻧﺎ ﻟﮫ ﻓﻲ ظﮭﺮاﻟﻐﯿﺐ ﻟﻌﻞ ﷲ أن ﯾﺴﺘﺠﯿﺐ دﻋﺎﺋﻨﺎ‬
‫‪ ‬ﻣﺮﯾﺾ ﻗﺪ آﻟﻤﮫ ﻣﺎ أﻟﻤﮫ ﻟﻌﻞ ﷲ ﯾﺸﻔﯿﮫ وﯾﻌﯿﺪ إﻟﯿﮫ ﺻﺤﺘﮫ وﻋﺎﻓﯿﺘﮫ‬
‫‪ ‬ﻟﺤﻈﺔ ﺗﺂﻟﻢ ﺑﮭﺎ أﺧﻲ وﺟﺪي وأﺣﺒﺘﻲ أﺛﻨﺎء ﻋﻼﺟﮭﻢ ﻟﻌﻞ ﷲ ﻻ ﯾﺮﯾﮭﻢ اﻵﻟﻢ وﻻ ﯾﻀﯿﻤﮭﻢ ﻣﺮة أﺧﺮى‬
‫‪ ‬ﻣﻮﻗﻒ اﺣﺘﺠﺖ ﺑﮫ ﻋﯿﻨﺎي ﻓﻠﻢ ﺗﻠﺒﯿﺎﻧﻲ ﻟﻌﻞ ﷲ ﻻ ﯾﻀﺮﻧﻲ ﺑﮭﻤﺎ ﻓﻲ ﺣﯿﺎﺗﻲ‬
‫‪ ‬ﺣﺎﻟﻢ ﺻﺎﺣﺐ أھﺪاف وﻣﺒﺎدئ ﻟﻌﻞ ﷲ ﯾﺤﻘﻖ ﻟﮫ ﻣﻘﺎﺻﺪه ﺣﺘﻰ ﯾﺮﺿﯿﮫ وﯾﺮﺿﻰ ﻋﻨﮫ‬
‫‪ ‬ﻣﺠﺘﮭﺪ ﻟﻌﻞ ﷲ ﯾﺠﻌﻞ ﻟﮫ ﻣﻦ اﻟﺘﻮﻓﯿﻖ أوﻓﺮ ﺣﻆ و ﻧﺼﯿﺐ وﯾﮭﻮن ﻟﮫ اﻷﺳﺒﺎب ﻛﻠﮭﺎ ﻣﻦ ﺣﯿﺚ ﻻ‬
‫ﯾﺪري ﻣﻦ ﻏﯿﺮ ﺿﺮاء ﻣﻀﺮة وﻻ ﻓﺘﻨﺔ ﻣﻀﻠﮫ‬
‫‪ ‬ﻣﻦ ﻓﻘﺪ ﺷﯿﺌﺎ ﻓﻲ اﻟﺤﯿﺎة ﻟﻌﻞ ﷲ ﯾﻌﯿﺪه إﻟﯿﮫ أو ﯾﻌﻮﺿﮫ ﺑﻤﺎ ﯾﺴﻌﺪه‬
‫‪ ‬ﺷﺨﺺ اﺑﺘﻌﺪﻧﺎ ﻋﻨﮫ ﻟﻌﻞ ﷲ ﯾﺠﻤﻌﮫ ﺑﻤﻦ ھﻢ أﻓﻀﻞ ﻣﻨﺎ‬
‫وأ‪‬ا وﻟﯿﺲ آﺧﺮا ﻷھﻠﻨﺎ ﻟﻌﻞ ﷲ ﯾﺠﻌﻠﮭﺎ ﻟﮭﻢ ﺻﺪﻗﺔ ﺟﺎرﯾﮫ ﺗﺮد إﻟﯿﮭﻢ ﻗﻠﯿﻼ ﻣﻦ ﻓﻀﻠﮭﻢ ﻋﻠﯿﻨﺎ وﺗﻜﻮن ﺳﺒﺒﺎ‬
‫ﯾﻠﻢ ﺷﻤﻠﻨﺎ ﺗﺤﺖ ﺳﻘﻒ واﺣﺪ ﺑﻌﺪ أن ھ ﱠﺠ َﺮﺗﻨﺎ اﻟﺤﺮوب واﻟﻤﺼﺎﺋﺐ‬

‫‪PT Part I 2019 index‬‬


‫‪Branch‬‬ ‫‪No. of Q‬‬
‫‪Cardiology‬‬ ‫)‪(386‬‬
‫‪Clinical haematology/oncology‬‬ ‫)‪(392‬‬
‫‪Clinical‬‬
‫)‪(513‬‬
‫‪pharmacology/therapeutics/toxicology‬‬
‫‪Dermatology‬‬ ‫)‪(138‬‬
‫‪Endocrinology‬‬ ‫)‪(414‬‬
‫‪Gastroenterology‬‬ ‫)‪(403‬‬
‫‪General Revision‬‬ ‫)‪(122‬‬
‫‪Infectious diseases/sexually‬‬
‫)‪(412‬‬
‫‪transmitted diseases‬‬
‫‪Nephrology‬‬ ‫)‪(335‬‬
‫‪Neurology‬‬ ‫)‪(413‬‬
‫‪Ophthalmology‬‬ ‫)‪(81‬‬
‫‪Psychiatry‬‬ ‫)‪(268‬‬
‫‪Respiratory Medicine‬‬ ‫)‪(457‬‬
‫‪Rheumatology‬‬ ‫)‪(380‬‬
Clinical Sciences
Cell/molecular and membrane
(72)
biology
Clinical anatomy (67)
Clinical biochemistry and metabolism (118)
Clinical physiology (92)
Genetics (107)
Immunology (118)
Statistics (158)

Past-Papers index
Passing mark range between 63-66%
Year No. of Q
MRCP 1 May 19 100
MRCP 1 Jan 19 100
MRCP 1 Sept 18 100
MRCP 1 May 18 100
MRCP 1 Jan 18 100
MRCP 1 Sept 17 100
MRCP 1 May 17 100
MRCP 1 Jan 17 100
MRCP 1 Sept 16 100
MRCP 1 May 16 100
MRCP 1 Jan 2016 100
MRCP 1 Sept 2015 100
MRCP 1 May 2015 100
MRCP 1 Jan 2015 100

A. H. Murad
‫ﻻ ﺗﻨﺴﻮﻧﺎ ﻣﻦ ﺻﺎﻟﺢ دﻋﺎﺋﻜﻢ‬
‫ﺗﻢ ﺑﺤﻤﺪ ﷲ وﺗﻮﻓﯿﻘﮫ وﻣﻨﮫ‬
‫ﺟﻌﻞ ﷲ ﻋﻤﻠﻨﺎ ﻣﺘﻘﺒﻼ ﺧﺎﻟﺼﺎ ﻟﻮﺟﮭﮫ اﻟﻜﺮﯾﻢ‬
0:00:14/03:00:00

A 38-year-old heroin addict is treated with buprenorphine for his addiction. He reports in the
clinic that although he has tried to buy street heroin, this is giving him much less of a high
than it did before beginning buprenorphine.
Which of the following best represents the mode of action of buprenorphine?

A Full kappa opioid receptor agonist

B Full kappa opioid receptor antagonist

C Partial delta opioid receptor agonist

D Full mu opioid receptor antagonist

E Partial mu opioid receptor agonist

Explanation 

E Partial mu opioid receptor agonist

Buprenorphine is a partial opiate agonist at mu and kappa opioid receptors. It also has a long
half-life of up to 32hrs. This means that it can be utilised in cases of addiction to short-acting
opiates such as diamorphine because it reduces the highs, and thus addictive potential,
associated with these agents.

A Full kappa opioid receptor agonist

Full kappa opioid receptor agonist is incorrect. Buprenorphine is a partial agonist at kappa
receptors, meaning that by occupying the receptor, it doesn’t achieve the effects of full
agonism, and thus has less addictive potential versus other opiates.

B Full kappa opioid receptor antagonist

Full kappa opioid receptor antagonist is incorrect. Due to the fact that buprenorphine is a
partial agonist, at higher doses it displays “functional antagonism”, meaning that by
occupying the receptor it blunts the effects of other full opiate agonists.
C Partial delta opioid receptor agonist

Partial delta opioid receptor agonist is incorrect. Buprenorphine has no significant effects at
delta opioid receptors. The exact function of delta opioid receptors isn’t known, although
they may play a role via cross talk with mu receptors to potentiate analgesic effects of
opiates.

D Full mu opioid receptor antagonist

Full mu opioid receptor antagonist is incorrect. Buprenorphine is not a full antagonist at the
mu opioid receptor; it does have some limited abuse potential in patients who are opiate
naïve due to partial agonist effects.
51196

Rate this question:

Next Question

Previous Question Tag Question

Feedback End Review

Difficulty: Average

Peer Responses %

Q. Answered Flagged

Q1

Q2

Q3

Q4
Q. Answered Flagged

Q5

Q6

Q7

Q8 

 External Links

Pharmacodynamic properties
medicines.org.uk/emc/medicine/26614#PHARMACOLOGICAL_PROPS
(https://www.medicines.org.uk/emc/medicine/26614#PHARMACOLOGICAL_PROPS)
0:00:14/03:00:00

A 23-year-old man is referred to the Haematology Clinic for review. He presented to the GP
with tiredness and was noted to have a neutrophil abnormality on his blood film with bilobed
nuclei. He has no past medical history of note and takes no regular medication. His father has
a skeletal anomaly with a short right arm, but there is no other history of disease in the family.
Examination reveals no lymphadenopathy, and abdominal examination is entirely normal.
Investigations:

Hb 139 g/l

WCC 6.7 x 10 9/l

PLT 203 x 10 9/l

Na + 139 mmol/l

K+ 4.5 mmol/l

Bicarbonate 24 mmol/l

Creatinine 90 micromol/l

ALT 35 mmol/l

Alb 38 g/l

Which of the following is the most likely diagnosis?

A Acute myeloid leukaemia

B Chronic myeloid leukaemia

C Congenital hypoparathyroidism

D Congenital Pelger–Huet anomaly

E Myelodysplasia

Explanation 

D Congenital Pelger–Huet anomaly


Pelger–Huet anomaly is a laminopathy associated with mutations in the lamin B receptor. This
leads to bilobed nuclei in neutrophils and in homozygotes, can also be associated with
skeletal abnormalities which include shortened limbs. Like this patient, heterozygotes usually
suffer no symptoms and the neutrophil anomaly is picked up as an incidental finding.

A Acute myeloid leukaemia

Acute myeloid leukaemia is incorrect. AML is associated with anaemia, immature white cells
seen on the blood film, and many more symptoms than those seen here, including
progressive tiredness and splenomegaly.

B Chronic myeloid leukaemia

Chronic myeloid leukaemia is incorrect. CML is associated with anaemia, elevated numbers of
granulocytes at all stages of development including eosinophils and basophils, and platelet
numbers may be normal or increased. It is also associated with hepatosplenomegaly.

C Congenital hypoparathyroidism

Congenital hypoparathyroidism is incorrect. Congenital hypoparathyroidism is associated


with other abnormalities, including cleft lip and palate, cardiac abnormalities and abnormal T-
cell immune function.

E Myelodysplasia

Myelodysplasia is incorrect. Myelodysplasia does present with neutrophils displaying the


Pelger–Huet anomaly, but it is also associated with anaemia, red blood cell abnormalities,
including dimorphic red cells with Pappenheimer bodies, and platelets may be large or
hypogranular.
51132

Rate this question:

Next Question

Previous Question Tag Question

Feedback End Review

Difficulty: Average

Peer Responses %
Q. Answered Flagged

Q1

Q2

Q3

Q4

Q5

Q6

Q7

Q8

Q9

0:00:14/03:00:00

A 31-year-old woman presents with easy bruising, petechial haemorrhages on her upper and
lower limbs, and bleeding from her gums occasionally when she cleans her teeth. These
problems have increased over the past few weeks. On examination, you note a BP of
122/82mmHg, pulse is 74 bpm and regular. Abdomen is soft and non-tender and there are no
masses.
Investigations:

Hb 121 g/l

WCC 6.7 x 10 9/l

PLT 38 x 10 9/l

Na + 139 mmol/l

K+ 4.5 mmol/l

Cr 98 micromol/l

ALT 32 IU/l

Alk phos 85 IU/l

Which of the following is the most appropriate initial intervention?

A High-dose dexamethasone

B Platelet transfusion

C Prednisolone

D Rituximab

E Tranexamic acid

Explanation 

C Prednisolone
This patient is likely to have immune thrombocytopenia (ITP) given the clinical picture seen
here with bleeding and an isolated low platelet count, in the absence of other signs such as
hepatosplenomegaly. ITP is caused by antibodies which react with platelet antigens.
Prednisolone and intravenous immunoglobulins are initial interventions of choice.

A High-dose dexamethasone

High-dose dexamethasone is incorrect. High-dose dexamethasone is a second-line option for


patients who fail to respond to oral prednisolone or IV immunoglobulin.

B Platelet transfusion

Platelet transfusion is incorrect. Platelet transfusion is an intervention of last resort for


patients with persistent or uncontrolled bleeding who present with ITP that is unresponsive
to other interventions.

D Rituximab

Rituximab is incorrect. Rituximab is a second-line option for the treatment of ITP which is
unresponsive to other interventions.

E Tranexamic acid

Tranexamic acid is incorrect. Tranexamic acid is usually prescribed in patients with ITP who
suffer significant uncontrolled menstrual bleeding.
51133

Rate this question:

Next Question

Previous Question Tag Question

Feedback End Review

Difficulty: Average

Peer Responses %
Q. Answered Flagged

Q1

Q2

Q3

Q4

Q5

Q6

Q7

Q8

Q9

0:00:14/03:00:00

A 34-year-old man presents to the Emergency Department complaining of central pleuritic


chest pain. He has suffered from a flu-like illness with fevers and joint pains over the past 2–3
days. Apart from a history of asthma, for which he takes a low-dose fluticasone inhaler, he is
otherwise well. On examination, his BP is 123/82 mmHg, pulse is 75 bpm and regular. There
are no murmurs. His chest is clear. ECG reveals widespread concave ST elevation in the
anterior chest leads, and evidence of PR depression.
Which of the following is the most appropriate intervention?

A Clopidogrel, aspirin and fondaparinux

B Colchicine

C Naproxen

D Percutaneous coronary intervention (PCI)

E Prednisolone

Explanation 

C Naproxen

This patient has widespread concave ST elevation in the precordial leads, coupled with
evidence of PR depression. These ECG findings, coupled with the recent viral illness are
consistent with a diagnosis of pericarditis, for which non-steroidal anti-inflammatory drugs
(NSAIDs) are the initial intervention of choice.

A Clopidogrel, aspirin and fondaparinux

Clopidogrel, aspirin and fondaparinux is incorrect. This is the treatment for an acute Non-ST-
elevation myocardial infarction (NSTEMI), which doesn’t fit with the symptomatic picture
seen here.

B Colchicine
Colchicine is incorrect. Colchicine either alone or in combination with an NSAID is appropriate
for patients with recurrent pericarditis or for those who have pain and symptoms persisting
for longer than 14 days. Tolerability in some patients may be limited by diarrhoea related to
therapy.

D Percutaneous coronary intervention (PCI)

PCI is incorrect. Given that the picture is of pericarditis rather than acute ischaemia, PCI isn’t
indicated in the management of this patient.

E Prednisolone

Prednisolone is incorrect. Corticosteroids should not be used for initial treatment of


pericarditis and should only be considered for patients who fail to respond to NSAIDs or
colchicine, or where colchicine is unsuitable or not tolerated.
51159

Rate this question:

Next Question

Previous Question Tag Question

Feedback End Review

Difficulty: Average

Peer Responses %

Q. Answered Flagged

Q1

Q2
Q. Answered Flagged

Q3

Q4

Q5

Q6

Q7

Q8
0:00:14/03:00:00

A 45-year-old man presents to the Rheumatology Department complaining of dropping


things on a frequent basis and has noticed some muscle wasting becoming visible when he
looks at the back of his right hand. On examination, you note wasting of the dorsal interossei.
Which of the following represents the nerve supply of the dorsal interossei?

A C4/5

B C5/6

C C6/C7

D C7/C8

E C8/T1

Explanation 

E C8/T1

The dorsal interossei are supplied by the deep branch of the ulnar nerve, which innervates the
muscles of the hypothenar eminence at its origin. As it crosses the deep part of the hand, it
innervates all the interosseous muscles and the third and fourth lumbricals. It ends by
innervating the adductor pollicis and flexor pollicis brevis.

A C4/5

C4/5 is incorrect. C4/5 injury results in damage to the dorsal scapular nerve, and leads to
weakness of the rhomboid muscles and levator scapulae.

B C5/6

C5/6 is incorrect. C5/6 nerve root injury can result in damage to the long thoracic nerve,
phrenic nerve, suprascapular nerve and the lateral pectoral nerve.

C C6/C7
C6/C7 is incorrect. C6/C7 nerve root injury can affect sensory supply to the hand because of
interference with the superficial branch of the radial nerve.

D C7/C8

C7/C8 is incorrect. C7/C8 lesions can also lead to radial nerve injury, innervation of the dorsal
interossei is from the ulnar nerve. Both the ulnar and median nerves are formed from the
C8/T1 nerve roots.
51192

Rate this question:

Next Question

Previous Question Tag Question

Feedback End Review

Difficulty: Average

Peer Responses %

Q. Answered Flagged

Q1

Q2

Q3

Q4

Q5

Q6

Q7
Q. Answered Flagged

Q8

0:00:14/03:00:00

A 46-year-old woman who is treated with long-standing lithium therapy comes to the
Endocrine Clinic with polyuria and polydipsia. A water deprivation test with intra-nasal
administration of DDAVP is suggestive of nephrogenic diabetes insipidus.
Which of the following is most important in free water reabsorption in the kidney?

A Aquaporin-2

B Epithelial sodium channel

C SGLT-1

D SGLT-2

E Thiazide-sensitive sodium chloride co-transporter

Explanation 

A Aquaporin-2

The aquaporin-2 channel is found in the collecting duct, within the apical cell membranes of
principal cells, and in intracellular vesicles located throughout the cell. Expression of
aquaporin-2 is driven by vasopressin, increasing free water reabsorption to maintain
intravascular volume.

B Epithelial sodium channel

Epithelial sodium channel is incorrect. The epithelial sodium channel is selectively permeable
to sodium ions, and function is modulated by aldosterone. It is found in the distal renal
tubule. In contrast to aquaporin-2, it modulates fluid balance by affecting movement of
sodium rather than movement of free water.

C SGLT-1

SGLT-1 is incorrect. SGLT-1 governs absorption of glucose in the intestine, and is a target for
modulation in the treatment of type 2 diabetes.
D SGLT-2

SGLT-2 is incorrect. SGLT-2 governs reabsorption of glucose in the kidney. A number of SGLT-
2 inhibitors are licensed for the treatment of type 2 diabetes. They act by increasing glucose
excretion into the urine.

E Thiazide-sensitive sodium chloride co-transporter

Thiazide-sensitive sodium chloride co-transporter (NCCT) is incorrect. The NCCT is found in


the distal convoluted tubule, and modulates around 5% of sodium reabsorption. Mutations in
the NCCT are recognised to lead to Gitelman’s syndrome, characterised by salt wasting,
hypotension, hypokalaemic metabolic alkalosis, hypomagnesemia and hypocalciuria.
51197

Rate this question:

Next Question

Previous Question Tag Question

Feedback End Review

Difficulty: Average

Peer Responses %

Q. Answered Flagged

Q1

Q2

Q3

Q4
Q. Answered Flagged

Q5

Q6

Q7

Q8 
0:00:14/03:00:00

A 62-year-old woman comes to the Emergency Medicine follow-up clinic for review. She
presented with headaches some 5 weeks earlier and was found to be hypertensive with a BP
of 155/105 mmHg. Apparently, she was known to be hypertensive on previous measurements
checked over a 4-month period by her GP but had previously refused treatment. She takes no
medication and has no significant past medical history. She is a non-smoker. At the time of
attendance at the Emergency Department, she was started on amlodipine 5mg daily but she
stopped this due to ankle oedema. Her BP is still elevated in clinic, at 165/110 mmHg. Her BMI
is 23 kg/m 2. Routine bloods are unremarkable.

Which of the following is the most appropriate intervention for BP control?

A Atenolol 50 mg

B Bisoprolol 2.5 mg

C Bendroflumethiazide 2.5 mg

D Hydrochlorothiazide 12.5 mg

E Indapamide 2.5 mg

Explanation 

E Indapamide 2.5 mg

In this situation, where a calcium channel blocker isn’t tolerated, a thiazide-like diuretic such
as indapamide or chlorthalidone is the preferred option according to NICE guidance. These
are preferred over traditional diuretics because of evidence of dysglycaemia in large outcome
studies with thiazides.

A Atenolol 50 mg

Atenolol 50 mg is incorrect. Like thiazides, β blockers are not preferred as initial therapy for
blood pressure because of the effects on blood glucose control seen in large meta-analyses.

B Bisoprolol 2.5 mg
Bisoprolol 2.5 mg is incorrect. Bisoprolol is not recommended for initial treatment of
hypertension in this population, but is first-line option for control of paroxysmal atrial
fibrillation, and in combination therapy for the treatment of heart failure.

C Bendroflumethiazide 2.5 mg

Bendroflumethiazide is incorrect. This is a traditional thiazide diuretic; as well as potentially


leading to dysglycaemia it can cause symptomatic hyponatraemia and hypokalaemia, it’s
therefore not a preferred option for the initial treatment of hypertension.

D Hydrochlorothiazide 12.5 mg

Hydrochlorothiazide is incorrect. Like bendroflumethiazide, this drug has been implicated as


a cause of dysglycaemia. As such, it should be avoided here.
51160

Rate this question:

Next Question

Previous Question Tag Question

Feedback End Review

Difficulty: Average

Peer Responses %

Q. Answered Flagged

Q1

Q2

Q3
Q. Answered Flagged

Q4

Q5

Q6

Q7

Q8 

 External Links

Hypertension in adults: diagnosis andHypertension in adults: diagnosis and managementmanagement


nice.org.uk/guidance/cg127/resources/hypertension-in-adults-diagnosis-and-management-pdf-35109454941637
(https://www.nice.org.uk/guidance/cg127/resources/hypertension-in-adults-
diagnosis-and-management-pdf-35109454941637)
0:00:14/03:00:00

A 73-year-old woman is admitted after an overdose of tricyclic anti-depressants. She says


that she intended to kill herself and was only stopped because she was found by her cleaner.
There is a past medical history of asthma which is well controlled with low dose seretide and
she recently lost her husband due to lung cancer. She receives a civil service pension from a
previous job in the foreign office. She hadn’t left a note.
Which of the following is the greatest contributor to risk of suicide in this patient?

A Over 70 years age group

B Bereavement

C History of asthma

D Receipt of a pension

E Spontaneity of the episode

Explanation 

B Bereavement

In this patient, recent death of a spouse has by far the greatest impact on risk of successful
suicide, where grief and feelings of loneliness prompt a desire to be with the person who has
died.

A Over 70 years age group

Over 70 years age group is incorrect. Increasing age is a risk factor for successful suicide,
although this patient is a relatively fit 73-year old with little evidence of significant chronic
disease, meaning her age isn’t a significant contributor to risk here.

C History of asthma

History of asthma is incorrect. This patient’s asthma would only impact on her risk of suicide
if it was poorly controlled and significantly impacted on her activities of daily living (ADLS).
D Receipt of a pension

Receipt of a pension is incorrect. Low annual income increases the risk of suicide, whereas a
civil service pension is of course likely to enable her to live comfortably.

E Spontaneity of the episode

Spontaneity of the episode is incorrect. It is rather planned suicide, where the patient has
bought or hoarded materials for the attempt, has left a note, and timed the attempt for
people not to discover her, which are associated with future successful episodes.
51226

Rate this question:

Next Question

Previous Question Tag Question

Feedback End Review

Difficulty: Average

Peer Responses %

Q. Answered Flagged

Q1

Q2

Q3

Q4

Q5
Q. Answered Flagged

Q6

Q7

Q8 
0:00:14/03:00:00

A 34-year-old man comes to the clinic for review of his hypertension. He is currently
prescribed ramipril 10 mg daily, indapamide 2.5 mg daily and amlodipine 5 mg daily.
Unfortunately, his blood pressure is still markedly elevated at 155/105 mmHg. He has no past
medical history, is slim and plays 2 h of football every weekend.
Investigations:

Hb 130 g/l

WCC 8.2 x 10 9/l

PLT 191 x 10 9/l

Na + 144 mmol/l

K+ 3.1 mmol/l

Cr 95 micromol/l

Glucose 5.0 mmol/l

Which of the following is the most likely diagnosis?

A Acromegaly

B Essential hypertension

C Hyporeninaemic hyperaldosteronism

D Phaeochromocytoma

E Renal artery stenosis

Explanation 

C Hyporeninaemic hyperaldosteronism

The main pointer here is the presence of hypertension resistant to three separate anti-
hypertensives at therapeutic dose, including an ACE inhibitor, coupled with the presence of
low potassium. This is highly suggestive of hyporeninaemic hyperaldosteronism as the
underlying diagnosis.

A Acromegaly

Acromegaly is incorrect. There are no changes in facial appearance or soft tissue swelling
consistent with acromegaly. Dysglycaemia is also absent, with a completely normal random
blood glucose.

B Essential hypertension

Essential hypertension is incorrect. Essential hypertension is very unlikely here given that this
patient has failed to gain control of his blood pressure on three agents and has
hypokalaemia. In essential hypertension, potassium would be expected to be in the normal
range.

D Phaeochromocytoma

Phaeochromocytoma is incorrect. Phaeochromocytoma is often associated with paroxysms


of hypertension and tachycardia, which can lead to presentation with severe headaches. This
may occur in association with psychological stressors.

E Renal artery stenosis

Renal artery stenosis is incorrect. Unilateral renal artery stenosis may lead to hypertension
and hypokalaemia, although there is no indication of atherosclerotic vascular disease, and
fibromuscular dysplasia of the renal arteries is more commonly seen in women of Afro-
Caribbean descent.
51161

Rate this question:

Next Question

Previous Question Tag Question

Feedback End Review

Difficulty: Average

Peer Responses %
Q. Answered Flagged

Q1

Q2

Q3

Q4

Q5

Q6

Q7

Q8

Q9

0:00:14/03:00:00

A 42-year-old woman presents to the Emergency Department with gradually progressive


weakness which she says began in her feet and ankles and has now progressed to her hips
and thighs such that she is unable to walk. Around 3 weeks earlier, she suffered a self-limiting
gastrointestinal illness which was diagnosed after stool culture to be campylobacter. On
examination, her BP is 122/72 mmHg, pulse is 67 bpm and regular. Heart sounds are normal,
chest is clear, abdomen is soft and non-tender. There is a 3/5 weakness affecting both lower
limbs, more distal than proximal and reflexes are diminished.
Investigations:

Hb 121 g/l

WCC 8.8 x 10 9/l

PLT 203 x 10 9/l

Na + 140 mmol/l

K+ 4.5 mmol/l

Cr 90 mmol/l

Glucose 5.2 mmol/l

CSF protein 1.8 g/l

Which of the following is the most appropriate intervention?

A Intravenous (IV) immunoglobulin

B IV methylprednisolone

C IV rituximab

D Oral cyclosporine

E Pyridostigmine

Explanation 

A Intravenous (IV) immunoglobulin


The ascending weakness here coupled with recent episode of campylobacter infection and
elevated CSF protein fits well with a diagnosis of Guillain–Barré syndrome. Initial intervention
of choice is IV immunoglobulin ± plasma exchange. Monitoring of forced vital capacity (FVC)
is important as impending respiratory failure drives the need for intervention and supportive
management on the ITU.

B IV methylprednisolone

IV methylprednisolone is incorrect. There is no evidence that corticosteroids are of benefit in


the management of Guillain–Barré syndrome; indeed, outcomes with respect to neurological
recovery may be poorer in patients treated with steroids.

C IV rituximab

IV rituximab is incorrect. IV rituximab is usually employed in patients who have refractory


symptoms of myasthenia despite intervention with corticosteroids and conventional disease-
modifying agents. Rituximab-responsive Guillain–Barré syndrome has been seen in patients
who undergo allogenic bone marrow transplant.

D Oral cyclosporine

Oral cyclosporine is incorrect. Oral cyclosporine is a disease-modifying agent used in the


treatment of myasthenia gravis.

E Pyridostigmine

Pyridostigmine is incorrect. Pyridostigmine is a cholinesterase inhibitor which is used as a


symptomatic treatment for myasthenia gravis, but it is not disease modifying.
51181

Rate this question:

Next Question

Previous Question Tag Question

Feedback End Review

Difficulty: Average

Peer Responses %
Q. Answered Flagged

Q1

Q2

Q3

Q4

Q5

Q6

Q7

Q8

Q9

0:00:14/03:00:00

A 54-year-old woman presents to the clinic for review. She complains of feeling weak and
lethargic over the past few weeks and feels that she is unable to look upward for any length
of time. Her relatives say that her voice appears to have become weaker. She is unable to
hold down her job as a solicitor and has had considered taking early retirement. There is no
past medical history of note and she is a non-smoker. On examination, her BP is 118/82
mmHg, pulse is 72 bpm and regular. She is unable to look upward for more than 8 or 9
seconds and has a weak, nasal quality to her speech. Limb tone is normal bilaterally, as are
reflexes; there is no muscle wasting or fasciculation and sensation is intact.

Which of the following autoantibodies are most likely to be found?

A Anti-acetyl-choline receptor antibodies

B Anti-amphiphysin antibodies

C Anti-smooth muscle antibodies

D Anti-Yo antibodies

E Anti-Zic4 antibodies

Explanation 

A Anti-acetyl-choline receptor antibodies

This patient’s presentation is consistent with myasthenia gravis, with gradual onset of central
muscle weakness with normal tone and sensation. The absence of a history of smoking or
other signs and symptoms of malignancy counts against a paraneoplastic syndrome.

B Anti-amphiphysin antibodies

Anti-amphiphysin antibodies is incorrect. Anti-amphiphysin antibodies are associated with


stiff-person syndrome and encephalomyelitis, and are most commonly found in breast and
lung cancer.

C Anti-smooth muscle antibodies


Anti-smooth muscle antibodies is incorrect. Anti-smooth muscle antibodies are found in
patients with autoimmune hepatitis and in primary biliary cirrhosis, and are not associated
with myasthenia gravis.

D Anti-Yo antibodies

Anti-Yo antibodies is incorrect. Anti-Yo antibodies are associated with cerebellar


degeneration and are found in breast and gynaecological cancers.

E Anti-Zic4 antibodies

Anti-Zic4 antibodies is incorrect. Anti-Zic4 antibodies are associated with cerebellar


degeneration and are found in association with small cell lung cancer.
51182

Rate this question:

Next Question

Previous Question Tag Question

Feedback End Review

Difficulty: Average

Peer Responses %

Q. Answered Flagged

Q1

Q2

Q3
Q. Answered Flagged

Q4

Q5

Q6

Q7

Q8
0:00:14/03:00:00

A 23-year-old man comes to the Allergy Clinic for review. He complains of itching and
burning in his mouth when he eats apples and celery and he is now avoiding these foods in
his diet. He has a past history of severe spring hayfever, but is otherwise well and takes no
medication. Examination in the clinic is entirely unremarkable.
Which of the following is the most appropriate next step?

A Avoidance of these foods where possible

B Epipen, in case of a severe allergic reaction

C Mast cell tryptase assay

D RAST testing

E Regular anti-histamines

Explanation 

A Avoidance of these foods where possible

This patient has oral allergy syndrome, which is associated with a very low risk of progression
to full-blown anaphylaxis. It occurs in patients with birch allergy (who often suffer severe
hayfever in spring), where antibodies cross-react with proteins found in fruits such as apple
and vegetables, including celery. No intervention is usually needed, with patients usually
avoiding foods which cause symptoms.

B Epipen, in case of a severe allergic reaction

Epipen, in case of a severe allergic reaction is incorrect. There is a very low risk of
anaphylaxis in oral allergy syndrome, meaning that an epipen is not required.

C Mast cell tryptase assay

Mast cell tryptase assay is incorrect. Mast cell tryptase is elevated in anaphylaxis, hence is
most useful where patients present with full-blown symptoms of an anaphylactic reaction.
D RAST testing

RAST testing is incorrect. RAST testing is expensive, and no more useful potentially than
patch testing, where allergy to birch pollens can be easily confirmed.

E Regular anti-histamines

Regular anti-histamines is incorrect. Regular anti-histamines are not required, apart from the
period in spring when birch trees produce most of their pollen.
51168

Rate this question:

Next Question

Previous Question Tag Question

Feedback End Review

Difficulty: Average

Peer Responses %

Q. Answered Flagged

Q1

Q2

Q3

Q4

Q5

Q6
Q. Answered Flagged

Q7

Q8

0:00:14/03:00:00

A 72-year-old woman is reviewed in the Rheumatology Clinic some 6 months after


commencing treatment for polymyalgia rheumatica. She is unable to taper her steroid dose
below 10 mg of prednisolone without a return of her symptoms and her erythrocyte
sedimentation rate (ESR) remains stubbornly elevated at 32 mm/hr.
Which of the following is the most effective intervention for her disease?

A Bevacizumab

B Golimumab

C Infliximab

D Rituximab

E Toclizumab

Explanation 

E Toclizumab

Toclizumab is an anti-IL6 monoclonal antibody which has proven effectiveness as both an


initial therapy for giant cell arteritis, and as add-in to reduce corticosteroid dose. One trial
showed successful remission in >50% of those treated versus only approximately 20% in
those treated with conventional therapies.

A Bevacizumab

Bevacizumab is incorrect. Bevacizumab is an anti-VEGF antibody used in the treatment of


solid tumours and macular degeneration.

B Golimumab

Golimumab is incorrect. Golimumab is an anti-tumour necrosis factor (TNF) monoclonal


antibody used in the treatment of psoriasis and psoriatic arthritis. Evidence from studies of
etanercept in polymyalgia suggests only a modest effect on disease activity of these agents.
C Infliximab

Infliximab is incorrect. Infliximab is an anti-TNF monoclonal antibody used in the treatment of


a range of inflammatory diseases. TNF is thought to play only a minor role in the
pathogenesis of polymyalgia.

D Rituximab

Rituximab is incorrect. Case reports exist to support use of rituximab, (an anti-CD20
monoclonal antibody), in patients with steroid-resistant Polymyalgia rheumatica (PMR),
although large-scale randomised controlled trial evidence like that for anti-IL6 is lacking.
51217

Rate this question:

Next Question

Previous Question Tag Question

Feedback End Review

Difficulty: Average

Peer Responses %

Q. Answered Flagged

Q1

Q2

Q3

Q4

Q5
Q. Answered Flagged

Q6

Q7

Q8

 External Links

Trial of Tocilizumab in Giant-Cell Arteritis


nejm.org/doi/full/10.1056/NEJMoa1613849
(http://www.nejm.org/doi/full/10.1056/NEJMoa1613849 )
0:00:14/03:00:00

A 22-year-old man is referred to the Endocrine Clinic for review. He has a strong family
history of type 2 diabetes and over the past 3 months, has suffered from two episodes of
balanitis. In addition, he is suffering from increasing tiredness. He has no past medical history
of note and takes no regular medication. On examination, his BP is 110/70 mmHg, pulse is 73
bpm and regular. He is slim with a BMI of 22 kg/m 2.
Investigations:

Hb 131 g/l

WCC 6.0 x 10 9/l

PLT 203 x 10 9/l

Na + 140 mmol/l

K+ 3.6 mmol/l

Cr 90 micromol/l

Glucose (fasting) 7.5 mmol/l

HbA1c 51 mmol/mol

Anti-GAD and anti-IA2 antibody negative


Which of the following is the most appropriate initial intervention?

A Empagliflozin

B Gliclazide

C Insulin

D Metformin

E Sitagliptin

Explanation 

B Gliclazide
Given this patient’s history, family history and body habitus, and the negative autoantibody
screen, the most likely diagnosis is maturity-onset diabetes of the young (MODY). MODY3
accounts for between 50% and 65% of cases, and is caused by a mutation in the HNF-1 alpha
gene. This results in a reduced insulin secretory response to glucose, which can be managed
sometimes for many years by treatment with a sulphonylurea.

A Empagliflozin

Empagliflozin is incorrect. Empagliflozin is an SGLT-2 inhibitor; this means that it increases


renal excretion of glucose. The class of agents is most useful in patients with type 2 diabetes
who are overweight. In this instance, it won’t impact on the insulin secretory defect and is
unlikely to be effective.

C Insulin

Insulin is incorrect. Insulin will control blood glucose adequately, but isn’t required in the early
stages of MODY1. Hence, it’s not appropriate to commit patients to injectable therapy early in
their disease.

D Metformin

Metformin is incorrect. Insulin resistance isn’t the primary problem in MODY1, metformin is
therefore not effective in managing the condition.

E Sitagliptin

Sitagliptin is incorrect. The primary defect in MODY1 isn’t one of endogenous incretin
production. As such, there is a limited response to treatment with DPPIV inhibitors.
51142

Rate this question:

Next Question

Previous Question Tag Question

Feedback End Review

Difficulty: Average

Peer Responses %
Q. Answered Flagged

Q1

Q2

Q3

Q4

Q5

Q6

Q7

Q8

Q9

0:00:14/03:00:00

A 45-year-old man with a history of hypertension comes to the clinic for review. He currently
takes ramipril 10 mg daily, indapamide 2.5 mg, and amlodipine 5 mg. On examination, his BP
is 155/98 mmHg, pulse is 76 bpm and regular. His BMI is 23 kg/m 2. Heart sounds are normal,
chest is clear, he has mild ankle oedema, there are no arterial bruits.
Investigations:

Hb 135 g/l

WCC 6.8 x 10 9/l

PLT 215 x 10 9/l

Na + 144 mmol/l

K+ 3.7 mmol/l

Cr 98 micromol/l

Which of the following is the most appropriate next step for BP control?

A Bisoprolol

B Increase amlodipine to 10 mg

C Nebivolol 5 mg

D Spironolactone 25 mg

E Valsartan 80 mg

Explanation 

D Spironolactone 25 mg

In this situation, where the potassium is below 4.5 mmol/l, NICE guidance recommends
addition of low-dose spironolactone, although this should be used with caution where the
eGFR is significantly reduced. Specialist advice should also be obtained because of the risk of
significant pathophysiology underlying the resistant hypertension seen here, such as an
adrenal adenoma leading to hyper-reninaemic hyperaldosteronism.
A Bisoprolol

Bisoprolol is incorrect. Bisoprolol is usually considered if there is underlying cardiac failure or


atrial fibrillation, where it has a role in rate control.

B Increase amlodipine to 10 mg

Increase amlodipine to 10 mg is incorrect. Increasing amlodipine to 10 mg has limited extra


impact with respect to blood pressure control, at the expense of a significant worsening in
symptoms of ankle oedema.

C Nebivolol 5 mg

Nebivolol 5 mg is incorrect. Nebivolol is a ß blocker with additional vasodilating properties


related to nitric oxide potentiation. Its use tends to be restricted to very resistant
hypertension and to some patients with heart failure.

E Valsartan 80 mg

Valsartan 80mg is incorrect. Angiotensin II receptor blocker (ARB) used in combination with
ACE inhibitor is not recommended because of the risk of hyperkalaemia and little additional
effect on blood pressure control.
51162

Rate this question:

Next Question

Previous Question Tag Question

Feedback End Review

Difficulty: Average

Peer Responses %
Q. Answered Flagged

Q1

Q2

Q3

Q4

Q5

Q6

Q7

Q8

Q9

 External Links

Hypertension in adults: diagnosis andHypertension in adults: diagnosis and managementmanagement


nice.org.uk/guidance/cg127/resources/hypertension-in-adults-diagnosis-and-management-pdf-35109454941637
(../../QA/www.nice.org.uk/guidance/cg127/resources/hypertension-in-adults-
diagnosis-and-management-pdf-35109454941637)
0:00:14/03:00:00

A 70-year-old man is reviewed on the ward whilst recovering from an exacerbation of chronic
obstructive pulmonary disease (COPD). He smoked 40 cigarettes per day until 4 years ago,
when he gave up after an inferior myocardial infarction. Current medication includes a PRN
salbutamol inhaler. Forced expiratory volume in 1 second (FEV 1) was measured as 60% of
predicted, some 4 weeks earlier by his GP. He is comfortable at rest and there is minor
wheeze only on auscultation of the chest. He has previously not seen a significant
improvement in response to corticosteroid therapy. His BMI is 26 kg/m 2.
Which of the following is the most appropriate intervention?

A Carbocysteine

B High-dose fluticasone and salmeterol

C Montelukast

D Roflumilast

E Tiotropium

Explanation 

E Tiotropium

In this case, with an FEV 1 > 50% of predicted, and no features of asthma / response to
steroids, National Institute for Clinical Excellence guidelines for COPD recommend the
addition of a long-acting muscarinic antagonist (LAMA), such as tiotropium. Earlier data
suggesting that long-acting β agonists (LABAs) and muscarinic antagonists may be
associated with increased risk of stroke has proved unfounded.

A Carbocysteine

Carbocysteine is incorrect. Carbocysteine is a mucolytic agent which impacts on symptoms


of chronic bronchitis, but isn’t proven to impact on outcomes.

B High-dose fluticasone and salmeterol


High-dose fluticasone and salmeterol is incorrect. High-dose inhaled corticosteroids and
LABA therapy are recommended as an alternative to LAMA in patients who have an FEV 1 <
50% predicted where there are asthmatic features.

C Montelukast

Montelukast is incorrect. Montelukast is a leukotriene receptor antagonist used in the


treatment of asthma rather than in the treatment of COPD.

D Roflumilast

Roflumilast is incorrect. Roflumilast is only recommended for the treatment of COPD in


patients who have failed to gain control of their symptoms on triple therapy with LAMA,
LABA and high-dose corticosteroid. It is a phosphodiesterase-4 (PDE-4 inhibitor).
51213

Rate this question:

Next Question

Previous Question Tag Question

Feedback End Review

Difficulty: Average

Peer Responses %

Q. Answered Flagged

Q1

Q2

Q3
Q. Answered Flagged

Q4

Q5

Q6

Q7

Q8 

 External Links

Chronic obstructive pulmonary disease in over 16s: diagnosis and management


nice.org.uk/guidance/CG101/chapter/1-Guidance#managing-stable-copd
(https://www.nice.org.uk/guidance/CG101/chapter/1-Guidance#managing-stable-
copd)
0:00:14/03:00:00

A 72-year-old woman presents to the GP with low mood, early morning wakening, poor
appetite and social withdrawal some 3 months after the death of her husband. She says that
she hears his voice calling out for her and believes that he is waiting for her. She is not
distressed by the hearing of voices and doesn’t want any treatment. Physical examination is
entirely normal. She realises that her mood is low.
Which of the following is the most likely diagnosis?

A Early dementia

B Encephalitis

C Frontal meningioma

D Reactive depression

E Schizophrenia

Explanation 

D Reactive depression

The features of low mood, early morning wakening and poor appetite fit well with a diagnosis
of reactive depression related to the loss of her husband. Cognitive behavioural therapy is the
initial intervention of choice, with SSRI where there is an inadequate response.

A Early dementia

Early dementia is incorrect. Although early dementia may be unmasked by the death of a
partner, the specific features of depression count more towards this as the underlying
diagnosis.

B Encephalitis

Encephalitis is incorrect. Encephalitis is more likely to be associated with short-term memory


loss, drowsiness and confusion, rather than the specific features of early morning wakening
and poor appetite, as seen here.
C Frontal meningioma

Frontal meningioma is incorrect. Although frontal meningiomas may be associated with


social withdrawal, they are more likely to be associated with inappropriate actions, anger or
exaggerated emotion.

E Schizophrenia

Schizophrenia is incorrect. Although schizophrenia is associated with delusions, it is


associated with lack of insight, and other features including thought broadcasting and
primary delusional perception.
51227

Rate this question:

Next Question

Previous Question Tag Question

Feedback End Review

Difficulty: Average

Peer Responses %

Q. Answered Flagged

Q1

Q2

Q3

Q4

Q5
Q. Answered Flagged

Q6

Q7

Q8

0:00:14/03:00:00

A 39-year-old man presents to the Gastroenterology Clinic for review. Although symptoms of
his ulcerative colitis have been stable for the past 2–3 years, he has been feeling increasingly
tired over the past 6 months with itching and vague right upper quadrant pain and has lost a
few kilogrammes in weight. On examination, his BP is 118/82 mmHg, pulse is 67 bpm and
regular. Abdomen is soft and non-tender, although you notice a few scratch marks consistent
with itching. His BMI is 23 kg/m 2.
Investigations:

Hb 112 g/l

WCC 7.0 x 10 9/l

PLT 112 x 10 9/l

Na + 139 mmol/l

K+ 3.9 mmol/l

Cr 89 mmol/l

Alkphos 386 IU/l

ALT 112 IU/l

Bilirubin 12 mmol/l

Which of the following is the most useful investigation for this patient?

A Computed tomography (CT) abdomen

B Endoscopic retrograde cholangiopancreatography (ERCP)

C Liver biopsy

D Magnetic resonance cholangiopancreatography (MRCP)

E Percutaneous transhepatic cholangiography (PTC)

Explanation 
D Magnetic resonance cholangiopancreatography (MRCP)

Magnetic resonance cholangiopancreatography (MRCP) is the preferred initial investigation


for primary sclerosing cholangitis (PSC), which is the most likely diagnosis here. MRCP
findings include multiple strictures and dilatations of both intrahepatic and extrahepatic bile
ducts.

A Computed tomography (CT) abdomen

Computed tomography (CT) abdomen is incorrect. The resolution of CT is such that it is


relatively insensitive with respect to identifying the structuring and dilatation seen in PSC. CT
scanning does however show abnormal liver contours and multiple low-density regions which
represent dilated bile ducts.

B Endoscopic retrograde cholangiopancreatography (ERCP)

Endoscopic retrograde cholangiopancreatography (ERCP) is incorrect. ERCP is not preferred


because it is invasive and is usually considered when intervention via the endoscope, for
example, stent placement, is required.

C Liver biopsy

Liver biopsy is incorrect. Liver biopsy shows periductal concentric fibrosis around interlobular
and septal bile ducts. It is not preferred versus MRCP, as the diagnosis can often be made
from the scan alone.

E Percutaneous transhepatic cholangiography (PTC)

Percutaneous transhepatic cholangiography (PTC) is incorrect. PTC or ERCP are useful when
a cholangiocarcinoma is suspected, where brush cytology can be obtained to confirm the
presence of a neoplasm.
51198

Rate this question:

Next Question

Previous Question Tag Question

Feedback End Review

Difficulty: Average
Peer Responses %

Q. Answered Flagged

Q1

Q2

Q3

Q4

Q5

Q6

Q7

Q8

Q9

0:00:14/03:00:00

A 54-year-old man presents with a gastric ulcer on the greater curvature of his stomach,
which has resulted in a significant gastrointestinal bleed.
Which of the following is the artery which supplies most blood to the greater curvature of
the stomach?

A Gastroduodenal artery

B Left gastroepiploic artery

C Left gastric artery

D Right gastric artery

E Short gastric artery

Explanation 

B Left gastroepiploic artery

The left gastroepiploic artery is the largest branch of the splenic artery and supplies several
ascending branches to the greater curvature of the stomach. It anastomoses with the right
gastroepiploic artery.

A Gastroduodenal artery

Gastroduodenal artery is incorrect. It supplies blood directly to the pylorus and proximal part
of the duodenum, and indirectly to the pancreatic head (via the anterior and posterior
superior pancreaticoduodenal arteries).

C Left gastric artery

Left gastric artery is incorrect. The left gastric artery arises from the coeliac artery and runs
along the superior portion of the lesser curvature of the stomach. Erosion of the mucosa of
the lesser curve into the artery by an ulcer is known to result in massive blood loss.

D Right gastric artery


Right gastric artery is incorrect. The right gastric artery arises from the hepatic artery proper,
supplying blood to the lesser curvature, and anastomosing with the left gastric artery.

E Short gastric artery

Short gastric artery is incorrect. The short gastric arteries consist of five to seven branches
which arise from the terminal divisions of the splenic artery. They supply blood to a short
segment of the greater curvature of the stomach.
51193

Rate this question:

Next Question

Previous Question Tag Question

Feedback End Review

Difficulty: Average

Peer Responses %

Q. Answered Flagged

Q1

Q2

Q3

Q4

Q5

Q6
Q. Answered Flagged

Q7

Q8
0:00:14/03:00:00

A 38-year-old woman who has suffered a previous deep vein thrombosis (DVT) some 3 years
earlier, wakes from sleep with painless, blurred vision affecting her left eye. Previously, her
vision has been normal bilaterally, and she is now finding it very difficult to read the Snellen
chart using her left eye. Fundoscopy reveals an arc of haemorrhages spreading out from the
central region of the visual field and dilated, tortuous veins within the arc. There is no
evidence of macular oedema. Screening for thrombophilia reveals that she is Factor V Leiden
positive.
Which of the following is the most appropriate long-term intervention?

A Aflibercept

B Aspirin

C Intra-ocular triamcinolone

D Pan-retinal photocoagulation

E Warfarin

Explanation 

E Warfarin

This patient has a second venous thrombosis against a background of Factor V Leiden
mutation. As such, long-term anti-coagulation is indicated.

A Aflibercept

Aflibercept is incorrect. Aflibercept is an anti-vascular endothelial growth factor (VEGF)


recombinant fusion protein licensed for the treatment of macular degeneration. Where there
is macular oedema associated with retinal vein occlusion, treatment with an anti-VEGF agent
may be indicated.

B Aspirin
Aspirin is incorrect. Anti-platelet agents such as aspirin are not effective for preventing
recurrent thrombosis in patients with the Factor V Leiden mutation.

C Intra-ocular triamcinolone

Intra-ocular triamcinolone is incorrect. Triamcinolone has short-lived effects in relieving


macular oedema, although over the longer term, it has little or no positive effects on
neovascularisation versus other options such as pan-retinal photocoagulation or anti-VEGF.

D Pan-retinal photocoagulation

Pan-retinal photocoagulation is incorrect. Pan-retinal photocoagulation is an alternative to


anti-VEGF therapy for the treatment of macular oedema associated with branch retinal vein
occlusion.
51189

Rate this question:

Next Question

Previous Question Tag Question

Feedback End Review

Difficulty: Average

Peer Responses %

Q. Answered Flagged

Q1

Q2

Q3
Q. Answered Flagged

Q4

Q5

Q6

Q7

Q8 
0:00:14/03:00:00

A 67-year-old woman comes to the Emergency Department having suffered a fall in which
she sustained a left Colles’ fracture. Medication of note includes denosumab for the
treatment of osteoporosis. Her calcium is in the normal range at 2.51 mmol/l (corrected) and
alkaline phosphatase is 92 IU/l.
Which of the following represents the mode of action of denosumab?

A Bisphosphonate

B Calcitonin receptor agonist

C PTH analogue

D Receptor activator of nuclear factor kappa-B (RANK) ligand inhibitor

E Vitamin D analogue

Explanation 

D Receptor activator of nuclear factor kappa-B (RANK) ligand inhibitor

RANK is a surface receptor on the surface of osteoclast precursors and osteoclasts. When
RANK ligand binds to it, this promotes differentiation and increased osteoclast activity.
Denosumab is a monoclonal antibody which binds to RANK ligand, preventing its action. It
reduces bone resorption by up to 85% within 3 days of administration and has a duration of
action of up to 6 months.

A Bisphosphonate

Bisphosphonate is incorrect. Alendronate and etidronate are examples of bisphosphonates:


they are large molecules which become incorporated into bone and effectively poison
osteoclasts, reducing their activity.

B Calcitonin receptor agonist


Calcitonin receptor agonist is incorrect. Calcitonin and calcitonin-like agents are not widely
prescribed for chronic use in osteoporosis because of the risk of osteosarcoma associated
with long-term calcitonin use.

C PTH analogue

PTH analogue is incorrect. Teriparatide is a PTH analogue, which is given as a once per day
subcutaneous injection. It is thought that rapid absorption, coupled with an elevated C max
drive osteoblast rather than osteoclast activity in response to teriparatide.

E Vitamin D analogue

Vitamin D analogue is incorrect. The vitamin D analogue 1-hydroxycholecalciferol is most


commonly given in conjunction with calcium therapy to patients with osteoporosis.
51150

Rate this question:

Next Question

Previous Question Tag Question

Feedback End Review

Difficulty: Average

Peer Responses %

Q. Answered Flagged

Q1

Q2

Q3
Q. Answered Flagged

Q4

Q5

Q6

Q7

Q8 

 External Links

Pharmacodynamic properties
medicines.org.uk/emc/medicine/23127#PHARMACOLOGICAL_PROPS
(https://www.medicines.org.uk/emc/medicine/23127#PHARMACOLOGICAL_PROPS)
0:00:14/03:00:00

A 52-year-old man presents to the diabetes clinic for review. He was diagnosed with type 2
diabetes some 5 years earlier and over the past 2 years has suffered from painful peripheral
neuropathy, affecting both feet. His main complaint today is of erectile dysfunction, where he
fails to maintain a sufficient erection to engage in sexual intercourse with his wife. He has no
history of cardiovascular disease. Blood pressure is 132/82 mmHg, pulse is 67 bpm and
regular.
You elect to begin sildenafil.
Which of the following represents the mode of action of sildenafil?

A Calcium channel antagonist

B PDE4 inhibitor

C PDE5 inhibitor

D PDE6 inhibitor

E Prostaglandin E1 analogue

Explanation 

C PDE5 inhibitor

Sildenafil is a member of the phosphodiesterase 5 inhibitor class, selective for the PDE5 in the
corpus cavernosum. This reduces breakdown of cyclic GMP, and improves the quality of
erections.

A Calcium channel antagonist

Calcium channel antagonist is incorrect. Oral calcium channel antagonists don’t worsen
erectile function when used for the treatment of hypertension, although they don’t
significantly improve erectile dysfunction.

B PDE4 inhibitor
PDE4 inhibitor is incorrect. PDE4 inhibitors block the degradation of cyclic AMP, and
modulate immune function. An example of a PDE4 inhibitor used in the treatment of COPD is
roflumilast.

D PDE6 inhibitor

PDE6 inhibitor is incorrect. Sildenafil does have some action in inhibiting PDE6, responsible
for photoreceptor function in the eye. It is 10-fold less selective, however, for PDE6 versus
PDE5. At high doses of sildenafil, patients may report blue discolouration of their vision.

E Prostaglandin E1 analogue

Prostaglandin E1 analogue is incorrect. “Muse” is an analogue of prostaglandin E1, and is


administered using an intra-urethral applicator for the treatment of erectile dysfunction.
51151

Rate this question:

Next Question

Previous Question Tag Question

Feedback End Review

Difficulty: Average

Peer Responses %

Q. Answered Flagged

Q1

Q2

Q3
Q. Answered Flagged

Q4

Q5

Q6

Q7

Q8 

 External Links

Pharmacodynamic properties
medicines.org.uk/emc/medicine/1474#PHARMACOLOGICAL_PROPS
(https://www.medicines.org.uk/emc/medicine/1474#PHARMACOLOGICAL_PROPS)
0:00:14/03:00:00

A 20-year-old man comes to the Endocrinology Clinic for review. He presented to his GP
complaining of chronic tiredness and nocturia and was diagnosed with diabetes mellitus after
a fasting plasma glucose was noted to be 8.3 mmol/l. He has initially been commenced on
gliclazide by his doctor, although it is proving quite ineffective. On examination, his BP is
128/82 mmHg, pulse is 70 bpm and regular. Abdomen is soft and non-tender, BMI is 27
kg/m 2.
Investigations:

Hb 130 g/l

WCC 6.3 x 10 9/l

PLT 231 x 10 9/l

Na + 139 mmol/l

K+ 4.5 mmol/l

Cr 89 micromol/l

Glucose (random) 8.8 mmol/l

CT abdomen pancreatic atrophy, bilateral renal cysts

Which of the following is the most likely defect causing his diabetes mellitus?

A Glucokinase mutation

B HNF-1 alpha mutation

C HNF-1 beta mutation

D IPF-1 mutation

E NEUROD-1 mutation

Explanation 

C HNF-1 beta mutation


HNF-1 beta drives tissue-specific gene expression. The hallmarks of the HNF-1 beta form of
maturity-onset diabetes of the young (MODY) are pancreatic atrophy and formation of
multiple renal cysts. They also have abnormal liver function and other genetic abnormalities,
including epididymal cysts in men and bicornate uterus in women.

A Glucokinase mutation

Glucokinase mutation is incorrect. The glucokinase mutation form of MODY is not associated
with abnormalities in organogenesis, and is in fact a defect of glucose sensing. Patients with
glucokinase mutations are usually controlled with diet alone and have a low risk of
complications of diabetes.

B HNF-1 alpha mutation

HNF-1 alpha mutation is incorrect. HNF-1 alpha mutations lead to abnormal insulin secretion
rather than abnormal organogenesis and can be managed with sulphonylureas, at least in the
early stages of the disease.

D IPF-1 mutation

IPF-1 mutation is incorrect. IPF-1 mutations reduce binding of the protein to the insulin gene
promoter. Less severe mutations in IPF-1 predispose to late-onset type 2 diabetes. They are
much rarer than HNF-1 alpha mutations as a cause of MODY.

E NEUROD-1 mutation

NEUROD-1 mutation is incorrect. NEUROD-1 is a switch for pancreatic endocrine


development and is a rare cause of MODY.
51143

Rate this question:

Next Question

Previous Question Tag Question

Feedback End Review

Difficulty: Average

Peer Responses %
Q. Answered Flagged

Q1

Q2

Q3

Q4

Q5

Q6

Q7

Q8

Q9

0:00:14/03:00:00

A 49-year-old man who has been helping to build an extension on his house comes to the
Rheumatology Clinic for review. He has pain over the lateral aspect of his elbow and
examination reveals pain and tenderness over the lateral epicondyle of the humerus, radiating
into the forearm. There is also pain on resisted dorsiflexion of the wrist. Movement of the
elbow is normal.
Tendonitis affecting which muscle has caused his pain?

A Biceps

B Extensor carpi radialis brevis

C Flexor carpi radialis

D Flexor carpi ulnaris

E Flexor digitorum superficialis

Explanation 

B Extensor carpi radialis brevis

Tennis elbow is related to tendonitis affecting the common extensor tendon, with most pain
and restriction of movement thought to be related to inflammation of the extensor carpi
radialis brevis tendon. Pain relief and physiotherapy form the mainstay of intervention, with
tendon release possible for patients with resistant symptoms.

A Biceps

Biceps is incorrect. The biceps tendon attaches to the radial tuberosity, not to the lateral
epicondyle. A biceps tendon tear results in pain and swelling in the upper arm, and weakness
of supination.

C Flexor carpi radialis

Flexor carpi radialis is incorrect. The flexor muscles of the forearm, including flexor carpi
radialis, attach to the medial epicondyle.
D Flexor carpi ulnaris

Flexor carpi ulnaris is incorrect. Like flexor carpi radialis, flexor carpi ulnaris attaches to the
medial epicondyle and inflammation of the tendon may contribute to golfer’s elbow. Like
tennis elbow, golfer’s elbow is managed with non-steroidal anti-inflammatory drugs and
physiotherapy, in the first instance.

E Flexor digitorum superficialis

Flexor digitorum superficialis is incorrect. Like the other flexor muscles, flexor digitorum
superficialis attaches to the medial epicondyle.
51218

Rate this question:

Next Question

Previous Question Tag Question

Feedback End Review

Difficulty: Average

Peer Responses %

Q. Answered Flagged

Q1

Q2

Q3

Q4

Q5
Q. Answered Flagged

Q6

Q7

Q8

0:00:14/03:00:00

A 24-year-old woman presents to the Emergency Department with a rash consisting of


multiple papules which itch and burn, forming small blisters, particularly affecting her scalp,
buttocks, elbows and knees. The rash keeps her awake at night and she tells you that she
feels tired all the time. There is no past medical history of note, and she takes no regular
medication. On examination, her BP is 115/72 mmHg, pulse is 78 bpm and regular. You
confirm the rash with multiple small papules and blisters. She is slim and pale with a body
mass index of 21 kg/m 2.
Investigations:

Hb 98 g/l

WCC 7.8 x 10 9/l

PLT 171 x 10 9/l

Na + 138 mmol/l

K+ 4.5 mmol/l

Cr 87 micromol/l

Ferritin 18 pmol/l

Which of the following is the most likely cause of her rash?

A Dermatitis herpetiformis

B Erythema multiforme

C Erythema nodosum

D Scabies

E Systemic lupus erythematosus

Explanation 

A Dermatitis herpetiformis
This intensely itchy rash, particularly affecting the buttocks, elbows and knees, is consistent
with dermatitis herpetiformis. The iron deficiency anaemia seen here raises the possibility of
coeliac disease, and anti-TTG antibody testing is an obvious next step. Exclusion of gluten is
the intervention of choice, coupled with dapsone in patients who don’t gain resolution of
their skin rash.

B Erythema multiforme

Erythema multiforme is incorrect. Erythema multiforme is associated with target lesions, and
it is seen in conjunction with infections including herpes simplex virus and Mycoplasma
pneumoniae.

C Erythema nodosum

Erythema nodosum is incorrect. Erythema nodosum is associated with prominent raised


red/purple discoloured lesions on both shins and occurs in patients with sarcoidosis, and with
a history of tuberculosis.

D Scabies

Scabies is incorrect. Although scabies is an intensely itchy rash associated with formation of
small papules and vesicles, these are particularly seen affecting the finger web spaces.

E Systemic lupus erythematosus

Systemic lupus erythematosus (SLE) is incorrect. Other features of connective tissue disease
such as joint pains and night sweats are absent here. The rash associated with SLE is also
usually more prominent over sun-exposed areas.
51138

Rate this question:

Next Question

Previous Question Tag Question

Feedback End Review

Difficulty: Average

Peer Responses %
Q. Answered Flagged

Q1

Q2

Q3

Q4

Q5

Q6

Q7

Q8

Q9

0:00:14/03:00:00

A 72-year-old woman with a history of myeloma comes to the Haematology Clinic for review.
She has been prescribed a combination of thalidomide, melphalan and prednisolone, but is
unable to tolerate the thalidomide.
Which of the following is the most appropriate alternative to thalidomide?

A Bendamustine

B Bortezomib

C Cyclophosphamide

D Dexamethasone

E Lenalidomide

Explanation 

B Bortezomib

Bortezomib is a proteasome inhibitor. It inhibits a range of proteins which govern cell cycle
progression and NF-kappa B activation, leading to cell cycle arrest and apoptosis. In patients
who are unsuitable for aggressive chemotherapy and stem cell transplant, it is recommended
by NICE as an alternative to thalidomide.

A Bendamustine

Bendamustine is incorrect. Bendamustine is the preferred option in combination with


prednisolone, where patients have clinical neuropathy at the time of diagnosis which
precludes use of either thalidomide or bortezomib.

C Cyclophosphamide

Cyclophosphamide is incorrect. Cyclophosphamide is an alternative to melphalan in the


elderly non-transplant setting, but is not a replacement for thalidomide.

D Dexamethasone
Dexamethasone is incorrect. Bortezomib and dexamethasone in combination form the
backbone of induction therapy for younger patients with a good functional status (generally
younger than 65 years).

E Lenalidomide

Lenalidomide is incorrect. Lenalidomide is a thalidomide derivative, hence patients who are


unsuitable for thalidomide are usually also unsuitable for lenalidomide.
51134

Rate this question:

Next Question

Previous Question Tag Question

Feedback End Review

Difficulty: Average

Peer Responses %

Q. Answered Flagged

Q1

Q2

Q3

Q4

Q5

Q6
Q. Answered Flagged

Q7

Q8

 External Links

Pharmacodynamic properties
medicines.org.uk/emc/medicine/17109#PHARMACODYNAMIC_PROPS
(https://www.medicines.org.uk/emc/medicine/17109#PHARMACODYNAMIC_PROPS)
0:00:14/03:00:00

A 64-year-old man is admitted to the Emergency Department with central crushing chest
pain radiating down his left arm. He is diagnosed with an anterior Non-ST-elevation
myocardial infarction (NSTEMI) when ST depression is noted in the anterior chest leads, and
troponin is elevated. He is taking ezetimibe for control of his cholesterol.
Which of the following represents the mode of action of ezetimibe?

A HMG CoA reductase inhibitor

B NPC1L1 inhibitor

C PCSK9 inhibitor

D PPAR-α agonist

E PPAR-γ agonist

Explanation 

B NPC1L1 inhibitor

The molecular target of ezetimibe is the sterol transporter, Niemann-Pick C1-Like 1 (NPC1L1),
which is responsible for the intestinal uptake of cholesterol and phytosterols. Use of
ezetimibe therefore reduces cholesterol absorption from the gut.

A HMG CoA reductase inhibitor

HMG CoA reductase inhibitor is incorrect. Statins are HMG CoA reductase inhibitors, which is
thought to drive both reduction of VLDL-cholesterol (VLDL-C) and induction of the LDL
receptor, leading to reduced production and increased catabolism of LDL-cholesterol.

C PCSK9 inhibitor

PCSK9 inhibitor is incorrect. Evolocumab is a PCSK9 inhibitor, which prevents degradation of


liver LDL receptors, driving a reduction in serum LDL cholesterol.

D PPAR-α agonist
PPAR-α agonist is incorrect. Fibrates are PPAR-α agonists which drive increased metabolism
of triglycerides and an increase in HDL cholesterol.

E PPAR-γ agonist

PPAR-γ agonist is incorrect. Thiazolidinediones are PPAR-γ agonists which drive increased
conversion of glucose into peripheral fat stores. They don’t have a role in the treatment of
dyslipidaemia, but pioglitazone is licensed as a treatment for type 2 diabetes.
51163

Rate this question:

Next Question

Previous Question Tag Question

Feedback End Review

Difficulty: Average

Peer Responses %

Q. Answered Flagged

Q1

Q2

Q3

Q4

Q5

Q6
Q. Answered Flagged

Q7

Q8

 External Links

Pharmacodynamic properties
medicines.org.uk/emc/medicine/12091#PHARMACOLOGICAL_PROPS
(../../QA/www.medicines.org.uk/emc/medicine/12091#PHARMACOLOGICAL_PROPS)
0:00:14/03:00:00

A 27-year-old man who suffers from hypophosphataemic rickets comes to the Genetics Clinic
with his new partner. His symptoms are controlled with vitamin D and phosphate
supplementation. They ask for advice about starting a family.
Which of the following is the risk of a male child inheriting his condition?

A 100%

B 75%

C 50%

D 25%

E 0%

Explanation 

E 0%

Hypophosphataemic rickets, is an X-linked dominant condition which occurs due to a


mutation in the PHEX gene. This regulates activity of FGF-23, which regulates hydroxylation
of vitamin D and renal phosphate reabsorption. In X-linked-dominant conditions, an affected
father cannot pass on the condition to their male children because they only inherit the
father’s Y chromosome.

A 100%

100% is incorrect. It would be a chance of 100% if a female child of an affected father is


inheriting the condition, as the X chromosome they inherit from the father is always
abnormal.

B 75%

75% is incorrect. It would be a chance of 75% for the offspring if two carriers of an autosomal
recessive condition are either being affected by, or carriers for, the disorder.
C 50%

50% is incorrect. The risk is 50% for the child of an affected female being affected by the
condition because they may pass on their normal or abnormal X chromosome to the
offspring.

D 25%

25% is incorrect. The risk of 25% inheritance doesn’t fit with an X-linked-dominant condition.
It fits with the inheritance risk for an autosomal recessive condition, where both parents are
carriers of the abnormal gene.
51167

Rate this question:

Next Question

Previous Question Tag Question

Feedback End Review

Difficulty: Average

Peer Responses %

Q. Answered Flagged

Q1

Q2

Q3

Q4

Q5
Q. Answered Flagged

Q6

Q7

Q8

0:00:14/03:00:00

You are examining the results of a potential new bedside screening test for meningococcal
meningitis. You are interested in the negative predictive value, which may help dictate the
need for early intervention.
Results of the test are shown below:

Have meningitis No meningitis

Test positive 18 3

Test negative 2 98

Which of the following is the negative predictive value?

A 20%

B 86%

C 90%

D 97%

E 98%

Explanation 

E 98%

The negative predictive value is the ability of the test to predict who truly doesn’t have the
disease. In this case, it is the true negatives/(true negatives + false negatives) = 98/100.

A 20%

20% is incorrect. The prevalence of meningitis is 20% – those presenting with symptoms to
the hospital, (20/101).

B 86%
86% is incorrect. The positive predictive value is 86%, (18/21).

C 90%

90% is incorrect. The sensitivity is 90% (18/20).

D 97%

97% is incorrect. The specificity is 97% (98/101).


51170

Rate this question:

Next Question

Previous Question Tag Question

Feedback End Review

Difficulty: Average

Peer Responses %

Q. Answered Flagged

Q1

Q2

Q3

Q4

Q5

Q6
Q. Answered Flagged

Q7

Q8 
0:00:14/03:00:00

A 49-year-old painter who specialises in renovating old houses comes to the clinic for review.
He has suffered from increasing tiredness over the past 3–4 months and now has bony aches
and pains which are making it very difficult for him to do his job. There is no past medical
history of note; he is a non-smoker and drinks ten units of alcohol per week. On examination,
his BP is 122/82 mmHg, pulse is 80 bpm and regular. He looks pale. Chest is clear, abdomen is
soft and non-tender, with no masses. His BMI is 22 kg/m 2.
Investigations:

Hb 90g/l

WCC 6.7 x 10 9/l

PLT 181 x 10 9/l

Na + 138 mmol/l

K+ 3.4 mmol/l

Bicarbonate 15 mmol/l

Cr 95 micromol/l

Ca 2.05 mmol/l

PO 4 0.7 mmol/l

Which of the following is the most likely diagnosis?

A Distal renal tubular acidosis

B Hypoparathyroidism

C Poor dietary intake of vitamin D

D Proximal renal tubular acidosis

E Renal tubular acidosis type IV

Explanation 
D Proximal renal tubular acidosis

The clue here is this man’s occupation, which raises the possibility of lead poisoning because
of exposure in his role as a renovator of old houses. Lead poisoning results in proximal
tubular damage, which leads to acidosis and hypophosphataemia, which can then lead to
symptoms consistent with osteomalacia.

A Distal renal tubular acidosis

Distal renal tubular acidosis is incorrect. Distal renal tubular acidosis is associated with
hypokalaemia and metabolic acidosis, but isn’t associated with lead poisoning. Distal renal
tubular acidosis is associated with an elevated risk of nephrolithiasis.

B Hypoparathyroidism

Hypoparathyroidism is incorrect. Hypoparathyroidism is associated with low levels of


calcium, but it is associated with elevated phosphate rather than the low levels seen here.

C Poor dietary intake of vitamin D

Poor dietary intake of vitamin D is incorrect. Given that this man was previously healthy,
holds down a job, and does not admit to excess alcohol intake, poor dietary intake of vitamin
D is very unlikely.

E Renal tubular acidosis type IV

Renal tubular acidosis type IV is incorrect. Renal tubular acidosis type IV is associated with
defective aldosterone action, and is manifested by metabolic acidosis and hyperkalaemia
rather than the picture seen here.
51194

Rate this question:

Next Question

Previous Question Tag Question

Feedback End Review

Difficulty: Average

Peer Responses %
Q. Answered Flagged

Q1

Q2

Q3

Q4

Q5

Q6

Q7

Q8

Q9

0:00:14/03:00:00

A 22-year-old woman is reviewed in the Emergency Department having taken an overdose of


paracetamol tablets. She has a history of eating disorder and according to her relatives, is
starving herself and exercising for up to 4 hours per day. Her BMI is 17 kg/m 2.
Which of the following is the most appropriate intervention?

A Amitriptyline

B Immersion therapy

C Individual eating-disorder-focused cognitive behavioural therapy (CBT-ED)

D Oestrogen supplementation

E Sertraline

Explanation 

C Individual eating-disorder-focused cognitive behavioural therapy (CBT-ED)

Individual eating-disorder-focused cognitive behavioural therapy (CBT-ED) is a specific form


of CBT designed to address the psychopathology of eating disorders such as anorexia
nervosa, the likely diagnosis here.

A Amitriptyline

Amitriptyline is incorrect. CBT is the cornerstone of intervention for anorexia nervosa, if CBT-
ED is not effective, alternatives include the Maudsley anorexia nervosa treatment for adults
(MANTRA) programme, and specialist support clinical management. Given that amitriptyline
is pro-arrhythmogenic, it should potentially be avoided here, and it’s not a conventional
therapy for eating disorders.

B Immersion therapy

Immersion therapy is incorrect. Immersion therapy is a potential intervention for patients


suffering from phobias, where they may be challenged by exposure to their object of fear.
D Oestrogen supplementation

Oestrogen supplementation is incorrect. Routine oestrogen supplementation is not


recommended by the National Institute for Clinical Excellence for the treatment of patients
with anorexia nervosa; strategies to drive weight regain are preferred.

E Sertraline

Sertraline is incorrect. Although sertraline may have some impact on depressive symptoms
associated with anorexia nervosa, CBT is the preferred option for treating the underlying
eating disorder.
51228

Rate this question:

Next Question

Previous Question Tag Question

Feedback End Review

Difficulty: Average

Peer Responses %

Q. Answered Flagged

Q1

Q2

Q3

Q4

Q5
Q. Answered Flagged

Q6

Q7

Q8

0:00:14/03:00:00

A 52-year-old man who has lived for much of his life in Australia comes to the Dermatology
Clinic with a malignant melanoma on his back. He is prepared for surgery.
Which of the following has most impact on his prognosis?

A Age 52 years at diagnosis

B Presence of nodularity

C Presence of ulceration

D Site of the lesion

E Thickness of the melanoma

Explanation 

E Thickness of the melanoma

Survival in melanoma is most strongly correlated with the Breslow thickness of the melanoma
at the point of diagnosis. Current staging criteria divide thickness into <1 mm, 1–2 mm, 2–4
mm, and >4 mm lesions which carry the worst outcome.

A Age 52 years at diagnosis

Age 52 years at diagnosis is incorrect. Surgery for melanoma, even with wide local excision is
not as significant as that for other forms of cancer and increasing age may actually be
associated with slower melanoma growth. As such, this gentleman’s age doesn’t significantly
impact on his outcome.

B Presence of nodularity

Presence of nodularity is incorrect. Nodular melanomas are the most aggressive type,
although it is still thickness which has the greatest impact on outcomes.

C Presence of ulceration
Presence of ulceration is incorrect. Ulceration may be associated with accentuated spread of
melanoma, although it is less important versus the thickness of the lesion.

D Site of the lesion

Site of the lesion is incorrect. Lesions on the trunk are associated with increased risk of
metastases versus distant lesions on the lower limbs, but the impact of site is smaller than
that of thickness of the lesion.
51135

Rate this question:

Next Question

Previous Question Tag Question

Feedback End Review

Difficulty: Average

Peer Responses %

Q. Answered Flagged

Q1

Q2

Q3

Q4

Q5

Q6

Q7
Q. Answered Flagged

Q8

0:00:14/03:00:00

A 68-year-old man presents with gradually worsening confusion and unsteadiness on his feet
over the past 3 months. He is referred by the Emergency Department having suffered a
tonic–clonic seizure whilst out shopping with his wife. You allow him to wake up after his
seizure. His BP is 122/82 mmHg, pulse is 80 bpm and regular. His Mini-Mental State
Examination (MMSE) is 25/30. You confirm cerebellar dysfunction on neurological exam. MRI
reveals two hypodense lesions on T2-weighted scan suspicious of primary CNS lymphoma.
He is HIV negative.
Which of the following is most likely to be associated with his presentation?

A Cytomegalovirus

B Epstein–Barr virus (EBV)

C Herpes simplex virus type 1

D JC virus

E Varicella zoster virus

Explanation 

B Epstein–Barr virus (EBV)

Genomic material from EBV has been detected within primary CNS lymphoma cells in both
immunocompromised and non-immunocompromised individuals, although its presence is by
no means universal. It may however be responsible for clonal expansion of B cells which then
develop into a lymphoma.

A Cytomegalovirus

Cytomegalovirus (CMV) is incorrect. CMV is associated with encephalitis in patients who are
immunocompromised and may be associated with lymphoma, although this is less strong an
association than that for EBV.

C Herpes simplex virus type 1


Herpes simplex virus type 1 (HSV1) is incorrect. HSV1 is a cause of encephalitis in episodes of
acute infection, but isn’t well recognised as a cause of CNS lymphoma. HSV encephalitis is
associated with progressive confusion and a gradual decrease in consciousness over a few
days.

D JC virus

JC virus is incorrect. JC virus is a cause of progressive multi-focal leukoencephalopathy in


patients who are immunocompromised.

E Varicella zoster virus

Varicella zoster virus (VZV) is incorrect. VZV is a cause of encephalitis and can be suspected
in patients with progressive confusion and interruption or deterioration in short-term memory
and consciousness level.
51136

Rate this question:

Next Question

Previous Question Tag Question

Feedback End Review

Difficulty: Average

Peer Responses %

Q. Answered Flagged

Q1

Q2
Q. Answered Flagged

Q3

Q4

Q5

Q6

Q7

Q8
0:00:14/03:00:00

A 74-year-old man with poorly controlled chronic obstructive pulmonary disease (COPD)
comes to the Respiratory Clinic for review. Despite triple therapy with salmeterol, fluticasone
and tiotropium, he has suffered two exacerbations over the last 4 months and is finding it
very difficult to climb the stairs at home. Forced expiratory volume in 1 second (FEV 1) is 38%
of predicted.
A decision is made to start roflumilast therapy. Which of the following is the mode of action
of roflumilast?

A Anti-IL5 monoclonal antibody

B CTLA-4 immunoglobulin

C Phosphodiesterase-4 (PDE-4) inhibitor

D PDE-5 inhibitor

E PDE-6 inhibitor

Explanation 

C Phosphodiesterase-4 (PDE-4) inhibitor

Roflumilast is a PDE-4 inhibitor indicated according to NICE guidelines for patients who fail
to gain control of symptoms of COPD on triple inhaled therapy with long-acting b agonist,
long-acting muscarinic antagonist and high-dose steroid therapy. By inhibiting PDE-4,
roflumilast leads to elevated intracellular Cyclic adenosine monophosphate (cAMP) levels and
mitigates COPD-related malfunctions of leukocytes, airway and pulmonary vascular smooth
muscle cells, endothelial and airway epithelial cells and fibroblasts in experimental models.

A Anti-IL5 monoclonal antibody

Anti-IL5 monoclonal antibody is incorrect. Mepolizumab is a treatment for asthma where


there are significantly elevated levels of eosinophils and a failure to gain control of symptoms
on maximal conventional therapies. Inhibiting the action of IL-5 leads to a significant
reduction in eosinophils by 84%, in clinical trials, by week 32.
B CTLA-4 immunoglobulin

CTLA-4 immunoglobulin is incorrect. Abatacept is a CTLA-4 immunoglobulin which interferes


with T-cell co-stimulation; it is a treatment for rheumatoid arthritis.

D PDE-5 inhibitor

PDE-5 inhibitor is incorrect. PDE-5 inhibition is a target for the treatment of erectile
dysfunction and is the mode of action of drugs such as sildenafil.

E PDE-6 inhibitor

PDE-6 inhibitor is incorrect. PDE-6 is involved in rod and cone function. Cross inhibition of
PDE-6 occurs in patients prescribed high-dose PDE-5 inhibitors for erectile dysfunction. It
leads to blue discoloration of vision.
51214

Rate this question:

Next Question

Previous Question Tag Question

Feedback End Review

Difficulty: Average

Peer Responses %

Q. Answered Flagged

Q1

Q2
Q. Answered Flagged

Q3

Q4

Q5

Q6

Q7

Q8 

 External Links

Chronic obstructive pulmonary disease in over 16s: diagnosis and management


nice.org.uk/guidance/cg101
(https://www.nice.org.uk/guidance/cg101 )
0:00:14/03:00:00

You are asked to deliver a synchronised shock to a 32-year-old man who has been
anaesthetised following admission with paroxysmal SVT.
With which part of the ECG trace is the electricity delivered via the shock synchronised?

A P wave

B Q wave

C R wave

D S wave

E T wave

Explanation 

C R wave

The electricity is synchronised with the peak of the QRS complex, (the R wave). This why
there is a delay when the shock button is pressed, whilst the machine synchronises with the
ECG trace.

A P wave

P wave is incorrect. The P wave indicates atrial depolarisation and is not a target of the
synchronised low-energy DC shock.

B Q wave

Q wave is incorrect. Pathologic Q waves are a sign of previous myocardial infarction. Small Q
waves would be considered normal in most ECG lead positions. They are considered
pathological when they are more than 2 mm deep.

D S wave
S wave is incorrect. The S wave represents depolarisation of the Purkinje fibres and is not a
target of the synchronised shock.

E T wave

T wave is incorrect. The T wave represents cardiac repolarisation, and if the shock is
delivered in synchrony with the T wave, ventricular tachycardia can result.
51164

Rate this question:

Next Question

Previous Question Tag Question

Feedback End Review

Difficulty: Average

Peer Responses %

Q. Answered Flagged

Q1

Q2

Q3

Q4

Q5

Q6

Q7
Q. Answered Flagged

Q8
0:00:14/03:00:00

A 35-year-old man presents to the Emergency Department with gradually worsening


weakness of his left leg and confusion, which has increased over the past 6–8 weeks.
According to his male partner, he has become increasingly unsteady on his feet, with short-
term memory loss and some tremor affecting his head, in particular. On examination, his BP is
132/82mmHg, pulse is 72 bpm and regular. You note a conjugate gaze palsy with a head
tremor and cerebellar signs and confirm the weakness of his left leg. Routine blood testing
reveals a low white cell count at 3.4 × 10 9/l and MRI scanning reveals multiple white matter
lesions, particularly in the parieto-occipital region.

Which of the following is the most likely diagnosis?

A Cerebral toxoplasmosis

B Cryptococcal meningitis

C HIV encephalopathy

D Multi-infarct disease

E Progressive multi-focal leukoencephalopathy (PML)

Explanation 

E Progressive multi-focal leukoencephalopathy (PML)

The history of gradual deterioration over a few weeks, white matter lesions on MRI scanning
and low white cell count in a patient at risk of HIV fit with immunosuppression and
subsequent PML, due to (JC) virus. Symptoms may improve on initiation of Highly Active
Anti-Retroviral Therapy (HAART).

A Cerebral toxoplasmosis

Cerebral toxoplasmosis is incorrect. Cerebral toxoplasmosis is associated with cystic lesions


which are associated with oedema and contrast enhancement, rather than the picture seen
here.

B Cryptococcal meningitis
Cryptococcal meningitis is incorrect. Cryptococcal meningitis is associated with a subacute
presentation, often with headaches, ataxia and a mild fever. MRI scanning in cryptococcal
meningitis usually shows basal ganglia pseudocysts.

C HIV encephalopathy

HIV encephalopathy is incorrect. HIV encephalopathy is associated with a more prolonged


presentation and loss of grey matter volume in the cortex and basal ganglia.

D Multi-infarct disease

Multi-infarct disease is incorrect. Although multi-infarct disease is associated with multiple


white matter lesions, it would be unexpected in a man of 35 years, particularly in the absence
of vascular risk factors.
51205

Rate this question:

Next Question

Previous Question Tag Question

Feedback End Review

Difficulty: Average

Peer Responses %

Q. Answered Flagged

Q1

Q2

Q3
Q. Answered Flagged

Q4

Q5

Q6

Q7

Q8 
0:00:14/03:00:00

A 56-year-old woman presents to the clinic with multiple flaccid blisters in her mouth and
over her skin which she tells you are painful but not particularly itchy and frequently burst to
leave an erythematous base. The blisters began in her mouth and then started appearing on
her skin a few months later. Her only medication is oral ramipril for control of blood pressure
and a PRN salbutamol inhaler for mild asthma. You confirm the blisters on skin examination,
with extensive lesions within the oral cavity.
Which of the following are involved in the pathogenesis of her condition?

A Anti-COL17 antibodies

B Anti-DSG3 antibodies

C Anti-nuclear antibodies

D Anti-smooth muscle antibodies

E Anti-tissue transglutamase antibodies

Explanation 

B Anti-DSG3 antibodies

The differentials here with multiple flaccid blisters on examination are pemphigus and
pemphigoid. Pemphigus is most associated with blisters in the oro-pharynx, which may
appear months before the skin rash. Desmogleins are the autoantigens in pemphigus, and
anti-DSG1 antibodies are most commonly found.

A Anti-COL17 antibodies

Anti-COL17 is incorrect. COL17 is type 17 collagen, which is the autoantigen involved in the
development of pemphigoid, which is less likely to be associated with blisters affecting
mucus membranes versus pemphigus.

C Anti-nuclear antibodies
Anti-nuclear antibodies is incorrect. Anti-nuclear antibodies are associated with SLE, which is
associated with an erythematous rash on sun-exposed areas rather than the blistering rash
seen here.

D Anti-smooth muscle antibodies

Anti-smooth muscle antibodies is incorrect. Anti-smooth muscle antibodies are found in up


to 70% of patients with chronic active hepatitis, and 25–40% of patients with primary biliary
cirrhosis; they are not associated with pemphigus.

E Anti-tissue transglutamase antibodies

Anti-tissue transglutamase antibodies is incorrect. Anti-TTG antibodies are associated with


dermatitis herpetiformis, which is associated with the formation of small papules and vesicles,
rather than the large flaccid blisters seen here.
51139

Rate this question:

Next Question

Previous Question Tag Question

Feedback End Review

Difficulty: Average

Peer Responses %

Q. Answered Flagged

Q1

Q2
Q. Answered Flagged

Q3

Q4

Q5

Q6

Q7

Q8
0:00:14/03:00:00

A 38-year-old man with a history of ulcerative colitis comes to the Gastroenterology Clinic for
review. He presented with tiredness and lethargy over the past 6–9 months despite his
ulcerative colitis being well controlled on oral mesalazine. On examination, his BP is 122/82
mmHg, pulse is 74 bpm and regular. Abdomen is soft and non-tender, BMI is 22 kg/m 2. Blood
testing has revealed an elevated alkaline phosphatase and MRCP has shown areas of bile duct
structuring and dilatation.
Which of the following is the most appropriate intervention?

A Cholestyramine

B Colchicine

C Mycophenolate

D Penicillamine

E Ursodeoxycholic acid

Explanation 

E Ursodeoxycholic acid

Ursodeoxycholic acid has been shown to improve liver function tests in patients with early
primary sclerosing cholangitis (PSC), the diagnosis here. Data are conflicting about use in late
disease because of an imbalance of liver adverse events in favour of placebo in this
population.

A Cholestyramine

Cholestyramine is incorrect. Cholestyramine has no impact on disease progression in PSC,


although it may relieve symptoms of itching in late disease.

B Colchicine

Colchicine is incorrect. Although colchicine has anti-inflammatory properties, it has no


significant impact on prognosis in patients with PSC.
C Mycophenolate

Mycophenolate is incorrect. Similar to colchicine, although PSC may be associated with


chronic inflammation, there is no evidence that mycophenolate impacts on outcomes in the
condition.

D Penicillamine

Penicillamine is incorrect. Although copper excretion may be disordered in patients with early
PSC, there is no evidence that using a chelating agent such as penicillamine has any impact
on outcomes.
51199

Rate this question:

Next Question

Previous Question Tag Question

Feedback End Review

Difficulty: Average

Peer Responses %

Q. Answered Flagged

Q1

Q2

Q3

Q4

Q5
Q. Answered Flagged

Q6

Q7

Q8

0:00:14/03:00:00

A 19-year-old man comes to the Respiratory Clinic for review. He suffers from recurrent nasal
and sinus infections every winter and has most recently had a spell in hospital with left lower
lobe pneumonia. Other past history of note includes an episode of salmonella some 3 years
earlier which required treatment with antibiotics. On examination, his BP is 122/82 mmHg,
pulse is 72 bpm and regular. He is apyrexial. There are a few residual crackles at the left base
on auscultation. Abdomen is soft and non-tender. He is 180 cm tall. BMI is 25 kg/m 2.
Investigations:

Hb 130 g/l

WCC 7.1 x 10 9/l

PLT 203 x 10 9/l

Na + 140 mmol/l

K+ 4.5 mmol/l

Cr 95 micromol/l

Which of the following is the most likely diagnosis?

A Ataxia telangiectasia

B Cystic fibrosis

C IgA deficiency

D IgD deficiency

E IgE deficiency

Explanation 

C IgA deficiency

Selective IgA deficiency most commonly presents with recurrent sinopulmonary or


gastrointestinal infection, as here. Patients who receive blood products containing
immunoglobulins without knowing they are deficient in IgA may also suffer an allergic
reaction.

A Ataxia telangiectasia

Ataxia telangiectasia is incorrect. Ataxia telangiectasia is associated with immunodeficiency,


although it is also associated with progressive neurological dysfunction and increased risk of
solid tumours.

B Cystic fibrosis

Cystic fibrosis is incorrect. Cystic fibrosis is associated with failure to clear mucosal
secretions, exocrine and later endocrine pancreatic insufficiency; this patient’s limited
symptoms make cystic fibrosis unlikely.

D IgD deficiency

IgD deficiency is incorrect. Although selective IgD deficiency is recognised, it isn’t usually
associated with increased risk of infection.

E IgE deficiency

IgE deficiency is incorrect. Data on any association between IgE deficiency and increased risk
of infection are conflicting; it may however be associated with increased risk of autoimmune
disease.
51169

Rate this question:

Next Question

Previous Question Tag Question

Feedback End Review

Difficulty: Average

Peer Responses %
Q. Answered Flagged

Q1

Q2

Q3

Q4

Q5

Q6

Q7

Q8

Q9

0:00:14/03:00:00

A 44-year-old woman who is ACh receptor antibody positive presents to the Neurology
Clinic for review. Despite using high-dose prednisolone and pyridostigmine, she still
complains of ocular muscle weakness and problems with weak speech and swallowing
difficulties.
Which of the following is the most appropriate next intervention?

A Azathioprine

B Cyclophosphamide

C Methotrexate

D Rituximab

E Thymectomy

Explanation 

A Azathioprine

This patient has myasthenia gravis and has not responded to intervention with prednisolone
and pyridostigmine alone. Typical steroid-sparing (second-line) agents include azathioprine,
mycophenolate and cyclosporine. Studies suggest that up to 12 months of therapy is required
to gain sufficient response from a second-line therapy to reduce the corticosteroid dose.

B Cyclophosphamide

Cyclophosphamide is incorrect. Cyclophosphamide is an intervention most commonly used in


acute pulmonary and renal vasculitis related to granulomatosis with polyangiitis.

C Methotrexate

Methotrexate is incorrect. Methotrexate is the preferred second-line agent in the treatment of


rheumatoid arthritis, psoriasis, and psoriatic arthritis.

D Rituximab
Rituximab is incorrect. Rituximab is an anti-CD20 monoclonal antibody which reduces B-cell
activity and hence antibody production. It is used for the treatment of patients with
myasthenia which is resistant to conventional therapies.

E Thymectomy

Thymectomy is incorrect. Thymectomy is reserved for patients with myasthenia which is


resistant to medical therapy and for those patients where there is an obvious thymoma
identified.
51183

Rate this question:

Next Question

Previous Question Tag Question

Feedback End Review

Difficulty: Average

Peer Responses %

Q. Answered Flagged

Q1

Q2

Q3

Q4

Q5

Q6
Q. Answered Flagged

Q7

Q8

0:00:14/03:00:00

A 34-year-old man presents to the Endocrine Clinic for review. He has diet-controlled type 2
diabetes which also affects his father, uncle and two brothers, who presented at an early age
and are still controlled with diet alone. He has no past medical history of note and takes no
regular medication. On examination, his BP is 122/82 mmHg, pulse is 67 bpm and regular.
Abdomen is soft and non-tender with no masses, BMI is 25 kg/m 2. Random glucose in the
clinic is recorded at 7.4 mmol/l.
Which of the following is the most likely cause of his diabetes?

A Glucokinase mutation

B HNF-1 alpha mutation

C LADA

D MEN-1

E Standard type 2 diabetes

Explanation 

A Glucokinase mutation

The hints here that the diabetes is due to a glucokinase mutation include the autosomal-
dominant inheritance pattern of type 2 diabetes, the relatively normal BMI at diagnosis and
long-term control with diet alone. Glucokinase mutations, in essence, lead to a defect in
sensing blood glucose outside the normal range; with diet control alone, risk of long-term
complications is low.

B HNF-1 alpha mutation

HNF-1 alpha mutation is incorrect. HNF-1 alpha mutations are associated with a more marked
elevation in blood glucose than those seen here, and diet control alone is not effective. Most
patients therefore require at least a sulphonylurea to limit hyperglycaemia.

C LADA
LADA is incorrect. LADA is associated with positive antibodies against beta-cell antigens or
insulin, and doesn’t follow an autosomal-dominant pattern of inheritance, as is suspected
here.

D MEN-1

MEN-1 is incorrect. MEN-1 is a rare cause of pancreatic tumours; although glucagonoma in the
context of MEN-1 can present with diabetes mellitus, it is very rare, and is usually associated
with weight gain.

E Standard type 2 diabetes

Standard type 2 diabetes is incorrect. The autosomal-dominant pattern and the relatively
early age and weight at the lower end of that expected fits better with genetic, rather than
standard, type 2 diabetes.
51144

Rate this question:

Next Question

Previous Question Tag Question

Feedback End Review

Difficulty: Average

Peer Responses %

Q. Answered Flagged

Q1

Q2
Q. Answered Flagged

Q3

Q4

Q5

Q6

Q7

Q8
0:00:14/03:00:00

A 42-year-old woman presents to the Rheumatology Clinic with chronic fatigue which has
worsened significantly over the last 3 months. She also has night sweats, joint pains and
gradual weight loss of 4 kg. There is a photosensitive rash which is particularly affecting her
face and the uncovered portion of her arms and legs. On examination, her BP is 148/82
mmHg, pulse is 72 bpm and regular. You note evidence of a small joint polyarthritis and
confirm the rash.
Investigations:

Hb 109 g/l

WCC 8.9 x 10 9/l

PLT 139 x 10 9/l

Na + 140 mmol/l

K+ 4.9 mmol/l

Cr 112 micromol/l

Urine blood +, protein +

Anti-nuclear antibody positive

PT 12.2 sec

APTT 48.2 sec

Which of the following is the most appropriate initial intervention?

A Azathioprine

B Cyclophosphamide

C Hydroxychloroquine

D Methotrexate

E Mycophenolate mofetil

Explanation 
C Hydroxychloroquine

Hydroxychloroquine is the intervention of choice as initial therapy for lupus (the diagnosis
here). Cutaneous manifestations of the disease respond to hydroxychloroquine within days,
although other clinical manifestations of the disease may take 6–12 weeks before significant
impact is seen.

A Azathioprine

Azathioprine is incorrect. Azathioprine is less effective than hydroxychloroquine in treating


lupus, although it may be preferred to other options in certain situations, such as pregnancy,
where it is considered a relative safe intervention.

B Cyclophosphamide

Cyclophosphamide is incorrect. Cyclophosphamide is reserved for life-threatening disease,


such as rapidly progressive lupus nephritis.

D Methotrexate

Methotrexate is incorrect. Methotrexate is effective in treating joint disease related to


systemic lupus erythematosus (SLE), although it may not be as effective in treating SLE-
related skin manifestations versus hydroxychloroquine.

E Mycophenolate mofetil

Mycophenolate mofetil is incorrect. Mycophenolate mofetil is the intervention of choice for


patients with SLE who have significant evidence of lupus nephritis.
51219

Rate this question:

Next Question

Previous Question Tag Question

Feedback End Review

Difficulty: Average

Peer Responses %
Q. Answered Flagged

Q1

Q2

Q3

Q4

Q5

Q6

Q7

Q8

Q9

0:00:14/03:00:00

A 49-year-old woman with a history of systemic lupus erythematosus (SLE) comes to the
Renal Clinic for review. She has evidence of worsening disease activity despite treatment
with both hydroxychloroquine and high-dose corticosteroids. Her creatinine has increased
from 112 mmol/l to 145 mmol/l over the last 2 months, and she complains of constant joint
pains. Her erythrocyte sedimentation rate (ESR) is elevated at 54 mm/hr, and C-reactive
protein (CRP) is 89 mg/l.
Which of the following is the most appropriate next intervention?

A Belimumab

B Infliximab

C Mepolizumab

D Omalizumab

E Toclizumab

Explanation 

A Belimumab

Belimumab is a monoclonal antibody which inhibits activation of the B-lymphocyte stimulator


(BLyS) receptor. Inhibiting BLyS activation downregulates B-lymphocyte activity and is an
intervention in SLE for patients who fail to gain control of their disease treated with
corticosteroids and conventional therapies alone.

B Infliximab

Infliximab is incorrect. Infliximab is an anti-tumour necrosis factor (TNF) monoclonal antibody


used in the treatment of other arthritides and inflammatory bowel disease, rather than SLE.

C Mepolizumab
Mepolizumab is incorrect. Mepolizumab is an anti-IL5 monoclonal antibody which reduces
eosinophils, it is therefore of particular value in patients with allergic asthma who fail to
respond to conventional therapies.

D Omalizumab

Omalizumab is incorrect. Omalizumab targets immunoglobulin E and like mepolizumab, can


be used in patients with asthma resistant to other therapies.

E Toclizumab

Toclizumab is incorrect. Toclizumab is an anti-IL6 antibody. It is most useful in the treatment


of rheumatoid arthritis, although some evidence does also support its use in SLE.
51220

Rate this question:

Next Question

Previous Question Tag Question

Feedback End Review

Difficulty: Average

Peer Responses %

Q. Answered Flagged

Q1

Q2

Q3
Q. Answered Flagged

Q4

Q5

Q6

Q7

Q8
0:00:14/03:00:00

A 35-year-old woman comes to the Emergency Department with a severe headache, nausea
and vomiting over the past 24 hours. On further questioning, she has suffered from morning
headaches, worse on coughing, straining and lying flat over the past few months. On
examination, her BP is 155/95 mmHg, pulse is 80 bpm and regular. Her BMI is 35 kg/m 2. There
is bilateral papilloedema on fundoscopy. Blood testing reveals raised D-dimers.
Which of the following is the most useful investigation?

A Autoimmune profile

B Computed tomography (CT) head

C Lumbar puncture

D Magnetic resonance (MR) venogram

E Thrombophilia screen

Explanation 

D Magnetic resonance (MR) venogram

The history of chronic postural headache, worse in the mornings, in an obese young woman,
is suggestive of idiopathic intracranial hypertension (IIH). Patients with obesity and IIH are at
increased risk of cerebral venous sinus thrombosis, which, given the sudden worsening of
headache and papilloedema is the most likely diagnosis. The elevated D-dimer result fits well
with a large thrombosis.

A Autoimmune profile

Autoimmune profile is incorrect. Autoimmune profiling is used to detect anti-phospholipid


antibodies which are associated with a prolonged activated partial thromboplastin time
(APTT) and increased risk of blood clotting.

B Computed tomography (CT) head


CT head is incorrect. CT head may not reveal sufficient evidence to confirm a venous
thrombosis, although classically, the d sign, (a superior sagittal sinus thrombosis), may be
visualised.

C Lumbar puncture

Lumbar puncture is incorrect. Lumbar puncture may be performed if infection is suspected,


although where there is raised intra-cranial pressure, as here, it is contraindicated.

E Thrombophilia screen

Thrombophilia screen is incorrect. A thrombophilia screen to identify inherited thrombophilia,


including Protein C, Protein S deficiency and the Factor V Leiden variant could be considered,
although they won’t confirm or refute the underlying diagnosis.
51184

Rate this question:

Next Question

Previous Question Tag Question

Feedback End Review

Difficulty: Average

Peer Responses %

Q. Answered Flagged

Q1

Q2

Q3
Q. Answered Flagged

Q4

Q5

Q6

Q7

Q8 
0:00:14/03:00:00

A 45-year-old man comes to the Diabetes Clinic for review. He is overweight with a history of
hypertension and was diagnosed with type 2 diabetes at a Well Man Clinic, with a fasting
glucose of 7.2 mmol/l and an HbA1c of 51 mmol/mol. There is a history of type 1 diabetes in
the family and antibody testing has revealed positive anti-GAD antibodies. On examination,
his BP is 152/82 mmHg, pulse is 72 bpm and regular. His BMI is 35 kg/m 2. Triglycerides are
elevated.
Which of the following is the most appropriate way to manage his diabetes, initially?

A Empagliflozin

B Gliclazide

C Insulin

D Metformin

E Saxagliptin

Explanation 

D Metformin

Although this person has anti-GAD antibodies, he fits the phenotype of a typical patient with
type 2 diabetes and insulin resistance. As such, the default initial treatment of choice is still
metformin, but studies suggest progression to insulin may be more rapid in patients who are
GAD positive versus those who are not.

A Empagliflozin

Empagliflozin is incorrect. Metformin is the first-line option for the treatment of patients with
type 2 diabetes, and there is no evidence that SGLT-2 inhibitors are a better option for
patients with LADA, (who are anti-GAD positive).

B Gliclazide
Gliclazide is incorrect. Gliclazide is not a preferred option because evidence suggests that
although sulphonylureas achieve an early marked improvement in glucose control, over the
longer term, they may accelerate deterioration in HbA1c.

C Insulin

Insulin is incorrect. Insulin isn’t an appropriate initial option here, although evidence suggests
in patients who are GAD positive, progression to insulin is more rapid, this patient can be
successfully treated with orals at least for the first few years after diagnosis.

E Saxagliptin

Saxagliptin is incorrect. DPPIV inhibitors such as saxagliptin would only be considered in the
event that metformin was unsuitable or not tolerated.
51145

Rate this question:

Next Question

Previous Question Tag Question

Feedback End Review

Difficulty: Average

Peer Responses %

Q. Answered Flagged

Q1

Q2

Q3
Q. Answered Flagged

Q4

Q5

Q6

Q7

Q8 
0:00:14/03:00:00

A 68-year-old man with a history of diabetes mellitus and hypertension comes to the clinic
complaining that he has been bumping into things on left-hand side. He has suffered a
previous trans-ischaemic attack (TIA) some 5 years earlier. On examination, his BP is 152/105
mmHg, pulse is 75 bpm and regular. Neurological assessment reveals a left homonymous
superior quadrantanopia.
Where is the most likely site of any lesion on computed tomography (CT) head?

A Left occipital lobe

B Left parietal lobe

C Right parietal lobe

D Optic chiasm

E Right temporal lobe

Explanation 

E Right temporal lobe

A superior quadrantanopia suggests damage to the temporal lobe, and given the visual loss
on the left-hand side, the lesion is most likely to be on the right.

A Left occipital lobe

Left occipital lobe is incorrect. Occipital lobe infarcts lead to a homonymous hemianopia
rather than a quadrantanopia.

B Left parietal lobe

Left parietal lobe is incorrect. A left parietal lobe would lead to a homonymous
quadrantanopia, although it is inferior rather than superior, and would affect the right-hand
side rather than the left, as here.

C Right parietal lobe


Right parietal lobe is incorrect. A right parietal lobe lesion leads to a left homonymous
inferior quadrantanopia.

D Optic chiasm

Optic chiasm is incorrect. Optic chiasm lesions, often related to a pituitary tumour, lead to a
bitemporal hemianopia.
51185

Rate this question:

Next Question

Previous Question Tag Question

Feedback End Review

Difficulty: Average

Peer Responses %

Q. Answered Flagged

Q1

Q2

Q3

Q4

Q5

Q6

Q7
Q. Answered Flagged

Q8 
0:00:14/03:00:00

A 67-year-old man with a history of type 2 diabetes, hypertension and cardiovascular disease
comes to the outpatient clinic for review. He takes a number of medications, is poorly
compliant with them and he has been admitted to the hospital three times over the past year
with cardiac failure. On examination, his BP is 132/82 mmHg, pulse is 78 bpm and regular.
Chest is clear and there is pitting oedema of both ankles. He has marked gum hypertrophy.
Which of the following medications is the most likely cause of his gum hypertrophy?

A Amlodipine

B Dapagliflozin

C Indapamide

D Ramipril

E Sitagliptin

Explanation 

A Amlodipine

Amlodipine, like other calcium channel antagonists, is a recognised cause of gum


hypertrophy. Clinical experience also suggests that improved oral hygiene, including regular
teeth cleaning by the dental hygienist, can reduce hypertrophy over the longer term.

B Dapagliflozin

Dapagliflozin is incorrect. Dapagliflozin is an SGLT-2 inhibitor which increases urinary glucose


excretion. The major concern about SGLT-2 inhibitors is increased risk of urinary tract and
genital mycotic infections.

C Indapamide

Indapamide is incorrect. Indapamide is a thiazide-like diuretic. It is associated with a lower


incidence of hyponatraemia than members of the thiazide class, and does not adversely
affect glucose metabolism.
D Ramipril

Ramipril is incorrect. Ramipril is an ACE inhibitor. ACE inhibitors are recognised to increase
the risk of angioedema but do not cause gum hypertrophy with chronic use.

E Sitagliptin

Sitagliptin is incorrect. Sitagliptin is a DPPIV inhibitor. DPPIV inhibitors reduce breakdown of


endogenous incretin hormones such as GLP1, potentiating their action in promoting glucose-
sensitive insulin release.
51152

Rate this question:

Next Question

Previous Question Tag Question

Feedback End Review

Difficulty: Average

Peer Responses %

Q. Answered Flagged

Q1

Q2

Q3

Q4

Q5
Q. Answered Flagged

Q6

Q7

Q8 
0:00:14/03:00:00

A 68-year-old man is brought to the Psychiatry Clinic for review. He is accompanied by his
wife, having retired from the solicitor business he owns because of inappropriate behaviour.
Apparently, over the past few months, he has developed increasingly bizarre actions, with
loss of inhibition, episodes of exposure, repetitive compulsive behaviours and rituals. Physical
examination in the clinic is entirely normal. You note that during the consultation, he begins
to repeat words that you have used when addressing him.
Which of the following is the most likely diagnosis?

A Alzheimer’s disease

B Frontal lobe dementia

C Hypomania

D Lewy body dementia

E Multi-infarct dementia

Explanation 

B Frontal lobe dementia

In frontal lobe dementia, bizarre and inappropriate behaviour is the usual presenting feature.
Repetitive behaviour including rituals is typical of the condition, with defects in memory
coming later.

A Alzheimer’s disease

Alzheimer’s disease is incorrect. Alzheimer’s disease is associated with earlier, more


prominent defects of working memory and executive function, without the bizarre behaviours
seen here.

C Hypomania
Hypomania is incorrect. Although disinhibition is seen in patients with hypomania,
presentation for the first time at age 68 would be unusual. Pressure of speech rather than
echolalia is the more likely feature.

D Lewy body dementia

Lewy body dementia is incorrect. Lewy body dementia is most commonly associated with
visual hallucinations, and is seen in patients with underlying Parkinson’s disease.

E Multi-infarct dementia

Multi-infarct dementia is incorrect. Multi-infarct dementia is seen in patients with risk factors
for vascular disease, and a stepwise change in personality, working memory and executive
function is seen.
51229

Rate this question:

Next Question

Previous Question Tag Question

Feedback End Review

Difficulty: Average

Peer Responses %

Q. Answered Flagged

Q1

Q2

Q3
Q. Answered Flagged

Q4

Q5

Q6

Q7

Q8 
0:00:14/03:00:00

A 25-year-old woman who is 12 weeks’ pregnant with her first child comes to the Emergency
Department for review. She is very concerned that she will catch chicken pox after her
husband was diagnosed with the infection that morning. As far as she is aware, she has no
history of chicken pox infection as a child, and has never been vaccinated. Examination
reveals no signs of chicken pox and she appears well.
Which of the following is the most appropriate intervention?

A Oral acyclovir

B Reassurance

C Varicella zoster immunoglobulin

D Varicella zoster serology testing

E Varicella zoster vaccination

Explanation 

D Varicella zoster serology testing

Some women may not remember infection from childhood with chicken pox; or symptoms
may have been so minor, they were dismissed as another infection. For this reason, serology
testing for immunity is recommended before initiating treatment. Treatment with
immunoglobulin is most effective when given within the first 10 days of exposure.

A Oral acyclovir

Oral acyclovir is incorrect. Oral acyclovir may be given in conjunction with immunoglobulin,
although it is immunoglobulin itself, which is the cornerstone of therapy in pregnant women
exposed to varicella zoster virus.

B Reassurance
Reassurance is incorrect. Chicken pox infection in pregnancy carries a significant burden,
including risk of encephalitis and pneumonitis, and increased risk of foetal loss. As such, post-
exposure prophylaxis against infection is considered effective.

C Varicella zoster immunoglobulin

Varicella zoster immunoglobulin (VZIG) is incorrect. VZIG is the intervention of choice in


patients who are confirmed as non-immune to varicella zoster after serology testing.

E Varicella zoster vaccination

Varicella zoster vaccination is incorrect. In patients who are found to be IgG negative pre-
pregnancy, or who have been identified as seronegative in pregnancy, vaccination should be
offered. As post-exposure prophylaxis, vaccination isn’t effective.
51206

Rate this question:

Next Question

Previous Question Tag Question

Feedback End Review

Difficulty: Average

Peer Responses %

Q. Answered Flagged

Q1

Q2

Q3
Q. Answered Flagged

Q4

Q5

Q6

Q7

Q8 

 External Links

Chickenpox in Pregnancy
rcog.org.uk/globalassets/documents/guidelines/gtg13.pdf
(https://www.rcog.org.uk/globalassets/documents/guidelines/gtg13.pdf)
0:00:14/03:00:00

A 44-year-old man is admitted to the Emergency Department with a tea cup full of fresh
haemoptysis, a severe headache and gradually worsening nausea over the past 2–3 weeks. He
is previously well although he smokes 10 cigarettes per day and has recently recovered from
an episode of influenza. On examination, his BP is 155/100 mmHg, pulse is 89 bpm and
regular. There are inspiratory crackles over both lung fields, more marked at the bases.
Abdomen is soft and non-tender.
Investigations:

Hb 109 g/l

WCC 10.7 x 10 9/l

PLT 302 x 10 9/l

ESR 11 mm/hr

Na + 140 mmol/l

K+ 5.5 mmol/l

Cr 231 mmol/l

Urine blood +++, protein +

Which of the features described is the strongest pointer to anti-glomerular basement


membrane (anti-GBM) disease versus vasculitis?

A BP of 155/100 mmHg

B Cr 231 µmol/l

C ESR 11 mm/hr

D Onset of disease after influenza

E Past history of smoking

Explanation 

C ESR 11 mm/hr
Erythrocyte sedimentation rate (ESR) should always be elevated in a patient with vasculitis,
whereas it is normal in patients with anti-GBM disease. Although we’re not given the results,
positive anti-GBM antibodies would be expected, with negative cANCA (a feature of
granulomatosis with polyangiitis).

A BP of 155/100 mmHg

BP of 155/100 mmHg is incorrect. Hypertension would be expected in patients with nephritis


related to vasculitis, and in nephritis related to anti-GBM disease.

B Cr 231 µmol/l

Cr 231 µmol/l is incorrect. Like elevated BP, elevated creatinine would be expected in
nephritis, regardless of the underlying cause.

D Onset of disease after influenza

Onset of disease after influenza is incorrect. Although anti-GBM disease is commoner after
influenza, it doesn’t completely rule out other causes of nephritis or pulmonary haemorrhage.

E Past history of smoking

Past history of smoking is incorrect. Anti-GBM disease is more likely in smokers, but other
potential causes of pulmonary haemorrhage or nephritis can’t be excluded.
51221

Rate this question:

Next Question

Previous Question Tag Question

Feedback End Review

Difficulty: Average

Peer Responses %
Q. Answered Flagged

Q1

Q2

Q3

Q4

Q5

Q6

Q7

Q8

Q9

0:00:14/03:00:00

A 66-year-old woman is admitted on the acute medical take with a urinary tract infection
which is resistant to multiple antibiotics but appears to be sensitive to gentamicin. She
weighs 60 kg and is initially treated with 300 mg once-a-day dosing. Her creatinine at
baseline is 122 micromol/l. A peak gentamicin level is measured at 4 mg/l. Trough is
measured at 2.1 mg/l.
Which of the following is the most appropriate way to manage her gentamicin dosing?

A Continue current regimen

B Decrease dose to 200 mg once a day

C Increase dosing interval to 36 h

D Omit dosing and re-test trough level in 24 h

E Dose at 100 mg BD

Explanation 

D Omit dosing and re-test trough level in 24 h

In this case, although the peak gentamicin level is acceptable, the trough level is too high.
Guidelines recommend that where the level is >2 mg/l, omission of dosing and re-testing in
24 hours is the acceptable strategy. Re-dosing can occur if clinically indicated when levels fall
below 1 mg/l, but the dosing window must be extended for subsequent doses.

A Continue current regimen

Continue current regimen is incorrect. Although the peak level is acceptable, the trough level
of gentamicin is too high. As such, an extension of the dose frequency is essential to reduce
the risk of nephron and ototoxicity.

B Decrease dose to 200 mg once a day


Decrease dose to 200 mg once a day is incorrect. This runs the risk of inadequate coverage
because of not achieving a high enough peak gentamicin level and failure to clear the
underlying infection.

C Increase dosing interval to 36 h

Increase dosing interval to 36 hours is incorrect. This can be considered if the level is
between 1 and 2 mg/l at trough point.

E Dose at 100 mg BD

Dose at 100 mg BD is incorrect. OD dosing is adequate for the treatment of urinary tract
infection, and modification of the dose interval is all that is required to achieve a safe
administration profile.
51153

Rate this question:

Next Question

Previous Question Tag Question

Feedback End Review

Difficulty: Average

Peer Responses %

Q. Answered Flagged

Q1

Q2

Q3
Q. Answered Flagged

Q4

Q5

Q6

Q7

Q8 

 External Links

Therapeutic Drug Monitoring (TDM)


surreyandsussex.nhs.uk/wp-content/uploads/2013/04/Gentamicin-Therapeutic-drug-monitoring.pdf
(https://www.surreyandsussex.nhs.uk/wp-content/uploads/2013/04/Gentamicin-
Therapeutic-drug-monitoring.pdf)
0:00:14/03:00:00

A 24-year-old man returns from a holiday to the Far East with a purulent urethral discharge
and pain on passing urine. He admits to unprotected sexual intercourse during his visit.
Clinical examination is unremarkable, although he is able to express some of the discharge,
which is sent to the lab for testing. Nucleic acid amplification testing (NAAT) is positive for
gonorrhoea; HIV testing is negative. He is highly penicillin allergic with an anaphylactic
response to benzylpenicillin some years ago.
Which of the following is the most appropriate intervention?

A Intra-muscular (IM) ceftriaxone

B Oral cefixime

C Oral ciprofloxacin

D Oral azithromycin

E Oral oxytetracycline

Explanation 

D Oral azithromycin

In this situation, options are relatively limited, although high-dose azithromycin (2g) is
recognised to be effective. It may be associated with significant GI disturbance in some
patients.

A Intra-muscular (IM) ceftriaxone

Intra-muscular (IM) ceftriaxone is incorrect. Given there is anaphylaxis to penicillin, with the
potential for cross-over allergy, ceftriaxone should be avoided, given there is an alternative.

B Oral cefixime

Oral cefixime is incorrect. Oral cefixime is used for patients where cephalosporins are
appropriate, or where an IM formulation is refused/is inappropriate.
C Oral ciprofloxacin

Oral ciprofloxacin is incorrect. Quinolones are not preferred for the treatment of gonorrhoea
infection due to high levels of resistance.

E Oral oxytetracycline

Oral oxytetracycline is incorrect. Like quinolones, there is widespread resistance to


tetracyclines. This class of agents is therefore not preferred in the treatment of gonorrhoea.
51207

Rate this question:

Next Question

Previous Question Tag Question

Feedback End Review

Difficulty: Average

Peer Responses %

Q. Answered Flagged

Q1

Q2

Q3

Q4

Q5

Q6
Q. Answered Flagged

Q7

Q8

0:00:14/03:00:00

A 32-year-old woman who is 18 weeks pregnant comes to the Emergency Department with
symptoms of a lower urinary tract infection. She has suffered from fevers and dysuria over
the past 24 hours, with lower abdominal pain. Examination reveals temperature is 37.9 °C,
pulse is 88 bpm and regular and BP is 105/70 mmHg. There is tenderness on palpation over
the lower abdomen.
Investigations:

Hb 130 g/l

WCC 12.9 x 10 9/l

PLT 212 x 10 9/l

Na + 140 mmol/l

K+ 4.5 mmol/l

Cr 84 mmol/l

CRP 82 mg/l

Urine positive for white cells, red cells, leukocytes.

Which of the following should not be used to treat her urinary tract infection?

A Amoxicillin

B Cefalexin

C Ciprofloxacin

D Nitrofurantoin

E Trimethoprim

Explanation 

C Ciprofloxacin
Quinolones are not recommended in pregnancy because they are recognised to increase the
risk of tendon rupture in animal models. In adults, increased risk of tendon rupture is seen in
patients prescribed quinolones in conjunction with oral corticosteroids.

A Amoxicillin

Amoxicillin is incorrect. Penicillins are considered safe in pregnancy, although there is


substantial resistance for urinary pathogens to amoxicillin, which drives other choices ahead
of it as options.

B Cefalexin

Cefalexin is incorrect. Cefalexin is considered a third choice option for the treatment of UTI in
pregnancy, with both nitrofurantoin and trimethoprim considered ahead of it, although it is
still used in patients with contraindications to the other two therapies.

D Nitrofurantoin

Nitrofurantoin is incorrect. Nitrofurantoin is the first-choice option for urinary tract infection
in pregnancy. It should not be used in patients with G6PD deficiency because it acts as an
oxidising agent, leading to increased risk of haemolysis.

E Trimethoprim

Trimethoprim is incorrect. Trimethoprim is a second-choice option for the treatment of


urinary tract infection in pregnancy. During the first trimester, it should be prescribed in
conjunction with folic acid.
51208

Rate this question:

Next Question

Previous Question Tag Question

Feedback End Review

Difficulty: Average

Peer Responses %
Q. Answered Flagged

Q1

Q2

Q3

Q4

Q5

Q6

Q7

Q8

Q9

0:00:14/03:00:00

A 49-year-old man with poorly controlled hypertension is prescribed indapamide 2.5 mg in


addition to ramipril for control of blood pressure. He achieves target BP and is interested in
how indapamide works.
Where is the site of action for indapamide?

A Ascending limb of the loop of Henle

B Collecting duct

C Descending limb of the loop of Henle

D Distal convoluted tubule

E Proximal convoluted tubule

Explanation 

D Distal convoluted tubule

Indapamide is a sulphonamide derivative with an indole ring, pharmacologically related to


thiazide diuretics, which acts by inhibiting the reabsorption of sodium in the distal
convoluted tubule. It increases the urinary excretion of sodium and chlorides and, to a lesser
extent, the excretion of potassium and magnesium, thereby increasing urine output and
having an antihypertensive action.

A Ascending limb of the loop of Henle

Ascending limb of the loop of Henle is incorrect. Loop diuretics act on the Na +–K +–2Cl −
symporter in the thick ascending limb of the loop of Henle to inhibit sodium, chloride and
potassium reabsorption. They do this by competing for the chloride binding site.

B Collecting duct

Collecting duct is incorrect. Aquaporin 2 channels are found in the collecting duct. Expression
of these channels is regulated by ADH, therefore impacting on total body water.
C Descending limb of the loop of Henle

Descending limb of the loop of Henle is incorrect. The descending limb of the loop of Henle
has low permeability to ions and urea, but is highly permeable to water.

E Proximal convoluted tubule

Proximal convoluted tubule is incorrect. Carbonic anhydrase inhibitors are recognised to act
on the proximal convoluted tubule.
51173

Rate this question:

Next Question

Previous Question Tag Question

Feedback End Review

Difficulty: Average

Peer Responses %

Q. Answered Flagged

Q1

Q2

Q3

Q4

Q5

Q6
Q. Answered Flagged

Q7

Q8

0:00:14/03:00:00

A 72-year-old man with a history of atrial fibrillation and ischaemic heart disease presents to
the Emergency Department by ambulance. He has awoken from sleep to walk to the
bathroom when he suffered sudden, painless visual field loss affecting his right eye. He has
gone from being able to read the paper to counting fingers only with high right eye. His BP is
155/95 mmHg and pulse is 75 bpm (atrial fibrillation). There is an afferent pupillary defect on
the right and a pale retina with vessel attenuation. You note bilateral carotid bruits.
Which of the following is the most appropriate intervention?

A Anterior chamber paracentesis

B Hyperbaric oxygen therapy

C Intra-arterial thrombolysis

D Intravenous (IV) pentoxifylline

E Ocular massage

Explanation 

C Intra-arterial thrombolysis

A meta-analysis of retrospective studies published in 2000 reported a potential benefit for


intra-arterial thrombolysis when compared with the natural history of central retinal artery
occlusion (the diagnosis here), with 15% of patients achieving normal vision and 27%
achieving 20/40 visual acuity or better, post therapy. A later trial was negative but this
included patients treated for up to 20 hours.

A Anterior chamber paracentesis

Anterior chamber paracentesis is incorrect. Although this reduces pressure in the eye acutely
and may dislodge a clot, evidence is anecdotal, with no trial evidence to decide on benefit.

B Hyperbaric oxygen therapy


Hyperbaric oxygen therapy is incorrect. Hyperbaric oxygen therapy used in an attempt to
maintain retinal perfusion has shown very mixed results in small clinical trials.

D Intravenous (IV) pentoxifylline

IV pentoxifylline is incorrect. Vasodilators, including pentoxifylline and nitrates, have been


used to improve retinal blood flow but trial evidence to support their use is weak.

E Ocular massage

Ocular massage is incorrect. If used within 90 minutes of onset of visual loss, this may
dislodge any clot, although the evidence for its use is weak and outcome may be very
dependent on technique.
51190

Rate this question:

Next Question

Previous Question Tag Question

Feedback End Review

Difficulty: Average

Peer Responses %

Q. Answered Flagged

Q1

Q2

Q3
Q. Answered Flagged

Q4

Q5

Q6

Q7

Q8

 External Links

Local intra-arterial fibrinolysis for acute occlusion of the central retinal artery: a meta-analysis of the publi…
ncbi.nlm.nih.gov/pubmed?term=10906103
(https://www.ncbi.nlm.nih.gov/pubmed?term=10906103)
0:00:14/03:00:00

A 37-year-old woman is referred to the Endocrine Clinic with diabetes mellitus, and
hypertension which is proving very difficult for her GP to manage. She also reports that her
facial appearance has changed, with a much more prominent lower jaw, facial acne and she
can no longer wear her rings because of swelling of her fingers. On examination, you note a
BP of 155/90 mmHg, pulse is 67 bpm and regular. She has evidence of greasy skin with some
facial and upper chest acne. There is obvious macroglossia and prognathism and her BMI is
24. Serum IGF-1 is markedly elevated.
Which of the following is the most appropriate initial intervention?

A Cabergoline

B Lanreotide

C Metyrapone

D Pegvisomant

E Trans-sphenoidal surgery

Explanation 

B Lanreotide

Lanreotide is a long-acting analogue of somatostatin, like octreotide. Intervention with


somatostatin analogues is thought to reduce perioperative morbidity when used pre-op.
Prolonged therapy is not however supported by trial evidence, with trials over 16 weeks
showing no effect on tumour shrinkage or risk of recurrence.

A Cabergoline

Cabergoline is incorrect. Cabergoline is a dopamine agonist; these agents are first-line


therapy for prolactinomas, and may be used in combination with somatostatin analogues for
acromegaly.

C Metyrapone
Metyrapone is incorrect. Metyrapone blocks corticosteroid production; it is used in the
treatment of Cushing’s related to adrenal carcinoma, not for the treatment of acromegaly.

D Pegvisomant

Pegvisomant is incorrect. Pegvisomant is a growth hormone antagonist which blocks


peripheral synthesis of IGF-1. It is considered a third-line agent for control of symptoms of
acromegaly in patients who fail to gain control on somatostatin analogue combined with
dopamine agonist.

E Trans-sphenoidal surgery

Trans-sphenoidal surgery is incorrect. This would be scheduled after appropriate work up,
including MRI to decide on the most appropriate approach to removal of the adenoma, and a
period of stabilisation with respect to blood pressure and glucose control.
51146

Rate this question:

Next Question

Previous Question Tag Question

Feedback End Review

Difficulty: Average

Peer Responses %

Q. Answered Flagged

Q1

Q2
Q. Answered Flagged

Q3

Q4

Q5

Q6

Q7

Q8
0:00:14/03:00:00

A 45-year-old woman with a history of systemic sclerosis comes to the clinic complaining of
intermittent abdominal pain, offensive eructation, bloating and diarrhoea. She has been
taking oral omeprazole for some years for reflux oesophagitis and has recently lost some
weight. Examination reveals typical changes of systemic sclerosis. Abdomen is soft and non-
tender. Breath testing reveals a significant increase in exhaled hydrogen at 90 minutes.
Which of the following is the most appropriate intervention?

A Ciprofloxacin

B Domperidone

C Amoxicillin

D Metoclopramide

E Metronidazole

Explanation 

E Metronidazole

The hydrogen breath testing here is suggestive of bacterial overgrowth, a recognised


consequence of systemic sclerosis. Metronidazole is the most appropriate intervention from
those listed, with tetracyclines or macrolides potential alternatives. A 7–10 day course of
metronidazole is thought to improve symptoms in 40–90% of patients.

A Ciprofloxacin

Ciprofloxacin is incorrect. Quinolones are a preferred option for symptomatic salmonella


infection rather than bacterial overgrowth.

B Domperidone

Domperidone is incorrect. Although prokinetics are utilised in the treatment of GI disturbance


associated with systemic sclerosis, the primary problem here is bacterial overgrowth.
C Amoxicillin

Amoxicillin is incorrect. Amoxicillin is not a preferred option for bacterial overgrowth therapy;
metronidazole, tetracyclines or macrolides are all potential options.

D Metoclopramide

Metoclopramide is incorrect. Metoclopramide is not recommended for long-term use as a


prokinetic because of neurological sequelae including the development of tardive dyskinesia.
Antibiotic therapy is also the preferred initial intervention here.
51210

Rate this question:

Next Question

Previous Question Tag Question

Feedback End Review

Difficulty: Average

Peer Responses %

Q. Answered Flagged

Q1

Q2

Q3

Q4

Q5
Q. Answered Flagged

Q6

Q7

Q8 
0:00:14/03:00:00

You are tasked with investigating if there is a difference in the methods of referral for
patients with chronic obstructive pulmonary disease (COPD) to the Emergency Department
by social class. Two methods of referral are compared, those who arrived via their GP and
those who came directly to the Emergency Department.
Which is the best statistical test to determine if the proportion of each social class differs
with respect to how they were referred in?

A Chi-squared test

B Mann–Whitney U test

C Pearson’s correlation coefficient

D T-test

E Wilcoxon signed rank sum test

Explanation 

A Chi-squared test

The Chi-squared test is most useful, as here, for comparing the distribution of categorical
variables across two different samples. Social class is a categorical variable and the
proportion of each class presenting via the GP or straight to A&E can be easily calculated.

B Mann–Whitney U test

Mann–Whitney U test is incorrect. The Mann–Whitney U can be used to compare the


distributions of two sets of unpaired data which may not be normally distributed.

C Pearson’s correlation coefficient

Pearson’s correlation coefficient is incorrect. Pearson’s correlation coefficient is used to


assess linear association between two different variables. An example might be height and
anatomical dead space on pulmonary function testing.
D T-test

T-test is incorrect. The t-test is used to compare two sample means, where the data are
normally distributed. An example where the t-test might be used could be to compare transit
times between two prokinetic agents used in the treatment of gastroparesis.

E Wilcoxon signed rank sum test

Wilcoxon signed rank sum test is incorrect. The Wilcoxon signed rank sum test is used to
compare the distribution of two sets of paired data.
51171

Rate this question:

Next Question

Previous Question Tag Question

Feedback End Review

Difficulty: Average

Peer Responses %

Q. Answered Flagged

Q1

Q2

Q3

Q4

Q5
Q. Answered Flagged

Q6

Q7

Q8 

 External Links

The Chi squared tests


bmj.com/about-bmj/resources-readers/publications/statistics-square-one/8-chi-squared-tests
(http://www.bmj.com/about-bmj/resources-readers/publications/statistics-square-
one/8-chi-squared-tests)
0:00:14/03:00:00

A 67-year-old man with a 40-pack per year smoking history comes to the Emergency
Department with nausea, vomiting and decreased consciousness. He has suffered from a
chronic cough over the past few months with occasional haemoptysis. On examination, his
BP is 112/82 mmHg, pulse is 92 bpm and regular. He looks dehydrated. There are decreased
breath sounds over the left upper zone on auscultation.

Hb 110 g/l

WCC 7.1 x 10 9/l

PLT 172 x 10 9/l

Na + 147 mmol/l

K+ 5.1 mmol/l

Cr 179 micromol/l

Ca 3.6 mmol/l

Left upper zone mass consistent with a bronchial carcinoma with associated
CXR
ipsilateral lymphadenopathy

Which of the following is the most likely diagnosis?

A Adenocarcinoma of the bronchus

B Bronchial carcinoid

C Large cell carcinoma of the bronchus

D Small cell carcinoma of the bronchus

E Squamous cell carcinoma of the bronchus

Explanation 

E Squamous cell carcinoma of the bronchus


Squamous cell bronchial carcinoma is associated with production of PTHrp which leads to
hypercalcaemia. Other causes of hypercalcaemia related to PTHrp include carcinoma of the
breast.

A Adenocarcinoma of the bronchus

Adenocarcinoma of the bronchus is incorrect. Adenocarcinoma of the bronchus is not usually


associated with ectopic hormone production. It is more common in non-smokers and is
associated with asbestos exposure.

B Bronchial carcinoid

Bronchial carcinoid is incorrect. Bronchial carcinoid presents in younger patients and is


usually associated with an isolated vascular lesion which presents with haemoptysis or
bronchial obstruction.

C Large cell carcinoma of the bronchus

Large cell carcinoma of the bronchus is incorrect. Large cell carcinoma makes up only 8% of
bronchial carcinomas; they are less differentiated forms of squamous and adenocarcinomas
and metastasise early.

D Small cell carcinoma of the bronchus

Small cell carcinoma of the bronchus is incorrect. Small cell carcinoma of the bronchus makes
up approximately 15% of total cases, metastasises early and is responsible for syndrome of
inappropriate antidiuretic hormone secretion (SIADH)and Cushing’s syndrome, in association
with lung cancer.
51137

Rate this question:

Next Question

Previous Question Tag Question

Feedback End Review

Difficulty: Average

Peer Responses %
Q. Answered Flagged

Q1

Q2

Q3

Q4

Q5

Q6

Q7

Q8

Q9

0:00:14/03:00:00

A 64-year-old man is reviewed in the Emergency Department immediately after arrival by


ambulance. He has suffered from central crushing chest pain, sweating and pre-syncope for
the past 2 hrs and has a history of hypertension and heavy smoking. On examination, his BP is
85/60 mmHg, pulse is 28 bpm. Heart sounds are normal and there are crackles at both bases
on auscultation of the lungs. ECG reveals 3 mm of ST elevation in the anterior chest leads and
complete heart block. There is no response to 3 mg of atropine.
Which of the following is the most appropriate next step?

A IV isoprenaline

B Percutaneous coronary intervention (PCI)

C Thrombolysis

D Abciximab

E Transvenous pacing

Explanation 

E Transvenous pacing

In this situation, with complete heart block and dangerously low blood pressure, pacing is
advised before progression to PCI. Although heart rate may improve post resolution of
ischaemia, this is less likely in the context of an anterior infarct.

A IV isoprenaline

IV isoprenaline is incorrect. IV isoprenaline will have limited effectiveness in the context of


ongoing ischaemia and is unlikely to significantly improve heart rate. As such, transvenous
pacing is the only appropriate intervention.

B Percutaneous coronary intervention (PCI)

PCI is incorrect. PCI should take place, although it is important to place the pacing wire first
as the procedure itself could lead to hypotension and further bradycardia.
C Thrombolysis

Thrombolysis is incorrect. In the context of ongoing ischaemia and an anterior myocardial


infarction, PCI is preferred to thrombolysis.

D Abciximab

Abciximab is incorrect. Abciximab is given at the time of PCI; if given before, it is recognised
to increase the risk of bleeding without improving outcomes.
51165

Rate this question:

Next Question

Previous Question Tag Question

Feedback End Review

Difficulty: Average

Peer Responses %

Q. Answered Flagged

Q1

Q2

Q3

Q4

Q5

Q6
Q. Answered Flagged

Q7

Q8

0:00:14/03:00:00

A 45-year-old woman presents with gradually worsening itching and lethargy over the past
few weeks. She says that she is unable to hold down her job because of tiredness and unable
to sleep properly due to persistent itching. She has no past medical history of note and takes
no regular medications. On examination, her BP is 132/85 mmHg, pulse is 72 bpm and regular.
You notice scratch marks over the upper body and a number of spider naevi consistent with
chronic hepatic impairment.
Investigations:

Hb 109 g/l

WCC 7.9 x 10 9/l

PLT 109 x 10 9/l

Na + 138 mmol/l

K+ 4.5 mmol/l

Cr 103 mmol/l

Albumin 25g/l

ALT 89 IU/l

Alkphos 302 IU/l

Bilirubin 19 mmol/l

Anti-mitochondrial antibody positive


Which of the following is the most appropriate intervention?

A High-dose prednisolone

B Methotrexate

C Penicillamine

D Rifampicin

E Ursodeoxycholic acid
Explanation 

E Ursodeoxycholic acid

Ursodeoxycholic acid is associated with an improvement in symptoms, markers of liver


dysfunction, and probably an outcome benefit in primary biliary cirrhosis (PBC), the most
likely diagnosis here. In spite of a Cochrane meta-analysis not confirming outcome benefits in
2007, subsequent trials appear to support an effect.

A High-dose prednisolone

High-dose prednisolone is incorrect. Although in clinical trials, high-dose prednisolone has


been shown to potentially slow progression of PBC, used as monotherapy, it is not
recommended due to a relatively modest benefit and significant adverse events associated
with therapy.

B Methotrexate

Methotrexate is incorrect. The benefit of methotrexate is uncertain. Although it appears to


slow progression in combination when used in patients who fail to respond to
ursodeoxycholic acid, these data do not come from the randomised controlled trial setting,
and five trials failed to confirm conclusive benefit in a meta-analysis.

C Penicillamine

Penicillamine is incorrect. Penicillamine has no positive benefit in the treatment of PBC, and
actually increases liver-related adverse events.

D Rifampicin

Rifampicin is incorrect. Rifampicin can be used in combination with other agents, such as
cholestyramine, for the treatment of itching. It isn’t, however, used as an initial intervention.
51200

Rate this question:

Next Question

Previous Question Tag Question

Feedback End Review

Difficulty: Average
Peer Responses %

Q. Answered Flagged

Q1

Q2

Q3

Q4

Q5

Q6

Q7

Q8

Q9

0:00:14/03:00:00

A 45-year-old woman presents to the Neurology Clinic some 6 weeks after suffering a road
traffic accident. She has suffered chronic neck pain since the crash and now has pain and loss
of sensation in her arms, worse on the left-hand side than the right. On examination, you note
weakness of elbow flexion and wrist extension, more marked on the left-hand side. There is
some sensory loss affecting the lateral forearm, thumb and index finger, more marked again
on the left-hand side.
Which of the following is the most likely diagnosis?

A Cervical spondylosis

B Fibromyalgia

C Prolapsed intervertebral disc

D Simple whiplash

E Syringomyelia

Explanation 

C Prolapsed intervertebral disc

Prolapsed intervertebral discs are commoner after a road traffic accident, and the features
here fit with compression of C6 nerve roots. It is usual for one side to have more marked
symptoms than the other. Analgesia and physiotherapy are key initial interventions, with
surgery where there are marked symptoms of nerve compression and changes seen on
magnetic resonance imaging (MRI).

A Cervical spondylosis

Cervical spondylosis is incorrect. Cervical spondylosis is due to chronic cervical disc


degeneration, and presents with severe neck pain, often referred up to the occiput or to
between the shoulder blades. It’s associated with severe limitation of neck movement which
gradually worsens over time.

B Fibromyalgia
Fibromyalgia is incorrect. Fibromyalgia is associated with pain over multiple points,
hypersensitivity to touch, stiffness and fatigue.

D Simple whiplash

Simple whiplash is incorrect. Simple whiplash is associated with pain and stiffness over the
neck, although features of radiculopathy would not be expected in this situation.

E Syringomyelia

Syringomyelia is incorrect. Syringomyelia is associated with sensory loss in a shawl-like


distribution over the upper limbs. There is often pain when the skin is touched, muscle
weakness and wasting begins in the hands.
51186

Rate this question:

Next Question

Previous Question Tag Question

Feedback End Review

Difficulty: Average

Peer Responses %

Q. Answered Flagged

Q1

Q2

Q3
Q. Answered Flagged

Q4

Q5

Q6

Q7

Q8
0:00:14/03:00:00

An 18-year-old man presents to the clinic with increased tiredness over the past few weeks
and tells you that he barely has the energy to attend college at the current time. On
examination, he looks pale, his BP is 112/82, pulse is 88 bpm and regular. He appears slightly
short of breath at rest. Abdomen is soft and non-tender, there is mild splenomegaly.
Investigations:

Hb 89 g/l

WCC 7.9 x 10 9/l

PLT 191 x 10 9/l

Na + 140 mmol/l

K+ 4.5 mmol/l

Cr 102 micromol/l

LDH 712 IU/l

Heinz bodies noted on the blood film.


Which of the following is the most likely diagnosis?

A G6PD deficiency

B Hereditary spherocytosis

C Hypersplenism

D Hyposplenism

E Pelger–Huet anomaly

Explanation 

A G6PD deficiency
There is strong evidence of haemolysis here, with mild splenomegaly, and anaemia with
raised LDH. The presence of Heinz bodies on the blood film, representing denatured
haemoglobin, is consistent with G6PD deficiency. Episodes of haemolysis can occur when
drugs which act as oxidising agents are prescribed to patients with G6PD deficiency.

B Hereditary spherocytosis

Hereditary spherocytosis is incorrect. This is associated with haemolysis, but it’s associated
with abnormally shaped red cells, rather than the presence of Heinz bodies.

C Hypersplenism

Hypersplenism is incorrect. Hypersplenism can be associated with anaemia, although red


cells containing Heinz bodies undergo enhanced removal in patients with the condition.

D Hyposplenism

Hyposplenism is incorrect. Hyposplenism may be associated with red cells containing Heinz
bodies, although it is also associated with other features such as Howell–Jolly bodies. Howell–
Jolly bodies are red cell nuclear fragments, whereas Heinz bodies represent denatured
haemoglobin.

E Pelger–Huet anomaly

Pelger–Huet anomaly is incorrect. These are bilobed neutrophils, which may be congenital
but are also seen in haematological conditions such as myelodysplasia.
51158

Rate this question:

Next Question

Previous Question Tag Question

Feedback End Review

Difficulty: Average

Peer Responses %
Q. Answered Flagged

Q1

Q2

Q3

Q4

Q5

Q6

Q7

Q8

Q9

0:00:14/03:00:00

An 18-year-old woman who is part of an elite athletics training group at her university comes
to the Endocrine Clinic for review. She is concerned because she has not had a menstrual
period for the past 7 months. She runs over 20 miles per week as part of her training. On
examination, her BP is 105/70 mmHg, pulse is 55 bpm and regular. Her BMI is 18 kg/m 2.
Routine full blood count and U&Es are normal.
Which of the following is the most likely cause for her amenorrhoea?

A Hyperthyroidism

B Hypopituitarism

C Low BMI

D Pregnancy

E Prolactinoma

Explanation 

C Low BMI

This patient remains an elite athlete, and apart from the low BMI, all other features of the
examination and investigations are unremarkable. As such, the most likely explanation for the
cessation of menses is this patient’s low weight, where pulsatility of LH and FSH is lost.

A Hyperthyroidism

Hyperthyroidism is incorrect. Hyperthyroidism is associated with gradually diminishing


menses rather than the absolute cessation seen here. Some proximal myopathy and
tachycardia would be expected that would preclude her sporting activities.

B Hypopituitarism

Hypopituitarism is incorrect. It is unlikely with panhypopituitarism that this patient could


maintain her sporting activity, and U&Es would be abnormal in this situation.
D Pregnancy

Pregnancy is incorrect. Pregnancy is unlikely given this patient’s low BMI and continued
athletic training. Given she has been amenorrhoeic for 7 months, some signs of pregnancy
would also be expected on physical examination.

E Prolactinoma

Prolactinoma is incorrect. Prolactinoma is associated with other symptoms apart from


amenorrhoea including galactorrhoea and decreased libido.
51147

Rate this question:

Next Question

Previous Question Tag Question

Feedback End Review

Difficulty: Average

Peer Responses %

Q. Answered Flagged

Q1

Q2

Q3

Q4

Q5
Q. Answered Flagged

Q6

Q7

Q8 
0:00:14/03:00:00

A 45-year-old woman is identified to have primary biliary cirrhosis (PBC), having presented a
few weeks earlier with progressive lethargy and icterus. She is commenced on obeticholic
acid therapy.
Which of the following is the mode of action of obeticholic acid?

A Farnesoid-X-receptor (FXR) agonist

B PPAR-α agonist

C PPAR-δ agonist

D PPAR-γ agonist

E Retinoid X-receptor agonist

Explanation 

A Farnesoid-X-receptor (FXR) agonist

Bile acids are FXR agonists, the receptor plays a role in bile acid homeostasis and local
inflammation. Obeticholic acid is around 100 times more potent for the receptor than
naturally occurring amino acids and is therefore thought to reduce hepatic dysfunction
associated with PBC. It does, however, lead to itching.

B PPAR-α agonist

PPAR-α agonist is incorrect. PPAR-α agonism does have positive effects on reducing hepatic
inflammation and steatosis, although this is targeted in combination with δ agonism in a
compound called elafibranor, currently in clinical trials for the treatment of non-alcoholic
steatohepatitis (NASH).

C PPAR-δ agonist

PPAR-δ agonist is incorrect. PPAR-δ and α agonism combined is a current therapeutic target
for the treatment of NASH
D PPAR-γ agonist

PPAR-γ agonist is incorrect. PPAR-γ agonists such as pioglitazone reduce central stores of fat.
They are used in the treatment of NASH and also type 2 diabetes mellitus because of this
property.

E Retinoid X-receptor agonist

Retinoid X-receptor agonist is incorrect. The retinoid X-receptor is important in cellular


growth and differentiation, it plays a role as a heterodimer with other nuclear receptors such
as PPAR in regulating gene expression.
51201

Rate this question:

Next Question

Previous Question Tag Question

Feedback End Review

Difficulty: Average

Peer Responses %

Q. Answered Flagged

Q1

Q2

Q3

Q4

Q5
Q. Answered Flagged

Q6

Q7

Q8

0:00:14/03:00:00

A 25-year-old woman presents with watery diarrhoea which has increased in frequency over
the past 24 hours, although has been present for 10 days. This is associated with a fever and
right iliac fossa (RIF) pain. She has no past medical history and her only medication is the
progestogen-only contraceptive pill. On examination, she is pyrexial at 37.9 °C, her BP is
95/70 mmHg, pulse is 87 bpm and regular. She looks dehydrated. Abdomen is soft, although
there is tenderness in the RIF and bowel sounds are very active.
Investigations:

Hb 115 g/l

WCC 13.2 x 10 9/l

PLT 203 x 10 9/l

Na + 141 mmol/l

K+ 3.4 mmol/l

Cr 132 mmol/l

CRP 89 mg/l

Stool culture sent the previous week by her GP is positive for Yersinia.
Which of the following is the most appropriate intervention?

A Amoxicillin

B Cephalexin

C Gentamicin

D Loperamide

E Ofloxacin

Explanation 

E Ofloxacin
Yersinia is a pathogen that causes diarrhoea in adults, and may also result in mesenteric
lymphadenopathy leading to abdominal pain and diagnostic confusion. It is sensitive to
treatment with quinolones, co-trimoxazole and tetracyclines. Ofloxacin is preferred here to
gentamicin because of the presence of pre-renal impairment.

A Amoxicillin

Amoxicillin is incorrect. Amoxicillin is not a preferred option for treatment of Yersinia


enterocolitica due to the fact many strains are β-lactamase producing.

B Cephalexin

Cephalexin is incorrect. Like amoxicillin, cephalexin is often not effective in treating Yersinia
infection due to β-lactamase-producing strains.

C Gentamicin

Gentamicin is incorrect. Gentamicin is not preferred here versus ofloxacin due to the
presence of pre-renal impairment. The bacterium is, however, sensitive to treatment with
gentamicin.

D Loperamide

Loperamide is incorrect. Although anti-motility agents may reduce symptoms of diarrhoea,


they are not recommended in Yersinia infection due to increasing the risk of local bacterial
invasion.
51211

Rate this question:

Next Question

Previous Question Tag Question

Feedback End Review

Difficulty: Average

Peer Responses %
Q. Answered Flagged

Q1

Q2

Q3

Q4

Q5

Q6

Q7

Q8

Q9

0:00:14/03:00:00

A 35-year-old man comes to the Emergency Department with acute, burning, throbbing pain
in his right shoulder, which he says came on abruptly and is now 9/10 in severity. He tells you
that he had an influenza vaccination some 3 days earlier. You note he is holding the arm
adducted and internally rotated. He is unable to make even minor movements of the shoulder
due to the increased pain. Movements of the neck appear pain free. Routine X-ray of the neck
and shoulder appears normal.
Which of the following is the most likely diagnosis?

A Acromioclavicular joint disruption

B Adhesive capsulitis

C Brachial neuritis

D Cervical radiculopathy

E Rotator cuff tear

Explanation 

C Brachial neuritis

The sudden onset of acute shoulder pain a short period after vaccination is typical of brachial
neuritis, a rare condition characterised by inflammation of the lower motor neurons of the
brachial plexus. Due to the level of pain involved, the shoulder is usually held in a fixed
position. After approximately 2 weeks, atrophy of affected muscles becomes apparent.

A Acromioclavicular joint disruption

Acromioclavicular joint disruption is incorrect. Acromioclavicular joint disruption results from


a fall onto the tip of the shoulder with the arm in adduction, or a direct blow to the acromion
with the humerus adducted, although there is no history of trauma here.

B Adhesive capsulitis
Adhesive capsulitis is incorrect. Adhesive capsulitis is associated with painful movements of
the shoulder in every direction, although the onset of pain and stiffness is gradual over a
number of days versus the rapid deterioration seen here.

D Cervical radiculopathy

Cervical radiculopathy is incorrect. Cervical radiculopathy is usually associated with pain


corresponding to a single nerve root which is subject to compression, rather than the pain
confined to just the shoulder itself, as seen here.

E Rotator cuff tear

Rotator cuff tear is incorrect. Rotator cuff tears are associated with pain on shoulder
abduction as a result of traumatic injury, rather than the acute onset related to vaccination
seen here.
51187

Rate this question:

Next Question

Previous Question Tag Question

Feedback End Review

Difficulty: Average

Peer Responses %

Q. Answered Flagged

Q1

Q2
Q. Answered Flagged

Q3

Q4

Q5

Q6

Q7

Q8
0:00:14/03:00:00

A 24-year-old woman who is 20 weeks’ pregnant with her first child is referred to the
Endocrine Clinic with weight loss, anxiety and palpitations. She tells you that she is finding it
impossible to maintain her weight and lies in bed at night feeling her heart pounding. On
examination, her BP is 112/75 mmHg, pulse is 88 bpm and regular. She has a fine tremor and
sweaty palms. You note a small, diffuse, smooth goitre on examination of her neck and mild
proptosis. TSH is <0.05 mU/l.
Which of the following is the most appropriate intervention?

A Carbimazole

B Carbimazole and thyroxine

C Propranolol

D Propylthiouracil

E Propylthiouracil and thyroxine

Explanation 

A Carbimazole

This patient has signs and symptoms consistent with new-onset Graves’ disease in pregnancy.
In this situation, untreated thyrotoxicosis carries significant risks to the unborn child with
respect to miscarriage. Intervention is therefore required, with carbimazole monotherapy the
best option at this stage of pregnancy, to minimise risk of maternal hepatic dysfunction.
Enough carbimazole should be given to maintain TSH at towards the lower end of the normal
range.

B Carbimazole and thyroxine

Carbimazole and thyroxine is incorrect. “Block-replace” cannot be used as a therapy option in


pregnancy because of the fact that thyroxine doesn’t cross the placenta. High-dose
carbimazole used in this regimen would run the risk of causing neonatal hypothyroidism.

C Propranolol
Propranolol is incorrect. Symptomatic treatment alone is inappropriate for thyrotoxicosis in
pregnancy because it doesn’t minimise any increased risk of spontaneous miscarriage.

D Propylthiouracil

Propylthiouracil is incorrect. Propylthiouracil used later in pregnancy is associated with


increased maternal risk of hepatic dysfunction. As such, it isn’t an appropriate intervention in
this case. There are other, more suitable, options available.

E Propylthiouracil and thyroxine

Propylthiouracil and thyroxine is incorrect. Propylthiouracil is not advised in later pregnancy


due to increasing risks of hepatic side effects. Although not contraindicated (can use with
close LFTs monitoring), it is not the ‘most appropriate’ option in this scenario. If used the
high dose ‘block and replace’ regimen is not recommended as PTU crosses the placenta and
could result in fetal goitre and hypothyroidism.
51148

Rate this question:

Next Question

Previous Question Tag Question

Feedback End Review

Difficulty: Average

Peer Responses %

Q. Answered Flagged

Q1

Q2
Q. Answered Flagged

Q3

Q4

Q5

Q6

Q7

Q8 
0:00:14/03:00:00

A 45-year-old man is reviewed in the Hepatology Clinic as an urgent appointment. He is


feeling unwell with increased abdominal pain and a small increase in abdominal swelling. He
has been started on tramadol for the pain some 2 days earlier by the on-call GP. Other
medication of note includes amitriptyline at night, B vitamin replacement and spironolactone.
On examination, his BP is 100/70 mmHg, pulse is 85 bpm and regular. There is mild
abdominal distension, with signs of chronic liver disease, including ascites.
Which of the following is of most concern?

A Decreased effectiveness of amitriptyline

B Decreased effectiveness of tramadol

C Increased risk of serotonin syndrome

D Increased risk of neuroleptic malignant syndrome

E Opiate toxicity

Explanation 

C Increased risk of serotonin syndrome

As well as being an opiate agonist, tramadol is recognised to increase release of serotonin.


When it is used in combination with anti-depressants, including amitriptyline, there is the
potential for this to lead to the development of serotonin syndrome.

A Decreased effectiveness of amitriptyline

Decreased effectiveness of amitriptyline is incorrect. Tramadol does not affect the


metabolism of amitriptyline; if anything, enhanced serotonin release by tramadol may further
improve symptoms of depression.

B Decreased effectiveness of tramadol

Decreased effectiveness of tramadol is incorrect. Tramadol is metabolised by CYP3A4 and


CYP2D6, amitriptyline is not a CYP450 inducer or inhibitor.
D Increased risk of neuroleptic malignant syndrome

Increased risk of neuroleptic malignant syndrome is incorrect. Neuroleptic malignant


syndrome is related to dopamine receptor antagonists, not to increased release of serotonin.

E Opiate toxicity

Opiate toxicity is incorrect. Opiate toxicity may occur when tramadol is co-prescribed with
CYP3A4 inhibitors such as imidazole anti-fungals or macrolides. It can be used in patients
with liver disease, but lower doses should be considered.
51154

Rate this question:

Next Question

Previous Question Tag Question

Feedback End Review

Difficulty: Average

Peer Responses %

Q. Answered Flagged

Q1

Q2

Q3

Q4

Q5
Q. Answered Flagged

Q6

Q7

Q8 
0:00:14/03:00:00

A 22-year-old woman presents to the Emergency Department having fainted whilst at work.
This is the third occasion that this has happened over the last 6 months. She has no past
medical history of note and takes no regular medication. On examination, her BP is 100/70
mmHg, pulse is 70 bpm and regular. Heart sounds are normal, chest is clear, abdomen is soft
and non-tender with no masses.
Investigations:

Hb 120 g/l

WCC 6.7 x 10 9/l

PLT 203 x 10 9/l

Na + 144 mmol/l

K+ 3.0 mmol/l

Bicarbonate 34 mmol/l

Cr 90 micromol/l

Renin is increased.

Urinary calcium:creatinine ratio 0.15 mmol/mmol


Which of the following is the most likely diagnosis?

A Bartter syndrome

B Gitelman syndrome

C Hyporeninaemic hyperaldosteronism

D Liddle syndrome

E Renal artery stenosis

Explanation 

B Gitelman syndrome
Gitelman syndrome is associated with normal or low blood pressure, and hypokalaemic
metabolic alkalosis. The difference with Bartter’s is that Gitelman is much more likely to
present in adulthood and is associated with low calcium excretion. Potassium
supplementation, potassium-sparing diuretics and non-steroidal anti-inflammatory drugs
(NSAIDs) have all been used as potential treatments for Gitelman. Gitelman is usually caused
by mutations in the SLC12A3 gene encoding the thiazide-sensitive sodium chloride
transporter.

A Bartter syndrome

Bartter syndrome is incorrect. Bartter syndrome is not associated with hypocalciuria, and is
much more likely to present in childhood with signs of vascular volume depletion.

C Hyporeninaemic hyperaldosteronism

Hyporeninaemic hyperaldosteronism is incorrect. Primary hyperaldosteronism is associated


with hypertension, rather than the normal blood pressure seen here.

D Liddle syndrome

Liddle syndrome is incorrect. Liddle syndrome is caused by a mutation in the epithelial


sodium channel and presents with marked hypertension from a very young age.

E Renal artery stenosis

Renal artery stenosis is incorrect. Renal artery stenosis is associated with hypertension and
low/normal potassium rather than the normal blood pressure seen here.
51174

Rate this question:

Next Question

Previous Question Tag Question

Feedback End Review

Difficulty: Average

Peer Responses %
Q. Answered Flagged

Q1

Q2

Q3

Q4

Q5

Q6

Q7

Q8

Q9

0:00:14/03:00:00

You are examining the results of a small open-label experimental medicine trial to reduce
appetite in people who are overweight which lasted for 4 weeks. In a trial of 20 actively
treated subjects versus 20 subjects who were given a standard over-the-counter therapy,
there was a small significant difference in favour of the new therapy. You suspect that this is
type I statistical error.
Which of the following is most likely to be the cause of the type 1 error here?

A The 4-week study duration

B Lack of blinding

C Lack of effect of existing treatment

D Small effect size of new therapy

E Small sample size

Explanation 

B Lack of blinding

Type 1 error refers to incorrectly rejecting the null hypothesis, (or a false positive result). In
this case, it’s likely the lack of blinding may have contributed to the effect size seen in
patients taking the new treatment, leading to a false positive study. It illustrates the
importance of blinding this type of trial.

A The 4-week study duration

The 4-week study duration is incorrect. Although therapies for appetite suppression lose
their effect over time, that’s irrelevant here. Much more important as a cause of type 1 error is
the fact that the therapies are not blinded.

C Lack of effect of existing treatment

Lack of effect of existing treatment is incorrect. Although the existing treatment may be less
effective, the perception it is less effective is more likely to be driven by the unblinding.
D Small effect size of new therapy

Small effect size of new therapy is incorrect. Small effect size is more likely to be associated
with a type 2 error, where a positive treatment effect is effectively missed.

E Small sample size

Small sample size is incorrect. Small sample sizes in clinical trials are also more likely to be a
cause of type 2, rather than type 1 error, where a positive effect of a drug is missed because
the trial is under-powered.
51172

Rate this question:

Next Question

Previous Question Tag Question

Feedback End Review

Difficulty: Average

Peer Responses %

Q. Answered Flagged

Q1

Q2

Q3

Q4

Q5
Q. Answered Flagged

Q6

Q7

Q8 
0:00:14/03:00:00

A 41-year-old woman is assessed in the Rheumatology Clinic for a possible diagnosis of


rheumatoid arthritis. She has suffered from multiple joint pains over the past 4 weeks and is
now unable to continue her job as a piano teacher because of joint swelling and pain. On
examination, her BP is 112/80 mmHg, pulse is 68 bpm and regular. Chest is clear. Abdomen is
soft and non-tender and you note rheumatoid nodules over both elbows.
Investigations:

Hb 119 g/l

WCC 8.1 x 10 9/l

PLT 329 x 10 9/l

Na + 140 mmol/l

K+ 4.5 mmol/l

Cr 88 mmol/l

ESR 62 mm/hr

CRP 89 mg/l

Anti-CCP and rheumatoid factor positive

Which of the above features most stratifies aggressive joint destruction in this patient?

A Acute onset

B Anti-CCP positivity

C CRP 89 µmol/l

D ESR 62 mm/hr

E Rheumatoid factor positivity

Explanation 

B Anti-CCP positivity
In 273 patients with rheumatoid arthritis who underwent serial radiographs over 6 years, anti-
CCP antibody positivity versus negativity was predictive of significantly more radiological
damage, independent of rheumatoid factor status.

A Acute onset

Acute onset is incorrect. Acute onset of joint symptoms, when managed appropriately, are
not necessarily predictive of aggressive disease.

C CRP 89 µmol/l

C-reactive protein (CRP) 89 µmol/l is incorrect. CRP merely implies active inflammation and
does not necessarily imply a poor long-term outlook.

D ESR 62 mm/hr

Erythrocyte sedimentation rate (ESR) 62 mm/hr is incorrect. Like CRP, ESR merely implies
active inflammation at the onset of disease and is a poor predictor of aggressive disease over
the long term.

E Rheumatoid factor positivity

Rheumatoid factor positivity is incorrect. Although rheumatoid factor positivity predicts


outcomes in rheumatoid arthritis, anti-CCP positivity incrementally predicts aggressive
disease. Hence, it’s anti-CCP which is more important with respect to predicting long-term
outcome.
51222

Rate this question:

Next Question

Previous Question Tag Question

Feedback End Review

Difficulty: Average

Peer Responses %
Q. Answered Flagged

Q1

Q2

Q3

Q4

Q5

Q6

Q7

Q8

Q9

 External Links

The prognostic value of anti-cyclic citrullinated peptide antibody in patients with recent-onset rheumatoi…
ncbi.nlm.nih.gov/pubmed/10943873
(https://www.ncbi.nlm.nih.gov/pubmed/10943873)
0:00:14/03:00:00

A 46-year-old woman with a history of sarcoidosis diagnosed after a routine chest X-ray
comes to the clinic for review. Over the past few months, she has become increasingly short
of breath and developed a persistent dry cough. Pulmonary function tests have revealed a
forced vital capacity which is 80% of predicted. Auscultation of the chest reveals scattered
coarse crackles throughout both lung fields.
Which of the following is the most appropriate intervention?

A Azathioprine

B High-dose inhaled fluticasone

C Infliximab

D Mycophenolate mofetil

E Oral prednisolone

Explanation 

E Oral prednisolone

Oral corticosteroids are the mainstay of initial therapy for sarcoidosis, although therapy is
reserved for patients who have symptomatic pulmonary disease (as is the case here), or who
have hypercalcaemia.

A Azathioprine

Azathioprine is incorrect. Azathioprine is a commonly used steroid-sparing agent for the


treatment of sarcoidosis, and is introduced as steroid therapy is weaned in patients with
symptomatic disease.

B High-dose inhaled fluticasone

High-dose inhaled fluticasone is incorrect. Inhaled corticosteroids are not effective in the
treatment of sarcoid-related lung disease.
C Infliximab

Infliximab is incorrect. Anti-tumour necrosis factor agents are used in the treatment of
sarcoidosis which fails to respond to prednisolone and a conventional steroid-sparing agent
alone, or where agents such as azathioprine and mycophenolate are unsuitable or
contraindicated. Data also support the use of rituximab in some patients with sarcoidosis.

D Mycophenolate mofetil

Mycophenolate mofetil is incorrect. Mycophenolate mofetil is an alternative steroid-sparing


agent to azathioprine in the treatment of sarcoidosis. Other options include methotrexate
and leflunomide.
51215

Rate this question:

Next Question

Previous Question Tag Question

Feedback End Review

Difficulty: Average

Peer Responses %

Q. Answered Flagged

Q1

Q2

Q3

Q4
Q. Answered Flagged

Q5

Q6

Q7

Q8 
0:00:14/03:00:00

A 57-year-old woman, who has been treated for rheumatoid arthritis for the past eight years,
comes to the Respiratory Clinic for review. Over the past six months, she has suffered from
increasing shortness of breath and is now unable to walk briskly to the bus stop.
On examination, her BP is 151/88 mmHg, pulse is 81 bpm and regular. There are coarse
crackles throughout both lung fields on auscultation. You note multiple rheumatoid nodules.
Chest X-ray reveals bilateral interstitial shadowing.
Which of the following is she most likely to have been treated with?

A Azathioprine

B Ciclosporin

C Infliximab

D Methotrexate

E Toclizumab

Explanation 

D Methotrexate

This patient is suffering from methotrexate-related lung fibrosis. There is little correlation
between dose and duration of therapy and presentation with symptoms, and patients may
often begin to suffer from significant fibrosis a number of years after commencing therapy.

A Azathioprine

Azathioprine is incorrect. Azathioprine is associated with interstitial pneumonia, although this


is rare and is reversible on cessation of therapy.

B Ciclosporin

Ciclosporin is incorrect. Ciclosporin is not associated with significant risk of respiratory


adverse events. The most common side-effect of concern is hirsuitism.
C Infliximab

Infliximab is incorrect. Infliximab is associated with increased risk of tuberculosis (TB) and
patients should be screened for TB before starting therapy. Anti-tumour necrosis factor
agents aren’t associated with pulmonary fibrosis.

E Toclizumab

Toclizumab is incorrect. Toclizumab is an anti-IL6 monoclonal antibody. It is most commonly


associated with upper respiratory tract infection, nasopharyngitis, headaches, hypertension
and increased transaminases.
51216

Rate this question:

Next Question

Previous Question Tag Question

Feedback End Review

Difficulty: Average

Peer Responses %

Q. Answered Flagged

Q1

Q2

Q3

Q4

Q5
Q. Answered Flagged

Q6

Q7

Q8 
0:00:14/03:00:00

A 47-year-old electrician comes to the clinic for review. He complains of pain and tingling in
his right hand which wakes him from sleep and causes him to hang his hand out of the bed as
the only way to relieve the symptoms. Tapping lightly at the wrist reproduces the tingling
over the median nerve distribution in the hand and there is weakness of the muscles of the
thenar eminence.
Which of the following is the most appropriate initial intervention?

A Acupuncture

B Gabapentin

C Local corticosteroid injection at the wrist

D Regular non-steroidal anti-inflammatory drugs (NSAIDs)

E Surgical carpal tunnel release

Explanation 

C Local corticosteroid injection at the wrist

Local corticosteroid injection is an appropriate initial intervention for carpal tunnel syndrome,
the diagnosis here. Studies suggest around 70% have a positive response at 1 month, and
those benefits are maintained for between 10 weeks and 1 year. Up to 75%, however, do
eventually progress to surgical release, with MRI scans that demonstrate greater nerve
compression predicting the need for surgery.

A Acupuncture

Acupuncture is incorrect. Acupuncture may relieve symptoms on top of other interventions,


but it shouldn’t be the sole modality of treatment.

B Gabapentin

Gabapentin is incorrect. Gabapentin may relieve symptoms of neuropathic pain, although not
the underlying nerve compression. As such, it is an inappropriate option here.
D Regular non-steroidal anti-inflammatory drugs (NSAIDs)

Regular NSAIDs is incorrect. Regular NSAIDs have been shown in studies to be no better than
placebo in the treatment of carpal tunnel syndrome. As such, they may play a role in pain
relief, but on top of other treatment modalities.

E Surgical carpal tunnel release

Surgical carpal tunnel release is incorrect. Surgical carpal tunnel release is usually reserved
for patients who fail to respond to steroid injection, although in patients with severe
compression of the median nerve on MRI, it may be an initial intervention.
51223

Rate this question:

Next Question

Previous Question Tag Question

Feedback End Review

Difficulty: Average

Peer Responses %

Q. Answered Flagged

Q1

Q2

Q3

Q4

Q5
Q. Answered Flagged

Q6

Q7

Q8

0:00:14/03:00:00

A 28-year-old woman who has a history of brittle asthma and has been admitted twice to the
Intensive Therapy Unit attends the Emergency Department with rapidly worsening shortness
of breath over the past few days. She takes a high-dose salmeterol/fluticasone combination
inhaler and her normal peak flow when well is 490 litres/min. You review her following back-
to-back salbutamol nebulisers which have been given for the last hour and one ipratropium
nebuliser. She has also been given intravenous (IV) coamoxiclav and IV hydrocortisone. On
examination, her BP is 110/80 mmHg, pulse is 105 bpm and regular. O 2 sats are 94% on 10
litres/min O 2. Peak flow is 200 litres/min. Respiratory rate is 30/min.

Which of the following is the most appropriate next intervention?

A IV aminophylline

B IV magnesium

C IV salbutamol

D Non-invasive ventilation

E Oral montelukast

Explanation 

B IV magnesium

In this case, with a peak flow less than 50% of normal, IV magnesium is indicated. Although
randomised controlled trial evidence is weak to support use of magnesium, this is thought to
be because the trial included a significant number of patients with peak expiratory flow rate
> 50% of normal. For this reason, the British Thoracic Society guidelines still support use of IV
magnesium, at a dose of 1.2–2.0 g, given as an IV bolus.

A IV aminophylline

IV aminophylline is incorrect. IV aminophylline is not thought to result in significant additive


effect over and above nebulised bronchodilators, and may cause significant vomiting and
hypokalaemia.
C IV salbutamol

IV salbutamol is incorrect. IV salbutamol, like IV aminophylline drives little added benefit over
and above nebulised bronchodilators.

D Non-invasive ventilation

Non-invasive ventilation (NIV) is incorrect. NIV is not recommended in the management of


acute asthma; in the event ventilatory support is required, intubation and ventilation are
preferred.

E Oral montelukast

Oral montelukast is incorrect. Oral montelukast is indicated in the treatment of chronic


asthma for patients who fail to gain control on inhaled corticosteroids or inhaled
corticosteroids plus Long-acting β adrenoceptor agonist (LABA). It may also be useful in
patients with predominantly exercise related symptoms.
51230

Rate this question:

Next Question

Previous Question Tag Question

Feedback End Review

Difficulty: Average

Peer Responses %

Q. Answered Flagged

Q1
Q. Answered Flagged

Q2

Q3

Q4

Q5

Q6

Q7

Q8 

 External Links

British guideline on the management of asthma


brit-thoracic.org.uk/document-library/clinical-information/asthma/btssign-asthma-guideline-2016/
(https://www.brit-thoracic.org.uk/document-library/clinical-
information/asthma/btssign-asthma-guideline-2016/)
0:00:14/03:00:00

A 34-year-old woman who is 28 weeks’ pregnant comes to the clinic for review. Her BP is
155/90 mmHg, having been 138/78 mmHg at booking. It has steadily increased over the past
few weeks. Her BMI is 27 kg/m 2 and abdominal examination is consistent with her stage of
pregnancy. Urine is negative for protein and a recent creatinine is measured at 88 micromol/l.
Which of the following is the most appropriate intervention for her blood pressure?

A Bendroflumethiazide

B Labetalol

C Methyldopa

D Nifedipine

E Ramipril

Explanation 

B Labetalol

There is extensive experience of anti-hypertensive use in pregnancy, and NICE guidelines


recommend labetalol as the first choice option unless there is a contra-indication to beta
blocker use. Reports of small for dates have been made in conjunction with labetalol
prescribing, although it’s unclear whether some of these are due to underlying microvascular
placental disease.

A Bendroflumethiazide

Bendroflumethiazide is incorrect. Thiazides are not recommended in pregnancy. Older


spontaneous case reports suggest they may be associated with increased risk of neonatal
thrombocytopaenia and electrolyte abnormalities.

C Methyldopa

Methyldopa is incorrect. Methyldopa is an alternative to labetalol for the control of blood


pressure in pregnancy where beta blockers are not tolerated or are contraindicated.
D Nifedipine

Nifedipine is incorrect. Calcium channel antagonists have extensive data relating to their use
in pregnancy and there is no suggestion of increased risk of congenital malformations. They
are therefore a further alternative to labetalol according to NICE guidelines.

E Ramipril

Ramipril is incorrect. Ramipril is associated with renal tract abnormalities when used in
pregnancy, as are angiotensin receptor blockers.
51155

Rate this question:

Next Question

Previous Question Tag Question

Feedback End Review

Difficulty: Average

Peer Responses %

Q. Answered Flagged

Q1

Q2

Q3

Q4

Q5
Q. Answered Flagged

Q6

Q7

Q8 
0:00:14/03:00:00

You are examining the effect of novel anti-arrhythmics on conduction speed in the heart.
When thinking about conduction velocity, where in the heart is this greatest?

A Anterior left ventricle

B AV node

C Posterior right ventricle

D Purkinje fibres

E SA node

Explanation 

D Purkinje fibres

The conduction velocity is fastest in the Purkinje fibres, at around 2–3 m/s. This is thought to
be because of differential amounts of Cx40, a connexin protein which sits at the level of gap
junctions between cardiac muscle cells in the Purkinje fibres, compared with other structures
within the heart.

A Anterior left ventricle

Anterior left ventricle is incorrect. Standard myocardial cells conduct on average eight times
more slowly in standard heart muscle cells, (around 0.3–0.4 m/s).

B AV node

AV node is incorrect. The AV node is partially responsible for generating heart rhythm, it
therefore generates a single stimulus and recharges during ventricular filling.

C Posterior right ventricle

Posterior right ventricle is incorrect. Like the anterior left ventricle, cells only conduct at
around 0.3–0.4 m/s.
E SA node

SA node is incorrect. The SA node is responsible for initiation of the heartbeat; it therefore
generates a single stimulus and then recharges during atrial filling.
51166

Rate this question:

Next Question

Previous Question Tag Question

Feedback End Review

Difficulty: Average

Peer Responses %

Q. Answered Flagged

Q1

Q2

Q3

Q4

Q5

Q6

Q7

Q8

Q9

0:00:14/03:00:00

A 49-year-old woman comes to the Hepatology Clinic for review. She has type 2 diabetes
(currently controlled with diet and exercise), hypertension, dyslipidaemia and a recent liver
biopsy has demonstrated fatty liver, inflammation and fibrosis consistent with F1 non-
alcoholic steatohepatitis (NASH). On examination, her BP is 148/92 mmHg, pulse is 65 bpm
and regular. She is obese, with a body mass index of 35 kg/m 2. Blood testing has revealed
elevated transaminases. Diets have so far failed to achieve more than 1–2kg of weight
reduction.
Which of the following is the most appropriate intervention?

A Atorvastatin 30 mg OD

B Fenofibrate 48 mg OD

C Liraglutide 3 mg OD

D Metformin 500 mg BD

E Pioglitazone 30 mg OD

Explanation 

C Liraglutide 3 mg OD

This patient is obese with early NASH, which is likely to progress. Substantial weight loss
does slow progression of NASH, and liraglutide 3 mg is licensed for the treatment of obesity
where there are concomitant risk factors such as hypertension and type 2 diabetes. Trials of
GLP1 agonists in the treatment of NASH are ongoing.

A Atorvastatin 30 mg OD

Atorvastatin 30 mg OD is incorrect. Although statins do substantially reduce low-density


lipoprotein (LDL) cholesterol, they have no benefit on progression of NASH.

B Fenofibrate 48 mg OD
Fenofibrate 48 mg OD is incorrect. Fibrates are PPAR-a agonists and have modest impact on
hepatic steatosis, although this isn’t of enough clinical benefit versus other options such as
liraglutide.

D Metformin 500 mg BD

Metformin 500 mg BD is incorrect. Although metformin does result in very modest weight
reduction and improved glycaemic control, any benefit is unlikely to be large enough to
reduce progression of NASH.

E Pioglitazone 30 mg OD

Pioglitazone 30 mg OD is incorrect. Pioglitazone does reduce intra-hepatic fat, but over the
long term, it has been shown to reduce bone mineral density, cause fluid retention and
increase weight, which, in a relatively young woman of 49 years, would be best avoided.
51202

Rate this question:

Next Question

Previous Question Tag Question

Feedback End Review

Difficulty: Average

Peer Responses %

Q. Answered Flagged

Q1

Q2
Q. Answered Flagged

Q3

Q4

Q5

Q6

Q7

Q8
0:00:14/03:00:00

A 54-year-old woman is referred to the Neurology Clinic for assessment. She originally
presented to her GP with mild unsteadiness when walking, which has progressed over time to
dysarthria, tremor, nystagmus and a clearly ataxic gait. General physical examination is
unremarkable apart from the presence of a hard mass in the left breast, suspicious of a
carcinoma.
Which of the following autoantibodies is most likely to be found?

A Anti-amphiphysin

B Anti-ANNA3

C Anti-Hu

D Anti-Tr

E Anti-Yo

Explanation 

E Anti-Yo

This woman presents with paraneoplastic cerebellar degeneration which has a recognised
association with breast cancer. The subacute presentation seen here is typical of the
condition, and patients may be unaware of symptoms from their underlying malignancy.

A Anti-amphiphysin

Anti-amphiphysin is incorrect. Anti-amphiphysin antibodies are seen in association with stiff


person syndrome and encephalomyelitis, and may be related to underlying breast or lung
cancer.

B Anti-ANNA3

Anti-ANNA3 is incorrect. ANNA3 antibodies are associated with sensory neuropathy and
encephalomyelitis, and may be seen in patients with underlying Hodgkin’s lymphoma.
C Anti-Hu

Anti-Hu is incorrect. Hu antibodies are a cause of encephalomyelitis and cerebellar


degeneration, but they are more commonly seen in small cell lung cancer rather than breast
cancer.

D Anti-Tr

Anti-Tr is incorrect. Anti-Tr antibodies are seen in cerebellar degeneration, although they are
associated with Hodgkin’s lymphoma, not with breast cancer.
51188

Rate this question:

Next Question

Previous Question Tag Question

Feedback End Review

Difficulty: Average

Peer Responses %

Q. Answered Flagged

Q1

Q2

Q3

Q4

Q5
Q. Answered Flagged

Q6

Q7

Q8 
0:00:14/03:00:00

A 34-year-old man presents with an erythematous scaly rash affecting his scalp and the folds
around his nose and upper lip. He has also noted fine skin scaling over both eyebrows which
has worsened significantly over the past few months. He has no past medical history of note
and takes no regular medications. He consents to an HIV test, which is negative.
Which of the following is the most appropriate initial intervention?

A Oral isotretinoin

B Oral oxytetracycline

C Topical ketoconazole

D Topical tacrolimus

E UV light therapy

Explanation 

C Topical ketoconazole

The most likely diagnosis here is seborrheic dermatitis. Anti-fungal ketoconazole shampoo is
the most appropriate initial treatment for this patient’s scalp rash, and cream preparations
are an option for the areas around his nose.

A Oral isotretinoin

Oral isotretinoin is incorrect. Isotretinoin is an off-license medication with respect to


treatment of seborrheic dermatitis and is only considered if patients fail to respond to other
interventions such as topical ketoconazole and topical corticosteroids.

B Oral oxytetracycline

Oral oxytetracycline is incorrect. Tetracyclines are more commonly used as a treatment for
rosacea rather than seborrheic dermatitis. Oxytetracycline may be used in patients with
disease resistant to other interventions.
D Topical tacrolimus

Topical tacrolimus is incorrect. Topical tacrolimus is the next intervention for patients who fail
to respond to topical ketoconazole and corticosteroids alone.

E UV light therapy

UV light therapy is incorrect. UV light therapy is a commonly used treatment for psoriasis; it
is also used in seborrheic dermatitis for patients who fail to respond to other interventions.
51140

Rate this question:

Next Question

Previous Question Tag Question

Feedback End Review

Difficulty: Average

Peer Responses %

Q. Answered Flagged

Q1

Q2

Q3

Q4

Q5

Q6
Q. Answered Flagged

Q7

Q8

0:00:14/03:00:00

A 59-year-old man is reviewed in the clinic after presenting some 5 weeks earlier with a left
ureteric stone which he eventually passed successfully. He has a history of two previous
attacks of gout which he has controlled with intermittent ibuprofen treatment. A CT scan of
the renal tract has revealed evidence of multiple renal stones. On examination, his BP is
148/88 mmHg, pulse is 72 bpm and regular. Abdomen is soft and non-tender. Uric acid tested
the week before clinic is elevated at 580 mmol/l.
Which of the following treatments should be avoided in treatment of his gout?

A Allopurinol

B Colchicine

C Febuxostat

D Lesinurad

E Prednisolone

Explanation 

D Lesinurad

Lesinurad inhibits URAT1 and OAT4, leading to increased excretion of uric acid. It is
associated with a small rise in creatinine in some patients on initiation of therapy, and may
lead to formation of new renal stones. It should therefore be avoided in this patient.

A Allopurinol

Allopurinol is incorrect. Allopurinol is a xanthine oxidase inhibitor which reduces formation of


uric acid. It is therefore appropriate for the treatment of gout, including in patients with renal
stones.

B Colchicine
Colchicine is incorrect. Colchicine inhibits microtubule polymerisation and acts as an anti-
inflammatory agent. It is used at high dose in the treatment of acute gout, and can be used at
lower doses for maintenance therapy when other options are not appropriate. It does not
increase the risk of renal stone formation.

C Febuxostat

Febuxostat is incorrect. Febuxostat is an alternative xanthine oxidase inhibitor to allopurinol.


It is usually considered a second-choice agent because of an imbalance against febuxostat in
phase 3 trials with respect to major adverse cardiovascular events.

E Prednisolone

Prednisolone is incorrect. Prednisolone is a reasonable option for treatment of acute gout in


this situation. Over the longer term, it wouldn’t usually be considered to have positive benefit
risk because of weight gain, dysglycaemia and osteoporosis.
51175

Rate this question:

Next Question

Previous Question Tag Question

Feedback End Review

Difficulty: Average

Peer Responses %

Q. Answered Flagged

Q1

Q2
Q. Answered Flagged

Q3

Q4

Q5

Q6

Q7

Q8 
0:00:14/03:00:00

A 45-year-old woman with a history of systemic lupus erythematosus (SLE) comes to the
clinic for review. Her symptoms had previously been well controlled with hydroxychloroquine,
although over the past few months, she has noticed worsening joint pains and swelling
affecting both lower limbs. On examination, you note the typical facial rash of SLE. Her BP is
155/92 mmHg, pulse is 69 bpm and regular. Chest is clear. Abdomen is soft and non-tender.
There is pitting oedema affecting both lower limbs up to the knees. The 24-h urinary albumin
excretion is 5.2 g. Creatinine is stable at 105 mmol/l. Prednisolone is commenced.
Which of the following is the most appropriate intervention?

A Azathioprine

B Cyclophosphamide

C Methotrexate

D Mycophenolate

E Infliximab

Explanation 

D Mycophenolate

This patient has nephrotic syndrome as a result of lupus nephritis. In this situation,
mycophenolate mofetil is the initial intervention of choice; 0.5 g is given twice per day for the
first week, with a dose increase of 0.5 g per dose each week, until a dose of 1.5 g BD is
reached. A period of 6 months of initial therapy is usual.

A Azathioprine

Azathioprine is incorrect. Azathioprine is used to treat SLE, although mycophenolate is


preferred for the treatment of patients with renal involvement.

B Cyclophosphamide
Cyclophosphamide is incorrect. Cyclophosphamide is the usual option in patients who are
unable to tolerate mycophenolate therapy. A dosage of 500 mg intravenously every 2 weeks
for a total of 6 weeks is the usual initial regimen for cyclophosphamide. Oral
cyclophosphamide is used by some nephrologists at a dose of 1.0–1.5 mg/day for no more
than 6 months.

C Methotrexate

Methotrexate is incorrect. Methotrexate is the standard intervention for rheumatoid and


psoriatic arthritis, rather than SLE-related membranous nephropathy.

E Infliximab

Infliximab is incorrect. Infliximab is not an initial option for SLE-related membranous


nephropathy; a lupus-like syndrome has been reported in some patients where anti-TNF
therapy is used, although some patients are seen to benefit.
51176

Rate this question:

Next Question

Previous Question Tag Question

Feedback End Review

Difficulty: Average

Peer Responses %

Q. Answered Flagged

Q1

Q2
Q. Answered Flagged

Q3

Q4

Q5

Q6

Q7

Q8 
0:00:14/03:00:00

A 58-year-old man is reviewed on the Cardiology Ward, some 24 h after admission, with an
anterior myocardial infarction. The nursing staff are concerned because of a fall-off in his
urine output over the past few hours. You understand he had a period of hypotension at the
time of presentation. On examination, his BP is 138/82 mmHg, pulse is 88 bpm and regular.
His jugular vein pulse (JVP) is just visible. Chest is clear. Abdomen is soft and non-tender and
there is no ankle swelling. Over the past few hours, he has only passed 35 ml of urine. Urinary
sodium is increased and his creatinine has increased from 90 mmol/l on admission to 140
mmol/l now.

Which of the following is the most likely diagnosis?

A Acute tubular necrosis

B Obstructive uropathy

C Pre-renal failure

D Renal artery embolus

E Renal vein thrombosis

Explanation 

A Acute tubular necrosis

The period of hypotension associated with the myocardial infarction, reduced urine output,
increased creatinine and increased urinary sodium all fit with a diagnosis of acute tubular
necrosis. Although it was previously thought both furosemide and dopamine may show renal
protection, there is little evidence to support this view. Maintaining equipoise with respect to
fluid balance is the intervention of choice.

B Obstructive uropathy

Obstructive uropathy is incorrect. Although urinary tract obstruction would present with an
increase in creatinine and a fall-off in urine output, the increased urinary sodium is
inconsistent with this as the diagnosis.
C Pre-renal failure

Pre-renal failure is incorrect. Hypotension and hypovolaemia can lead to a fall-off in urine
output, although the increased urinary sodium isn’t seen in pre-renal failure.

D Renal artery embolus

Renal artery embolus is incorrect. Renal artery embolism is unusual, especially in the absence
of atrial fibrillation. Flank/loin pain and a marked increase in urinary protein excretion would
be expected.

E Renal vein thrombosis

Renal vein thrombosis is incorrect. Renal vein thrombosis is seen against a background of
prothrombotic tendency and is associated with vague flank/loin pain and proteinuria.
51177

Rate this question:

Next Question

Previous Question Tag Question

Feedback End Review

Difficulty: Average

Peer Responses %

Q. Answered Flagged

Q1

Q2
Q. Answered Flagged

Q3

Q4

Q5

Q6

Q7

Q8 
0:00:14/03:00:00

A 38-year-old woman with a history of severe asthma and allergic rhinitis is unable to get her
steroid dose down to below 10 mg per day oral prednisolone without a return of her
symptoms. Other medication includes high-dose inhaled fluticasone/salmeterol and
montelukast. On examination, her BP is 122/82 mmHg, pulse is 74 bpm and regular. There is
bilateral quiet wheeze on auscultation of the chest. Peak flow is 490 litres/min. Bloods are
unremarkable, apart from a raised immunoglobulin E (IgE) and patch testing reveals multiple
allergies to trees, grasses, cats, fungus and moulds.
Which of the following is the most appropriate next intervention?

A Inhaled tiotropium

B Subcutaneous omalizumab

C Pulsed intravenous (IV) corticosteroid therapy

D Subcutaneous immunotherapy

E Sublingual immunotherapy

Explanation 

B Subcutaneous omalizumab

Omalizumab is a humanised monoclonal antibody which binds to circulating IgE, reducing


levels of free serum IgE. Key factors which drive use of omalizumab include persistent
symptoms against a proven background of allergy, elevated IgE, and maximal of conventional
therapies for asthma. It is given as a subcutaneous injection and dosed according to weight.

A Inhaled tiotropium

Inhaled tiotropium is incorrect. Inhaled tiotropium is a long-acting anti-cholinergic used in the


management of chronic obstructive pulmonary disease.

C Pulsed intravenous (IV) corticosteroid therapy


Pulsed IV corticosteroid therapy is incorrect. Pulsed IV steroid therapy offers little
therapeutic advantage over and above oral steroids, and carries similar effects in reducing
bone mineral density over the longer terms.

D Subcutaneous immunotherapy

Subcutaneous immunotherapy is incorrect. Subcutaneous immunotherapy is not effective in


improving symptoms of asthma versus conventional therapies, although it may be of value in
allergic rhinitis.

E Sublingual immunotherapy

Sublingual immunotherapy is incorrect. Sublingual immunotherapy is effective in the


treatment of allergy and has less systemic side effects versus subcutaneous delivered
antigens. It is, however, less effective than conventional therapy in the treatment of asthma.
51231

Rate this question:

Next Question

Previous Question Tag Question

Feedback End Review

Difficulty: Average

Peer Responses %

Q. Answered Flagged

Q1

Q2
Q. Answered Flagged

Q3

Q4

Q5

Q6

Q7

Q8

 External Links

British guideline on the management of asthma


brit-thoracic.org.uk/document-library/clinical-information/asthma/btssign-asthma-guideline-2016/
(https://www.brit-thoracic.org.uk/document-library/clinical-
information/asthma/btssign-asthma-guideline-2016/)
0:00:14/03:00:00

A 34-year-old woman comes to the Obstetric and Gynaecology Clinic for review. She is 16
weeks’ pregnant with her first child and has just recovered from an episode of genital herpes.
What is the correct advice with respect to managing her to the point of delivery?

A No intervention is required

B Prophylactic acyclovir should be started at week 36

C She should be treated with acyclovir until delivery

D She should have a caesarean at 38 weeks

E The child should be treated for the first 2 weeks with acyclovir

Explanation 

B Prophylactic acyclovir should be started at week 36

For all women with a genital herpes lesion occurring during pregnancy, be it a primary or a
recurrent lesion, suppressive therapy with acyclovir at week 36 is recommended. A dosage of
400 mg acyclovir TDS is the usual recommendation.

A No intervention is required

No intervention is required is incorrect. Not intervening after week 36 runs the risk of
neonatal herpes infection during delivery and is therefore not advised.

C She should be treated with acyclovir until delivery

She should be treated with acyclovir until delivery is incorrect. Even recurrent infection is not
usually treated with continuous acyclovir, although where patients have recurrent episodes,
acyclovir is started a week earlier at week 35.

D She should have a caesarean at 38 weeks


She should have a Caesarean at 38 weeks is incorrect. Caesareans need only be offered to
women with prodromal symptoms or active lesions at the time of delivery.

E The child should be treated for the first 2 weeks with acyclovir

The child should be treated for the first 2 weeks with acyclovir is incorrect. It’s much better to
prevent neonatal infection with prophylaxis for the mother than to risk infection in the child.
51212

Rate this question:

Next Question

Previous Question Tag Question

Feedback End Review

Difficulty: Average

Peer Responses %

Q. Answered Flagged

Q1

Q2

Q3

Q4

Q5

Q6

Q7
Q. Answered Flagged

Q8
0:00:14/03:00:00

A 27-year-old woman presents to the Emergency Department with symptoms of a UTI. She is
28 weeks’ pregnant. A urine specimen is positive for Escherichia Coli. Examination reveals a
temperature of 37.8 °C, she has supra-pubic tenderness and routine blood testing reveals a
neutrophilia.
Which of the following antibiotics would you avoid using as a treatment for the UTI?

A Amoxicillin

B Cephalexin

C Ciprofloxacin

D Nitrofurantoin

E Trimethoprim

Explanation 

C Ciprofloxacin

Ciprofloxacin, like the quinolone class as a whole, is contra-indicated in pregnancy. This is


because of an association in juvenile animal models with tendon rupture. This is seen in adults
when quinolones are co-prescribed with corticosteroids.

A Amoxicillin

Amoxicillin is incorrect. Penicillins are considered a safe option in pregnancy. There is also no
evidence that clavulanic acid, a constituent of co-amoxiclav, significantly worsens the benefit
risk profile of the class.

B Cephalexin

Cephalexin is incorrect. Cephalosporins are considered an appropriate option in pregnancy


for the treatment of UTI where other options, such as nitrofurantoin, are unsuitable.

D Nitrofurantoin
Nitrofurantoin is incorrect. Nitrofurantoin is generally considered safe in pregnancy and is
widely prescribed as a first choice option for the treatment of UTI.

E Trimethoprim

Trimethoprim is incorrect. Trimethoprim may affect folate metabolism and is not


recommended in early pregnancy because of this. It may however be safely used in later
pregnancy.
51156

Rate this question:

Next Question

Previous Question Tag Question

Feedback End Review

Difficulty: Average

Peer Responses %

Q. Answered Flagged

Q1

Q2

Q3

Q4

Q5

Q6
Q. Answered Flagged

Q7

Q8
0:00:14/03:00:00

A 62-year-old man comes to the Cardiology Clinic for review. He has recently been admitted
with an episode of pulmonary oedema and is noted previously by his GP to have
hypertension and ischaemic heart disease. Current medication includes ramipril 10 mg,
furosemide 40 mg, aspirin 75 mg and atorvastatin 10 mg. On examination, his BP 155/82
mmHg, pulse is 80 bpm and regular. There are bilateral basal crackles on auscultation of the
chest, with no ankle swelling.
Investigations:

Hb 129 g/l

WCC 6.1 x 10 /l
9

PLT 291 x 10 9/l

Na + 141 mmol/l

K+ 3.8 mmol/l

Cr 135 mmol/l

CT angiography abdomen – evidence of bilateral renal artery atheroma.

Which of the following is the most appropriate intervention?

A Amlodipine 5 mg daily

B Bilateral renal artery angioplasty and stenting

C Doxazosin 10 mg daily

D Reassurance

E Renal artery bypass

Explanation 

A Amlodipine 5 mg daily
In this situation, with only a single episode of pulmonary oedema, improved control of blood
pressure with further medical therapy is the intervention of choice. Angioplasty and stenting
has shown no advantage over medical therapy in the absence of recurrent oedema or a
progressive rise in creatinine.

B Bilateral renal artery angioplasty and stenting

Bilateral renal artery angioplasty and stenting is incorrect. Stenting is usually reserved for
patients with a progressive increase in serum creatinine despite adequate blood pressure
control, or there are recurrent episodes of pulmonary oedema.

C Doxazosin 10 mg daily

Doxazosin 10 mg daily is incorrect. Alpha blockers are not recommended in this situation for
blood pressure control because they may exacerbate left ventricular failure.

D Reassurance

Reassurance is incorrect. Reassurance is incorrect here given there has already been one
admission with left ventricular failure and the patient’s blood pressure is not yet at target.

E Renal artery bypass

Renal artery bypass is incorrect. Renal artery bypass is reserved for rare cases when renal
artery angioplasty is indicated but the arterial lesion is not amenable to stenting.
51178

Rate this question:

Next Question

Previous Question Tag Question

Feedback End Review

Difficulty: Average

Peer Responses %
Q. Answered Flagged

Q1

Q2

Q3

Q4

Q5

Q6

Q7

Q8

Q9

0:00:14/03:00:00

A 72-year-old man with a history of Parkinsonism and significant alcohol consumption is


admitted to the medical ward with acute confusion, thought to be related to a urinary tract
infection. He is constantly trying to get out of bed and has attempted to bite two of the
nursing staff. There is a significant risk of falling.
Which of the following is the most appropriate intervention to reduce the risk of falls?

A Chlorpromazine

B Cot sides

C Haloperidol

D Midazolam

E Olanzapine

Explanation 

D Midazolam

The complicating factor here is the history of Parkinson’s disease, which skews us away from
using a dopamine antagonist for the treatment of delirium. The best option is midazolam,
which can be titrated from very low doses of 0.5 mg, and has a rapid onset.

A Chlorpromazine

Chlorpromazine is incorrect. Chlorpromazine may significantly worsen any underlying


Parkinsonian movement disorder and should therefore be avoided in patients with a history
of the disease.

B Cot sides

Cot sides is incorrect. Cot sides are not advisable because they may actually increase the risk
of a serious fall because patients try and climb over them and sustain damage from a greater
height.
C Haloperidol

Haloperidol is incorrect. Like chlorpromazine, haloperidol is a typical dopamine antagonist.


As such, it significantly worsens Parkinson’s-related movement disorder, and should be
avoided here.

E Olanzapine

Olanzapine is incorrect. Atypical anti-psychotics like olanzapine are associated with a more
benign profile in Parkinson’s than older agents, although here, particularly with the history of
alcohol consumption, a benzodiazepine approach is preferred.
51224

Rate this question:

Next Question

Previous Question Tag Question

Feedback End Review

Difficulty: Average

Peer Responses %

Q. Answered Flagged

Q1

Q2

Q3

Q4

Q5
Q. Answered Flagged

Q6

Q7

Q8

0:00:14/03:00:00

A 32-year-old man comes to the Respiratory Clinic for review. He has been suffering from
worsening symptoms of asthma over the last 5 months, despite having had montelukast
introduced into his regimen at his last appointment. Other medication includes high-dose
salmeterol/fluticasone and he has required three doses of oral prednisolone in the last year.
Examination reveals a BP of 132/84 mmHg, pulse is 73 bpm and regular. Chest appears clear
apart from occasional wheeze on auscultation. Bloods reveal an elevated eosinophil count,
although other blood tests, including aspergillus precipitins are unremarkable.
Which of the following is the most useful intervention?

A Golimumab

B Infliximab

C Mepolizumab

D Omalizumab

E Vedolizumab

Explanation 

C Mepolizumab

Mepolizumab is an anti-IL5 monoclonal antibody that plays a role in the management of


severe asthma, which is associated with raised eosinophil count and isn’t responsive to
conventional therapies.

A Golimumab

Golimumab is incorrect. Golimumab is an anti-tumour necrosis factor (TNF) agent which is


used in the treatment of psoriasis and psoriatic arthritis, rather than asthma.

B Infliximab

Infliximab is incorrect. Like golimumab, infliximab acts via the TNF system and is used in the
treatment of a number of diseases, including arthritis and inflammatory bowel disease.
D Omalizumab

Omalizumab is incorrect. Omalizumab neutralises circulating immunoglobulin E (IgE) and has


a role to play in the management of patients with asthma not controlled on conventional
therapies who have elevated levels of IgE. It’s the eosinophil count that specifically drives us
towards mepolizumab here.

E Vedolizumab

Vedolizumab is incorrect. Vedolizumab is an antibody directed against a4-b7, an integrin


receptor which allows T cells to pass through the wall of the small bowel. It is used in the
treatment of inflammatory bowel disease.
51232

Rate this question:

Next Question

Previous Question Tag Question

Feedback End Review

Difficulty: Average

Peer Responses %

Q. Answered Flagged

Q1

Q2

Q3

Q4
Q. Answered Flagged

Q5

Q6

Q7

Q8 
0:00:14/03:00:00

A 56-year-old man with a history of type 2 diabetes comes to the clinic for review. Current
medication includes metformin 1 g BD and empagliflozin 25 mg daily. His HbA1c has drifted
up over the past few months to 64 mmol/mol. Creatinine is stable at 105 micromol/l. On
examination, his BP is 138/82 mmHg, pulse is 64 bpm and regular. His BMI is elevated at 35
kg/m 2. A decision is made to commence exenatide long acting.
Which of the following is the mode of action of exenatide?

A Alpha glucosidase inhibitor

B DPPIV inhibitor

C GLP-1 agonist

D PPAR-gamma activator

E SGLT-2 inhibitor

Explanation 

C GLP-1 agonist

Exenatide is a GLP-1 agonist, which acts at the level of the GLP-1 receptor to potentiate
glucose-dependent insulin release. It also delays gastric emptying and may have central
effects on mediating appetite. As a consequence, it is associated with improved HbA1c and
modest weight reduction.

A Alpha glucosidase inhibitor

Alpha glucosidase inhibitor is incorrect. Acarbose is an alpha glucosidase inhibitor, which


results in reduced GI absorption of starch. It has modest effects on HbA1c and weight
reduction.

B DPPIV inhibitor

DPPIV inhibitor is incorrect. Sitagliptin is an example of a DPPIV inhibitor, which blocks the
breakdown of endogenous incretin hormones including GLP-1.
D PPAR-gamma activator

PPAR-gamma activator is incorrect. Pioglitazone is a PPAR-gamma activator which results in


increased conversion of sugar to subcutaneous fat. It is also associated with fluid retention
and increased risk of bone fracture.

E SGLT-2 inhibitor

SGLT-2 inhibitor is incorrect. Empagliflozin is an example of an SGLT-2 inhibitor, which blocks


reabsorption of glucose, leading to a reduction in HbA1c and modest weight reduction. It is
also recognised to reduce mortality in cardiovascular outcome studies by up to 30%.
51157

Rate this question:

Next Question

Previous Question Tag Question

Feedback End Review

Difficulty: Average

Peer Responses %

Q. Answered Flagged

Q1

Q2

Q3

Q4

Q5
Q. Answered Flagged

Q6

Q7

Q8

0:00:14/03:00:00

A 43-year-old man comes to the Nephrology Clinic for review. His GP has noted an increase
in his creatinine from 95 mmol/l to 145 mmol/l over the past 2 months. He has also noted
blood and proteinuria, worsening asthma despite maximal inhaled steroid and β-2-agonist
therapy and an episode of common peroneal nerve palsy. On examination, his BP is 155/90
mmHg, pulse is 78 bpm and regular. There is bilateral wheeze on auscultation of the chest,
abdomen is soft and non-tender.
Which of the following is most likely to be found on investigation?

A Hypogammaglobulinaemia

B Positive anti-mitochondrial antibodies

C Positive anti-nuclear antibodies

D Positive c-ANCA

E Raised eosinophils

Explanation 

E Raised eosinophils

The history of worsening asthma, coupled with mononeuritis and evidence of renal vasculitis
fits well with a diagnosis of eosinophilic granulomatosis with polyangiitis. Elevated
eosinophils, anaemia and raised erythrocyte sedimentation rate (ESR) are seen, and positive
pANCA is seen in 30–40% of patients.

A Hypogammaglobulinaemia

Hypogammaglobulinaemia is incorrect. A polyclonal increase in globulins, rather than low


levels of immunoglobulins, is seen in patients with eosinophilic granulomatosis.

B Positive anti-mitochondrial antibodies

Positive anti-mitochondrial antibodies is incorrect. Primary anti-mitochondrial antibodies are


associated with autoimmune hepatitis and primary biliary cirrhosis.
C Positive anti-nuclear antibodies

Positive anti-nuclear antibodies is incorrect. Anti-nuclear antibodies are seen in association


with systemic lupus erythematosus, rather than eosinophilic granulomatosis, where pANCA is
more likely to be a feature.

D Positive c-ANCA

Positive c-ANCA is incorrect. Positive c-ANCA is seen in patients with granulomatosis with
polyangiitis, where worsening asthma and a raised eosinophil count are not seen.
51179

Rate this question:

Next Question

Previous Question Tag Question

Feedback End Review

Difficulty: Average

Peer Responses %

Q. Answered Flagged

Q1

Q2

Q3

Q4

Q5
Q. Answered Flagged

Q6

Q7

Q8 
0:00:14/03:00:00

A 25-year-old woman with a history of Hashimoto’s disease comes to the clinic for review.
She is 12 weeks’ pregnant with her first child. She was maintained on 100 mcg of thyroxine
per day prior to becoming pregnant. The pregnancy has gone fine so far.
What dose of thyroxine would you expect her to be maintained on in her last few weeks of
pregnancy?

A 50 mcg

B 75 mcg

C 100 mcg

D 150 mcg

E 200 mcg

Explanation 

D 150 mcg

Around 50% more thyroxine is required by the end of pregnancy to maintain the euthyroid
state. Levels of thyroxine and TSH should be checked every 8 weeks during the course of the
pregnancy.

A 50 mcg

50 mcg is incorrect. Given the increased body mass and hormonal changes associated with
pregnancy, a reduction in thyroid hormone dose risks driving significant maternal
hypothyroidism.

B 75 mcg

75 mcg is incorrect. Even a small reduction in thyroid hormone dose risks driving significant
hypothyroidism, taking into account the increased physiological demands of pregnancy.

C 100 mcg
100 mcg is incorrect. Maintaining a stable dose of thyroxine is inappropriate because of the
increased physiological demands of pregnancy. This risks maternal hypothyroidism and
consequent increased risk of pregnancy failure.

E 200 mcg

200 mcg is incorrect. Although a small number of patients may require a dose increase to
200 mcg, the average dose increase is 50%, or in this case, 50 mcg. Monitoring every 8
weeks should govern any dose increase.
51149

Rate this question:

Next Question

Previous Question Tag Question

Feedback End Review

Difficulty: Average

Peer Responses %

Q. Answered Flagged

Q1

Q2

Q3

Q4

Q5

Q6
Q. Answered Flagged

Q7

Q8

0:00:14/03:00:00

A 25-year-old woman who is 12 weeks’ pregnant comes to the Emergency Department for
review. She works in a pre-school where two children have just been diagnosed with hepatitis
A, having returned from a holiday to north Africa and is concerned about risks of infection?
Which of the following is the most appropriate intervention?

A Advice on avoidance of paracetamol

B Hepatitis A IgM testing

C Hepatitis A vaccination

D Liver function tests weekly

E Reassurance

Explanation 

C Hepatitis A vaccination

There is a 2-week window within which both hepatitis A vaccination and immunoglobulin
administration may reduce both the risk and severity of infection. Although serious sequelae
of hepatitis are rare in association with hepatitis A, they are more common in pregnancy,
hence there is a benefit of intervention.

A Advice on avoidance of paracetamol

Advice on avoidance of paracetamol is incorrect. There is no evidence that avoidance of


paracetamol would be of benefit in a patient with no symptoms of hepatic dysfunction. Even
in a woman with active infection, significant impact on hepatic synthetic function, and thus
paracetamol metabolism is unlikely.

B Hepatitis A IgM testing

Hepatitis A IgM testing is incorrect. Hepatitis A serology testing only lengthens the time
before potential successful intervention; as such, post-exposure prophylaxis is the more
appropriate next step.
D Liver function tests weekly

Hepatitis A IgM testing is incorrect. Hepatitis A serology testing only lengthens the time
before potential successful intervention; as such, post-exposure prophylaxis is the more
appropriate next step.

E Reassurance

Reassurance is incorrect. Reassurance alone is not appropriate given that post-exposure


prophylaxis against hepatitis A may reduce the risk of infection occurring.
51203

Rate this question:

Next Question

Previous Question Tag Question

Feedback End Review

Difficulty: Average

Peer Responses %

Q. Answered Flagged

Q1

Q2

Q3

Q4

Q5
Q. Answered Flagged

Q6

Q7

Q8 
0:00:14/03:00:00

A 45-year-old man presents with a 2–3-week history of worsening breathlessness, chills,


pleuritic chest pains. He has lost 4 kg in weight and over the past few days has begun to feel
increasingly nauseous. Just before he began to feel unwell, he suffered an episode of
influenza and apart from smoking ten cigarettes per day, he is otherwise well. On
examination, his BP is 165/100 mmHg, pulse is 89 bpm and regular and there are inspiratory
crackles over both lung bases. Respiratory rate is increased to 28/min and he looks cyanosed.
Investigations:

Hb 89 g/l

WCC 12.4 x 10 9/l

PLT 191 x 10 9/l

Na + 140 mmol/l

K+ 5.9 mmol/l

Cr 213 mmol/l

ESR 13mm/1 st hour

c-ANCA negative

Urine blood 3+, protein 1+

CXR Patchy bilateral consolidation

Which of the following is the most likely diagnosis?

A Anti-glomerular basement membrane (GBM) disease

B Eosinophilic granulomatosis with polyangiitis

C Granulomatosis with polyangiitis

D Microscopic polyangiitis

E Systemic lupus erythematosus (SLE)

Explanation 
A Anti-glomerular basement membrane (GBM) disease

The relatively rapid onset of pulmonary symptoms with glomerulonephritis, coupled with
normal erythrocyte sedimentation rate (ESR) and c-ANCA negativity are typical of anti-GBM
disease. The fact that there is no history of sinusitis (typically seen with granuloma formation
in granulomatosis with polyangiitis) is also supportive of the diagnosis. Plasmapheresis is the
intervention of choice.

B Eosinophilic granulomatosis with polyangiitis

Eosinophilic granulomatosis with polyangiitis is incorrect. Eosinophilic granulomatosis is


associated with elevated eosinophils and symptoms of worsening asthma, rather than the
more non-specific respiratory symptoms seen here and marked glomerulonephritis.

C Granulomatosis with polyangiitis

Granulomatosis with polyangiitis is incorrect. Granulomatosis with polyangiitis is often


associated with symptoms of sinusitis which may pre-date presentation with pulmonary and
renal disease, and is associated with elevated ESR.

D Microscopic polyangiitis

Microscopic polyangiitis is incorrect. The more rapid presentation seen here and the fact ESR
is essentially normal, counts against microscopic polyangiitis as the underlying diagnosis.

E Systemic lupus erythematosus (SLE)

Systemic lupus erythematosus (SLE) is incorrect. SLE is associated with a typical rash
affecting the face and sun-exposed areas, small-joint polyarthritis, and a less rapid
deterioration of symptoms versus those seen here. It is commoner in women than men.
51180

Rate this question:

Next Question

Previous Question Tag Question

Feedback End Review

Difficulty: Average

Peer Responses %
Q. Answered Flagged

Q1

Q2

Q3

Q4

Q5

Q6

Q7

Q8

Q9

0:00:14/03:00:00

A 32-year-old woman comes to the Emergency Department for review. She has suffered
progressive nausea and vomiting over the past few days and has noticed that morning that
she has become jaundiced. She is 36 weeks’ pregnant with her first child. On examination, her
BP is 122/82 mmHg, pulse is 75 bpm and regular. She has jaundiced sclerae and some bruises
on her arms and legs. Abdominal examination is consistent with a 36-week pregnancy, there
is vague right upper quadrant tenderness.
Investigations:

Hb 109 g/l

WCC 12.9 x 10 9/l

PLT 102 x 10 9/l

Na + 139 mmol/l

K+ 3.7 mmol/l

Cr 80 mmol/l

ALT 285 IU/l

Alkphos 180 IU/l

Bilirubin 54 mmol/l

INR 1.7

Which of the following is the most likely diagnosis?

A Acute fatty liver of pregnancy

B Cholelithiasis

C Intra-hepatic cholestasis of pregnancy

D Pre-eclampsia

E Viral hepatitis

Explanation 
A Acute fatty liver of pregnancy

The picture here, with acute hepatic deterioration after the 35 th week of pregnancy fits well
with a diagnosis of acute fatty liver. Rates of progression to fulminant liver failure are high,
early delivery is therefore recommended, (the condition resolves after delivery).

B Cholelithiasis

Cholelithiasis is incorrect. Gallstones are found in approximately 1 in 20 of women in


pregnancy, although only 5% of those suffer symptomatic jaundice. A greater elevation of
alkaline phosphatase with more marked right upper quadrant pain would be expected in
cholecystitis.

C Intra-hepatic cholestasis of pregnancy

Intra-hepatic cholestasis of pregnancy is incorrect. This presents with progressive lethargy


and itching after the 25 th week of pregnancy, jaundice is rarely seen, presenting 2–4 weeks
after the onset of itching.

D Pre-eclampsia

Pre-eclampsia is incorrect. Although pre-eclampsia presents with disordered LFTs and


clotting, the normal blood pressure here counts against pre-eclampsia.

E Viral hepatitis

Viral hepatitis is incorrect. Acute viral hepatitis (usually hepatitis A or hepatitis E) is seen in
pregnancy, but it is not usually associated with abnormalities of clotting.
51204

Rate this question:

Next Question

Previous Question Tag Question

Feedback End Review

Difficulty: Average

Peer Responses %
Q. Answered Flagged

Q1

Q2

Q3

Q4

Q5

Q6

Q7

Q8

Q9

0:00:14/03:00:00

A 77-year-old man is admitted to the Medical Ward having taken an overdose of tricyclic anti-
depressant tablets, after the death of his wife from Alzheimer’s disease. You are asked to see
him as he is trying to leave the ward. He tells you that he has prepared a noose at home and
intends to hang himself. He also tells you that everyone is trying to get him and he is hearing
voices telling him that he is worthless.
You plan to detain him under the mental health act. Which section is appropriate to detain
him under?

A 136

B 5(2)

C 5(4)

D 7

E 17A

Explanation 

B 5(2)

Section 5(2) allows the detention of a patient who lacks mental capacity for up to 72 hours in
the hospital. It requires the doctor in charge to apply to duty hospital management. It cannot
apply to the A&E Department as the patient has to be already in hospital.

A 136

136 is incorrect. Section 136 allows the police to arrest a person believed to be suffering from
a mental disorder, for up to 72 hours. The person must be conveyed to a place of safety, such
as a hospital, for further assessment.

C 5(4)

5(4) is incorrect. Section 5(4) allows for nurses to detain a patient who is already in hospital
for up to 6 hours for further assessment (they must be an authorised psychiatric nurse).
D 7

7 is incorrect. Section 7 is an application for guardianship to allow a patient to receive


community care without this needing to be under compulsory powers.

E 17A

17A is incorrect. Section 17A is a supervised compulsory treatment order, and can be used for
patients with long-term psychiatric conditions, particularly where they lack insight or have
refused intervention previously.
51225

Rate this question:

Next Question

Previous Question Tag Question

Feedback End Review

Difficulty: Average

Peer Responses %

Q. Answered Flagged

Q1

Q2

Q3

Q4

Q5
Q. Answered Flagged

Q6

Q7

Q8 
0:00:14/03:00:00

A 67-year-old man returns to the hospital some 3 days after discharge following admission
with shortness of breath and pleuritic chest pain. During his admission, CT pulmonary
angiography (CTPA) was performed, which was negative. On examination, his BP is 145/82
mmHg, pulse is 87 bpm and regular. His chest is clear, although respiratory rate is elevated at
25/min.
Investigations:

PaO 2 14.1 kPa

PaCO 2 3.3 kPa

HCO 3 13 mmol/l

pH 7.34

Which of the following is the most likely diagnosis?

A Partially compensated metabolic acidosis

B Partially compensated respiratory acidosis

C Decompensated metabolic acidosis

D Metabolic alkalosis

E Respiratory alkalosis

Explanation 

A Partially compensated metabolic acidosis

The suspicion here, given the history, is of contrast nephropathy leading to metabolic
acidosis. Evidence from the blood gases, with low CO 2 and low bicarbonate to maintain a pH
of 7.34, suggests respiratory compensation for the acidosis.

B Partially compensated respiratory acidosis


Compensated respiratory acidosis is incorrect. Given the fact that the CO 2 is low, increased
respiratory effort to compensate for the low bicarbonate is implied. In respiratory acidosis,
the bicarbonate would likely be normal or elevated against a background of raised CO 2.

C Decompensated metabolic acidosis

Decompensated metabolic acidosis is incorrect. The presentation would be similar to here,


although the pH would be markedly lower, without adequate respiratory effort to drive a
reduction in CO 2.

D Metabolic alkalosis

Metabolic alkalosis is incorrect. The bicarbonate reported here is low rather than high, which
in essence rules out a metabolic alkalosis.

E Respiratory alkalosis

Respiratory alkalosis is incorrect. Although the CO 2 is low, the pH is just below the mean
normal pH of 7.4, ruling out a respiratory alkalosis.
51195

Rate this question:

Next Question

Previous Question Tag Question

Feedback End Review

Difficulty: Average

Peer Responses %

Q. Answered Flagged
Q. Answered Flagged

Q1

Q2

Q3

Q4

Q5

Q6

Q7

Q8

0:00:14/03:00:00

A 57-year-old man with a history of psoriasis which is usually well controlled with topical
therapies comes to the clinic as an urgent review appointment. His psoriasis has significantly
worsened over the past few weeks since he was admitted as an Emergency having suffered
an inferior ST-Elevation Myocardial Infarction (STEMI). At the time, he was started on a
number of new medications.
Which of his current medications is of most concern with respect to worsening of psoriasis?

A Atenolol

B Atorvastatin

C Clopidogrel

D Indapamide

E Ramipril

Explanation 

A Atenolol

Beta blockers are well recognised to increase the severity of psoriatic lesions, which is of
significant concern given that after myocardial infarction, they are associated with an
outcome benefit. The latency period between drug exposure and the worsening of psoriasis
can vary from several days to up to 48 weeks.

B Atorvastatin

Atorvastatin is incorrect. Statins are most commonly associated with urticaria, itching and
hair loss; there is not a proven association with worsening of psoriasis.

C Clopidogrel

Clopidogrel is incorrect. Clopidogrel, as with other anti-platelets, is most likely to be


associated with bruising. Rarer dermatological conditions associated with clopidogrel include
bullous dermatitis and Stevens–Johnson syndrome.
D Indapamide

Indapamide is incorrect. Indapamide is most commonly associated with maculopapular


rashes, purpura, and areas of urticaria are less common.

E Ramipril

Ramipril is incorrect. Ramipril has been mentioned in case reports as worsening psoriasis,
although this is the subject of debate. The main concern with respect to ACE inhibitor use is
angioedema.
51141

Rate this question:

Next Question

Previous Question Tag Question

Feedback End Review

Difficulty: Average

Peer Responses %

Q. Answered Flagged

Q1

Q2

Q3

Q4

Q5
Q. Answered Flagged

Q6

Q7

Q8 
0:00:14/03:00:00

You are examining the effectiveness of a new addition to the range of anti-HIV medications. It
is an inhibitor of reverse transcriptase.
Which of the following is the function of reverse transcriptase?

A Transcription of DNA into mRNA

B Generation of cDNA from an RNA template

C Translation of mRNA to produce a polypeptide

D Loading of amino acids onto tRNA

E DNA synthesis from deoxyribonucleotides

Explanation 

B Generation of cDNA from an RNA template

Reverse transcriptase allows RNA viruses such as HIV to be incorporated into the genome. It
is responsible for generation of cDNA from an RNA template, and whilst it is mainly
associated with retroviruses, it is also found in hepatitis B.

A Transcription of DNA into mRNA

Transcription of DNA into mRNA is incorrect. DNA transcription occurs when DNA is read by
RNA polymerase to form a complementary strand of RNA. A single length of DNA may be
responsible for producing different proteins because of differences in the reading frame.

C Translation of mRNA to produce a polypeptide

Translation of mRNA to produce a polypeptide is incorrect. Translation of mRNA to produce


a polypeptide is a complex process involving both tRNA attached to individual amino acids,
and ribosomes.

D Loading of amino acids onto tRNA


Loading of amino acids onto tRNA is incorrect. The attachment of amino acids onto tRNA is
catalysed by aminoacyl tRNA synthetases.

E DNA synthesis from deoxyribonucleotides

DNA synthesis from deoxyribonucleotides is incorrect. This is the function of DNA


polymerases, which are essential for the process of DNA replication.
51191

Rate this question:

End Session

Previous Question Tag Question

Feedback End Review

Difficulty: Average

Peer Responses %

Q. Answered Flagged

Q1

Q2

Q3

Q4

Q5

Q6

Q7
Q. Answered Flagged

Q8

You might also like